Anda di halaman 1dari 221

Series

SeriesISSN:
Series ISSN:
ISSN:1939-5221
1939-5221
1939-5221

M
M

WHITMAN TERRY
WHITMAN
WHITMAN TERRY
SSSYNTHESIS
YNTHESIS L
LECTURES
YNTHESISL ECTURES ONE
ECTURESON
ON E
ENGINEERING
NGINEERING
NGINEERING
&C
& C Mor
Mor gan&
Morgan
gan &
& Cl
Claypool
Cl aypool
aypool Publishers
Publishers
Publishers

TERRY
Series
SeriesEditor:
Series Editor:
Editor:Steven
Steven
StevenF.F.F.Barrett,
Barrett,
Barrett,University
University
UniversityofofofWyoming
Wyoming
Wyoming
Fundamentals of
Fundamentals of
Fundamentalsof
Fundamentals
Fundamentals ofEngineering
of Engineering
Engineering
Engineering Economics
Engineering Economics

FUNDAMENTALS OF
FUNDAMENTALS
FUNDAMENTALS OF ENGINEERING
Economicsand
Economics
Economics andDecision
and DecisionAnalysis
Decision Analysis
Analysis
David
DavidL.
David
Ronald
L.
L.Whitman,
RonaldE.
Ronald E.
Whitman,
Whitman,University
E.Terry,
Terry,
University
UniversityofofofWyoming
Terry,Brigham
Brigham
BrighamYoung
Young
Wyoming
Wyoming
YoungUniversity
University
University
and Decision
and Decision Analysis
Analysis

OF ENGINEERING ECONOMICS
The
Theauthors
The authors
authorscover
cover
covertwo
two
twogeneral
general
generaltopics:
topics:
topics:basic
basic
basicengineering
engineering
engineeringeconomics
economics
economicsand and
andrisk
risk
riskanalysis
analysis
analysisinininthis
this
thistext.
text.
text.Within
Within
Within

ENGINEERING ECONOMICS AND


the
thetopic
the topic
topicofofofengineering
engineering
engineeringeconomics
economics
economicsare are
arediscussions
discussions
discussionson on
onthe
the
thetime
time
timevalue
value
valueofofofmoney
money
moneyand and
andinterest
interest
interestrelationships.
relationships.
relationships.
These
Theseinterest
These interest
interestrelationships
relationships
relationshipsare are
areused
used
usedtoto
todene
dene
denecertain
certain
certainproject
project
projectcriteria
criteria
criteriathat
that
thatare
are
areused
used
usedby by
byengineers
engineers
engineersand and
and
project
projectmanagers
project managers
managerstototoselectselect
selectthe
the
thebest
best
besteconomic
economic
economicchoice choice
choiceamong
among
amongseveral
several
severalalternatives.
alternatives.
alternatives.Projects
Projects
Projectsexamined
examined
examinedwill will
will
include
includeboth
include both
bothincome-and
income-and
income-andservice-producing
service-producing
service-producinginvestments.
investments.
investments.The The
Theeffects
effects
effectsofofofescalation,
escalation,
escalation,ination,
ination,
ination,and and
andtaxes
taxes
taxes
on
ononthe
the
theeconomic
economic
economicanalysis
analysis
analysisofofofalternatives
alternatives
alternativesareare
arediscussed.
discussed.
discussed.Risk
Risk
Riskanalysis
analysis
analysisincorporates
incorporates
incorporatesthe the
theconcepts
concepts
conceptsofofofprobability
probability
probability

ECONOMICS AND DECISION


and
andstatistics
and statistics
statisticsinininthe
the
theevaluation
evaluation
evaluationofofofalternatives.
alternatives.
alternatives.This This
Thisallows
allows
allowsmanagement
management
managementtototodetermine
determine
determinethe the
theprobability
probability
probabilityofofof
success
successor
success or
orfailure
failure
failureofofofthe
the
theproject.
project.
project. Two
Twotypes
Two types
typesofofofsensitivity
sensitivity
sensitivityanalyses
analyses
analysesare
are
arepresented.The
presented.The
presented.Therst rst
rstisisisreferred
referred
referredtototoasasas
the
therange
the range
rangeapproach
approach
approachwhile while
whilethe the
thesecond
second
secondusesuses
usesprobabilistic
probabilistic
probabilisticconcepts
concepts
conceptsto to
todetermine
determine
determineaaameasure
measure
measureofofofthe the
therisk
risk
risk
involved.
involved. The
The authors
authors have
have designed
designed the
the text
text toto assist
assistindividuals
individuals to
to prepare
prepare
involved. The authors have designed the text to assist individuals to prepare to successfully complete the to
tosuccessfully
successfully complete
complete the
the

David
DavidWhitman
David Whitman
Whitman

AND DECISION ANALYSIS


economics
economicsportions
economics portions
portionsofofofthe
the
theFundamentals
Fundamentals
FundamentalsofofofEngineering
Engineering
EngineeringExam. Exam.
Exam.

DECISION ANALYSIS
Ronald
RonaldE.
Ronald E.
E.Terry
Terry
Terry

ANALYSIS
About
AboutSYNTHESIs
About SYNTHESIs
SYNTHESIs
This
Thisvolume
This volume
volumeisisisaaaprinted
printed
printedversion
version
versionofofofaaawork
work
workthat
that
thatappears
appears
appearsinininthe
the
theSynthesis
Synthesis
Synthesis
Digital
DigitalLibrary
Digital Library
LibraryofofofEngineering
Engineering
Engineeringand and
andComputer
Computer
ComputerScience.
Science.
Science.Synthesis
Synthesis
SynthesisLectures
Lectures
Lectures

Mor gan
Mor
Mor gan &
provide
provideconcise,
provide concise,
concise, original
originalpresentations
original presentations
presentationsofofofimportant
important
importantresearch
research
researchand
and
anddevelopment
development
development
topics,
topics,
topics, published
publishedquickly,
published quickly,
quickly,
ininindigital
digital
digitaland
and
andprint
print
printformats.
formats.
formats. For
Formore
For more
moreinformation
information
information

gan & Cl
visit
visitwww.morganclaypool.com
www.morganclaypool.com
visit www.morganclaypool.com

SSSYNTHESIS
YNTHESIS L
LECTURES
YNTHESISL ECTURES ONE
ECTURESON
ON E
ENGINEERING
NGINEERING
NGINEERING

& Cl aypool
Cl aypool
ISBN:
ISBN:978-1-60845-864-6
ISBN: 978-1-60845-864-6
978-1-60845-864-6
Mor
Morgan
Mor gan
gan Cl
Cl
Claypool &
&
&
aypoolPublishers
aypool Publishers
Publishers 90000
90000
90000

aypool
wwwwwwwww. .m
.mmooorrgrggaaannncccl lalaayyypppoooool l.l.c.cocoommm
99978
78
78
11608
1608
608458646
458646
458646
Steven
StevenF.F.F.Barrett,
Steven Barrett, Series
Barrett,Series Editor
SeriesEditor
Editor
Fundamentals of
Engineering Economics
and Decision Analysis
Synthesis Lectures on
Engineering
Editor
Steven S. Barrett, University of Wyoming
Fundamentals of Engineering Economics and Decision Analysis
David L. Whitman and Ronald E. Terry
2012
A Little Book on Teaching: A Beginners Guide for Educators of Engineering and Applied
Science
Steven F. Barrett
2012
Engineering Thermodynamics and 21st Century Energy Problems: A Textbook Companion
for Student Engagement
Donna Riley
2011
MATLAB for Engineering and the Life Sciences
Joseph V. Tranquillo
2011
Systems Engineering: Building Successful Systems
Howard Eisner
2011
Fin Shape Thermal Optimization Using Bejans Constructal Theory
Giulio Lorenzini, Simone Moretti, and Alessandra Conti
2011
Geometric Programming for Design and Cost Optimization (with illustrative case study
problems and solutions), Second Edition
Robert C. Creese
2010
Survive and Thrive: A Guide for Untenured Faculty
Wendy C. Crone
2010
iii
Geometric Programming for Design and Cost Optimization (with Illustrative Case Study
Problems and Solutions)
Robert C. Creese
2009

Style and Ethics of Communication in Science and Engineering


Jay D. Humphrey and Jeffrey W. Holmes
2008

Introduction to Engineering: A Starters Guide with Hands-On Analog Multimedia


Explorations
Lina J. Karam and Naji Mounsef
2008

Introduction to Engineering: A Starters Guide with Hands-On Digital Multimedia and


Robotics Explorations
Lina J. Karam and Naji Mounsef
2008

CAD/CAM of Sculptured Surfaces on Multi-Axis NC Machine: The DG/K-Based


Approach
Stephen P. Radzevich
2008

Tensor Properties of Solids, Part Two: Transport Properties of Solids


Richard F. Tinder
2007

Tensor Properties of Solids, Part One: Equilibrium Tensor Properties of Solids


Richard F. Tinder
2007

Essentials of Applied Mathematics for Scientists and Engineers


Robert G. Watts
2007

Project Management for Engineering Design


Charles Lessard and Joseph Lessard
2007

Relativistic Flight Mechanics and Space Travel


Richard F. Tinder
2006
Copyright 2012 by Morgan & Claypool

All rights reserved. No part of this publication may be reproduced, stored in a retrieval system, or transmitted in
any form or by any meanselectronic, mechanical, photocopy, recording, or any other except for brief quotations in
printed reviews, without the prior permission of the publisher.

Fundamentals of Engineering Economics and Decision Analysis


David L. Whitman and Ronald E. Terry
www.morganclaypool.com

ISBN: 9781608458646 paperback


ISBN: 9781608458653 ebook

DOI 10.2200/S00410ED1V01Y201203ENG018

A Publication in the Morgan & Claypool Publishers series


SYNTHESIS LECTURES ON ENGINEERING

Lecture #18
Series Editor: Steven S. Barrett, University of Wyoming
Series ISSN
Synthesis Lectures on Engineering
Print 1939-5221 Electronic 1939-523X
Fundamentals of
Engineering Economics
and Decision Analysis

David L. Whitman
University of Wyoming

Ronald E. Terry
Brigham Young University

SYNTHESIS LECTURES ON ENGINEERING #18

M
&C Morgan & cLaypool publishers
ABSTRACT
The authors cover two general topics: basic engineering economics and risk analysis in this text.
Within the topic of engineering economics are discussions on the time value of money and
interest relationships. These interest relationships are used to define certain project criteria that are
used by engineers and project managers to select the best economic choice among several alternatives.
Projects examined will include both income- and service-producing investments. The effects of
escalation, inflation, and taxes on the economic analysis of alternatives are discussed. Risk analysis
incorporates the concepts of probability and statistics in the evaluation of alternatives. This allows
management to determine the probability of success or failure of the project. Two types of sensitivity
analyses are presented.The first is referred to as the range approach while the second uses probabilistic
concepts to determine a measure of the risk involved. The authors have designed the text to assist
individuals to prepare to successfully complete the economics portions of the Fundamentals of
Engineering Exam.

KEYWORDS
engineering economics, time value of money, net present value, internal rate of return,
cash flow analysis, probability, statistics, risk analysis
vii

To our parents, wives, children, and grandchildren


with much love and gratitude for everything.
ix

Contents
Preface . . . . . . . . . . . . . . . . . . . . . . . . . . . . . . . . . . . . . . . . . . . . . . . . . . . . . . . . . . . . . . . . . xiii

1 Introduction . . . . . . . . . . . . . . . . . . . . . . . . . . . . . . . . . . . . . . . . . . . . . . . . . . . . . . . . . . . . . . 1
1.1 Engineering Economics . . . . . . . . . . . . . . . . . . . . . . . . . . . . . . . . . . . . . . . . . . . . . . . . . 1
1.1.1 Basic Engineering Economics . . . . . . . . . . . . . . . . . . . . . . . . . . . . . . . . . . . . . . 1
1.1.2 Risk Analysis . . . . . . . . . . . . . . . . . . . . . . . . . . . . . . . . . . . . . . . . . . . . . . . . . . . . 1
1.2 Decision Analysis . . . . . . . . . . . . . . . . . . . . . . . . . . . . . . . . . . . . . . . . . . . . . . . . . . . . . . . 1
1.3 Fundamentals of Engineering Exam . . . . . . . . . . . . . . . . . . . . . . . . . . . . . . . . . . . . . . . 2

2 Interest and the Time Value of Money . . . . . . . . . . . . . . . . . . . . . . . . . . . . . . . . . . . . . . . 3


2.1 Time Value of Money . . . . . . . . . . . . . . . . . . . . . . . . . . . . . . . . . . . . . . . . . . . . . . . . . . . 3
2.2 Sources of Capital . . . . . . . . . . . . . . . . . . . . . . . . . . . . . . . . . . . . . . . . . . . . . . . . . . . . . . 3
2.3 Interest Concepts . . . . . . . . . . . . . . . . . . . . . . . . . . . . . . . . . . . . . . . . . . . . . . . . . . . . . . . 4
2.3.1 Simple Interest . . . . . . . . . . . . . . . . . . . . . . . . . . . . . . . . . . . . . . . . . . . . . . . . . . . 4
2.3.2 Compound Interest . . . . . . . . . . . . . . . . . . . . . . . . . . . . . . . . . . . . . . . . . . . . . . . 4
2.3.3 Nominal, Effective, and Continuous Interest Rates . . . . . . . . . . . . . . . . . . . . 5
2.4 Cash Flow Diagrams . . . . . . . . . . . . . . . . . . . . . . . . . . . . . . . . . . . . . . . . . . . . . . . . . . . . 6
2.5 Interest Formulas for Discrete Compounding . . . . . . . . . . . . . . . . . . . . . . . . . . . . . . . 8
2.5.1 Single Payments . . . . . . . . . . . . . . . . . . . . . . . . . . . . . . . . . . . . . . . . . . . . . . . . . . 9
2.5.2 Uniform Series (Annuities) . . . . . . . . . . . . . . . . . . . . . . . . . . . . . . . . . . . . . . . 10
2.5.3 Uniform Gradient . . . . . . . . . . . . . . . . . . . . . . . . . . . . . . . . . . . . . . . . . . . . . . . 11
2.5.4 The use of Financial Functions in Excel . . . . . . . . . . . . . . . . . . . . . . . . . . . 13
2.5.5 Example Problems . . . . . . . . . . . . . . . . . . . . . . . . . . . . . . . . . . . . . . . . . . . . . . . 15
2.6 Interest Formulas for Continuous Compounding . . . . . . . . . . . . . . . . . . . . . . . . . . . 19
2.6.1 Continuous Compounding for Discrete Payments . . . . . . . . . . . . . . . . . . . . 19
2.6.2 Continuous Compounding for Continuous Payments . . . . . . . . . . . . . . . . . 19
2.7 Problems . . . . . . . . . . . . . . . . . . . . . . . . . . . . . . . . . . . . . . . . . . . . . . . . . . . . . . . . . . . . . 20

3 Project Evaluation Methods . . . . . . . . . . . . . . . . . . . . . . . . . . . . . . . . . . . . . . . . . . . . . . 25


3.1 Introduction . . . . . . . . . . . . . . . . . . . . . . . . . . . . . . . . . . . . . . . . . . . . . . . . . . . . . . . . . . 25
3.2 Alternate Uses of Capital . . . . . . . . . . . . . . . . . . . . . . . . . . . . . . . . . . . . . . . . . . . . . . . 26
x
3.3 Minimum Acceptable Rate of Return (MARR) . . . . . . . . . . . . . . . . . . . . . . . . . . . . 26
3.4 Equivalence Methods . . . . . . . . . . . . . . . . . . . . . . . . . . . . . . . . . . . . . . . . . . . . . . . . . . 27
3.5 Net Present Value . . . . . . . . . . . . . . . . . . . . . . . . . . . . . . . . . . . . . . . . . . . . . . . . . . . . . . 27
3.5.1 Analysis of a Single Investment Opportunity . . . . . . . . . . . . . . . . . . . . . . . . 27
3.5.2 Do Nothing Project . . . . . . . . . . . . . . . . . . . . . . . . . . . . . . . . . . . . . . . . . . . . . . 29
3.5.3 Analysis of Multiple Investment Opportunities . . . . . . . . . . . . . . . . . . . . . . 30
3.6 Rate of Return Methods . . . . . . . . . . . . . . . . . . . . . . . . . . . . . . . . . . . . . . . . . . . . . . . . 31
3.6.1 Internal Rate of Return (IRR) . . . . . . . . . . . . . . . . . . . . . . . . . . . . . . . . . . . . . 31
3.6.2 Spreadsheet Formula for IRR . . . . . . . . . . . . . . . . . . . . . . . . . . . . . . . . . . . . . 33
3.6.3 External Rate of Return (ERR) . . . . . . . . . . . . . . . . . . . . . . . . . . . . . . . . . . . . 34
3.6.4 Spreadsheet Formula for ERR . . . . . . . . . . . . . . . . . . . . . . . . . . . . . . . . . . . . . 37
3.7 The Reinvestment Question in Rate of Return Calculations . . . . . . . . . . . . . . . . . . 37
3.7.1 Perception #1 . . . . . . . . . . . . . . . . . . . . . . . . . . . . . . . . . . . . . . . . . . . . . . . . . . . 39
3.7.2 Perception #2 . . . . . . . . . . . . . . . . . . . . . . . . . . . . . . . . . . . . . . . . . . . . . . . . . . . 40
3.7.3 Final Comments on ERR and IRR Relationships . . . . . . . . . . . . . . . . . . . . 41
3.8 Acceleration Projects . . . . . . . . . . . . . . . . . . . . . . . . . . . . . . . . . . . . . . . . . . . . . . . . . . . 41
3.9 Payout . . . . . . . . . . . . . . . . . . . . . . . . . . . . . . . . . . . . . . . . . . . . . . . . . . . . . . . . . . . . . . . 44
3.10 Problems . . . . . . . . . . . . . . . . . . . . . . . . . . . . . . . . . . . . . . . . . . . . . . . . . . . . . . . . . . . . . 46

4 Service Producing Investments . . . . . . . . . . . . . . . . . . . . . . . . . . . . . . . . . . . . . . . . . . . 49


4.1 Introduction . . . . . . . . . . . . . . . . . . . . . . . . . . . . . . . . . . . . . . . . . . . . . . . . . . . . . . . . . . 49
4.2 Equal Life Alternatives . . . . . . . . . . . . . . . . . . . . . . . . . . . . . . . . . . . . . . . . . . . . . . . . . 49
4.2.1 Equivalence Techniques . . . . . . . . . . . . . . . . . . . . . . . . . . . . . . . . . . . . . . . . . . 49
4.2.2 Rate of Return Methods . . . . . . . . . . . . . . . . . . . . . . . . . . . . . . . . . . . . . . . . . . 50
4.3 Unequal Life Alternatives . . . . . . . . . . . . . . . . . . . . . . . . . . . . . . . . . . . . . . . . . . . . . . . 54
4.3.1 Least Common Multiple Method . . . . . . . . . . . . . . . . . . . . . . . . . . . . . . . . . . 54
4.3.2 Common Study Period . . . . . . . . . . . . . . . . . . . . . . . . . . . . . . . . . . . . . . . . . . . 55
4.4 Problems . . . . . . . . . . . . . . . . . . . . . . . . . . . . . . . . . . . . . . . . . . . . . . . . . . . . . . . . . . . . . 57

5 Income Producing Investments . . . . . . . . . . . . . . . . . . . . . . . . . . . . . . . . . . . . . . . . . . . 61


5.1 Introduction . . . . . . . . . . . . . . . . . . . . . . . . . . . . . . . . . . . . . . . . . . . . . . . . . . . . . . . . . . 61
5.2 Investment in a Single Project . . . . . . . . . . . . . . . . . . . . . . . . . . . . . . . . . . . . . . . . . . . 61
5.3 Mutually Exclusive Alternatives . . . . . . . . . . . . . . . . . . . . . . . . . . . . . . . . . . . . . . . . . 62
5.3.1 Equivalence Techniques . . . . . . . . . . . . . . . . . . . . . . . . . . . . . . . . . . . . . . . . . . 62
5.3.2 Rate of Return Techniques . . . . . . . . . . . . . . . . . . . . . . . . . . . . . . . . . . . . . . . . 64
5.3.3 Using Excel . . . . . . . . . . . . . . . . . . . . . . . . . . . . . . . . . . . . . . . . . . . . . . . . . . . 70
xi
5.4 Unequal Life Alternatives . . . . . . . . . . . . . . . . . . . . . . . . . . . . . . . . . . . . . . . . . . . . . . . 72
5.5 Independent and Contingent Investments . . . . . . . . . . . . . . . . . . . . . . . . . . . . . . . . . 75
5.5.1 Independent Investments . . . . . . . . . . . . . . . . . . . . . . . . . . . . . . . . . . . . . . . . . 75
5.5.2 Contingent Investments . . . . . . . . . . . . . . . . . . . . . . . . . . . . . . . . . . . . . . . . . . 75
5.5.3 Limited Investment Capital . . . . . . . . . . . . . . . . . . . . . . . . . . . . . . . . . . . . . . . 77
5.6 Ranking Alternatives . . . . . . . . . . . . . . . . . . . . . . . . . . . . . . . . . . . . . . . . . . . . . . . . . . . 79
5.7 Problems . . . . . . . . . . . . . . . . . . . . . . . . . . . . . . . . . . . . . . . . . . . . . . . . . . . . . . . . . . . . . 83

6 Determination of Project Cash Flow . . . . . . . . . . . . . . . . . . . . . . . . . . . . . . . . . . . . . . . 87


6.1 Introduction . . . . . . . . . . . . . . . . . . . . . . . . . . . . . . . . . . . . . . . . . . . . . . . . . . . . . . . . . . 87
6.2 Escalation and Inflation . . . . . . . . . . . . . . . . . . . . . . . . . . . . . . . . . . . . . . . . . . . . . . . . 87
6.3 Depreciation . . . . . . . . . . . . . . . . . . . . . . . . . . . . . . . . . . . . . . . . . . . . . . . . . . . . . . . . . . 94
6.3.1 Straight-Line Depreciation (SL) . . . . . . . . . . . . . . . . . . . . . . . . . . . . . . . . . . . 95
6.3.2 Declining-Balance Depreciation . . . . . . . . . . . . . . . . . . . . . . . . . . . . . . . . . . . 96
6.3.3 Sum-of-the-Years-Digits (SYD) Depreciation . . . . . . . . . . . . . . . . . . . . . . . 97
6.3.4 Modified Accelerated Cost Recovery System (MACRS) . . . . . . . . . . . . . 102
6.4 Cash Flow Computation . . . . . . . . . . . . . . . . . . . . . . . . . . . . . . . . . . . . . . . . . . . . . . . 106
6.4.1 Capital Investment . . . . . . . . . . . . . . . . . . . . . . . . . . . . . . . . . . . . . . . . . . . . . 106
6.4.2 Gross Revenue . . . . . . . . . . . . . . . . . . . . . . . . . . . . . . . . . . . . . . . . . . . . . . . . . 107
6.4.3 Operating Expenses . . . . . . . . . . . . . . . . . . . . . . . . . . . . . . . . . . . . . . . . . . . . . 107
6.4.4 Before-Tax Profit Computation . . . . . . . . . . . . . . . . . . . . . . . . . . . . . . . . . . . 107
6.4.5 Before-Tax Cash Flow Computation . . . . . . . . . . . . . . . . . . . . . . . . . . . . . . 107
6.4.6 Depreciation . . . . . . . . . . . . . . . . . . . . . . . . . . . . . . . . . . . . . . . . . . . . . . . . . . . 108
6.4.7 Taxable Income . . . . . . . . . . . . . . . . . . . . . . . . . . . . . . . . . . . . . . . . . . . . . . . . 109
6.4.8 State and Federal Income Tax . . . . . . . . . . . . . . . . . . . . . . . . . . . . . . . . . . . . 109
6.4.9 Net Profit . . . . . . . . . . . . . . . . . . . . . . . . . . . . . . . . . . . . . . . . . . . . . . . . . . . . . 110
6.4.10 Cash Flow . . . . . . . . . . . . . . . . . . . . . . . . . . . . . . . . . . . . . . . . . . . . . . . . . . . . . 110
6.5 Problems . . . . . . . . . . . . . . . . . . . . . . . . . . . . . . . . . . . . . . . . . . . . . . . . . . . . . . . . . . . . 115

7 Financial Leverage . . . . . . . . . . . . . . . . . . . . . . . . . . . . . . . . . . . . . . . . . . . . . . . . . . . . . 121


7.1 Introduction . . . . . . . . . . . . . . . . . . . . . . . . . . . . . . . . . . . . . . . . . . . . . . . . . . . . . . . . . 121
7.2 Financial Leverage and Associated Risk . . . . . . . . . . . . . . . . . . . . . . . . . . . . . . . . . . 121
7.3 Adjustment to Cash Flow Equations . . . . . . . . . . . . . . . . . . . . . . . . . . . . . . . . . . . . 121
7.3.1 Leverage and Mutually Exclusive Projects . . . . . . . . . . . . . . . . . . . . . . . . . . 132
7.3.2 Excel Spreadsheet . . . . . . . . . . . . . . . . . . . . . . . . . . . . . . . . . . . . . . . . . . . . . 132
7.4 Problems . . . . . . . . . . . . . . . . . . . . . . . . . . . . . . . . . . . . . . . . . . . . . . . . . . . . . . . . . . . . 134
xii
8 Basic Statistics and Probability . . . . . . . . . . . . . . . . . . . . . . . . . . . . . . . . . . . . . . . . . . . 135
8.1 Introduction . . . . . . . . . . . . . . . . . . . . . . . . . . . . . . . . . . . . . . . . . . . . . . . . . . . . . . . . . 135
8.2 Statistics . . . . . . . . . . . . . . . . . . . . . . . . . . . . . . . . . . . . . . . . . . . . . . . . . . . . . . . . . . . . 135
8.2.1 Measures of Central Tendency . . . . . . . . . . . . . . . . . . . . . . . . . . . . . . . . . . . . 135
8.2.2 Measures of Dispersion . . . . . . . . . . . . . . . . . . . . . . . . . . . . . . . . . . . . . . . . . . 139
8.2.3 Frequency Distributions . . . . . . . . . . . . . . . . . . . . . . . . . . . . . . . . . . . . . . . . . 142
8.2.4 Relative Frequency Distribution . . . . . . . . . . . . . . . . . . . . . . . . . . . . . . . . . . 146
8.3 Probability . . . . . . . . . . . . . . . . . . . . . . . . . . . . . . . . . . . . . . . . . . . . . . . . . . . . . . . . . . . 149
8.3.1 Classical Definition . . . . . . . . . . . . . . . . . . . . . . . . . . . . . . . . . . . . . . . . . . . . . 149
8.3.2 Relative Frequency Definition . . . . . . . . . . . . . . . . . . . . . . . . . . . . . . . . . . . . 151
8.3.3 Subjective Definition . . . . . . . . . . . . . . . . . . . . . . . . . . . . . . . . . . . . . . . . . . . . 151
8.3.4 Probability Distributions . . . . . . . . . . . . . . . . . . . . . . . . . . . . . . . . . . . . . . . . 151
8.4 Problems . . . . . . . . . . . . . . . . . . . . . . . . . . . . . . . . . . . . . . . . . . . . . . . . . . . . . . . . . . . . 168

9 Sensitivity Analysis . . . . . . . . . . . . . . . . . . . . . . . . . . . . . . . . . . . . . . . . . . . . . . . . . . . . . 171


9.1 Introduction . . . . . . . . . . . . . . . . . . . . . . . . . . . . . . . . . . . . . . . . . . . . . . . . . . . . . . . . . 171
9.1.1 Range Approach . . . . . . . . . . . . . . . . . . . . . . . . . . . . . . . . . . . . . . . . . . . . . . . 171
9.1.2 Monte Carlo Simulation . . . . . . . . . . . . . . . . . . . . . . . . . . . . . . . . . . . . . . . . . 175
9.2 Problems . . . . . . . . . . . . . . . . . . . . . . . . . . . . . . . . . . . . . . . . . . . . . . . . . . . . . . . . . . . . 187

A Compound Interest Factors . . . . . . . . . . . . . . . . . . . . . . . . . . . . . . . . . . . . . . . . . . . . . . 189

Authors Biographies . . . . . . . . . . . . . . . . . . . . . . . . . . . . . . . . . . . . . . . . . . . . . . . . . . . 205


xiii

Preface
Those individuals working on the development of an income-generating project, either for personal
use or company use, are frequently called upon to determine if the endeavor will prove profitable if
fully developed. By profitable, we simply mean that the project will provide a desirable rate of return
on investment through the generation of revenue that offsets any capital and/or operating costs.
The intent of this book is to provide individuals with the tools to evaluate projects to determine
profitability.The subject has been called: Engineering Economics or Project Evaluation or Economic
Evaluation or Decision Analysis. Whatever one chooses to call it, the reader who studies this material
and becomes proficient in its content, will be able to analyze project cash flows and make a decision
as to the profitability of the project. The authors, mainly because of their engineering backgrounds,
have chosen to refer to the subject matter as engineering economics.
In addition to income-generating projects, this book will also assist those individuals who are
analyzing two or more ways of doing a service-producing project. A service-producing project is
one, that instead of generating income for the investor, provides a service at a cost to the investor.
An example could be the renting versus purchasing of a vehicle to provide a needed service for a
company.
The authors cover two general topics: basic engineering economics and risk analysis in the
text. Chapters 2-6 contain content relative to basic engineering economics and Chapters 7-9 present
material on risk analysis.
Within the topic of engineering economics are discussions on the time value of money and
interest relationships. These interest relationships are used to define certain project criteria that are
used by engineers and project managers to select the best economic choice among several alterna-
tives. Projects examined will include both income and service producing investments. The effects of
escalation, inflation, and taxes on the economic analysis of alternatives are discussed.
There is always risk involved in undertaking a project. Risk analysis incorporates the concepts
of probability and statistics in the evaluation of alternatives. This allows management to determine
the probability of success or failure of the project. Two types of sensitivity analyses are presented. The
first is referred to as the range approach while the second uses probabilistic concepts to determine a
measure of the risk involved.
The authors have designed the text to assist individuals to prepare to successfully complete
the economics portions of the Fundamentals of Engineering Exam.
xiv PREFACE
The authors wish to thank Joel Claypool and his associates at Morgan & Claypool for their
encouragement and excellent work on the preparation and production of this text.

David L. Whitman and Ronald E. Terry


May 2012
1

CHAPTER 1

Introduction
1.1 ENGINEERING ECONOMICS
Nearly all projects that are proposed to be undertaken by any engineering firm will be, at some point,
subjected to close economic scrutiny. The results of this analysis will be a basis (perhaps one of many)
for deciding whether or not to proceed with the project. The major emphasis of this text, therefore,
is to provide the engineer with the tools necessary to make the aforementioned economic decision.
There are two general topics which are included in this textbook: basic engineering economics
and risk analysis. A very brief overview of each of these topics is presented in the following paragraphs.

1.1.1 BASIC ENGINEERING ECONOMICS


Within this topic are discussions on the time value of money and interest relationships. These
interest relationships are used to define certain project criteria that are used by engineers and project
managers to select the best economic choice among several alternatives. Projects examined will
include traditional projects that generate a profit for the company and service producing projects
which do not provide income, but do provide a needed service. The effects of escalation, inflation,
and taxes on the economic analysis of alternatives will be discussed.

1.1.2 RISK ANALYSIS


There is always risk involved in undertaking a project. Management is interested in the quantification
of that risk. Risk analysis incorporates the concepts of probability and statistics in the evaluation
of alternatives. This allows management to determine the probability of success or failure of the
project. While there are a variety of ways to incorporate risk analysis into the evaluation of a project,
the authors will present two methods that utilize what is known as sensitivity analysis. That is, deter-
mining the sensitivity of the economic viability of a project as the costs and/or incomes vary about
estimated values. The first is referred to as the range approach, while the second uses probabilistic
concepts to determine a measure of the risk involved.

1.2 DECISION ANALYSIS


As described above, the overall objective of any economic analysis is to provide a basis for making
a sound decision regarding any particular project. For example, suppose that an engineer is given
the assignment to implement a project for which there are multiple alternative methods that will
achieve the goals of the project. The question is: which alternative should be chosen? The reader
2 1. INTRODUCTION
should recognize that there is always a choice among two or more alternatives. However, if only one
technical alternative is found, then that alternative must be compared with the do nothing case.
The do nothing case represents the situation where a company keeps its money invested in other
alternatives which earn some minimum rate of return. The minimum rate of return will be referred
to as the minimum acceptable rate of return (MARR) and will be discussed in detail in Chapter 2.
Thus, there are always at least two alternatives in any economic decision.
This textbook will provide the engineer with the necessary tools to determine the best
economic choice among the alternatives. However, one must realize that final decisions are not only
made on the results of the economic evaluations. Other general areas of consideration could be
classified as financial and intangible issues.
The best economic choice will be made through the proper use of the time value of money
formulas that will be presented. Financial aspects have to do with the obtaining of funds required
to initiate the project. There are several sources which may be considered, i.e., internal company
funds, lending institutions, the issuing of bonds, or the issuing of new stock. The intangible area of a
project is the most difficult to analyze. Included in the intangible aspects are environmental, social,
and political impacts. These are the most difficult to quantify. The focus of this textbook will be on
the economic aspects of a project and very little time will be devoted to the areas of financial and
intangible aspects. However, they are alluded to from time to time in order to remind the engineer
of their importance and the obligation to consider them in the final decision.

1.3 FUNDAMENTALS OF ENGINEERING EXAM


It is envisioned that information found in this textbook will prepare students to successfully complete
the economics portions of the Fundamentals of Engineering Exam. The specifications for this exam
can be found at http://www.ncees.org/Exams/FE_exam.php.
3

CHAPTER 2

Interest and the Time Value of


Money
2.1 TIME VALUE OF MONEY
When an individual or a company desires to invest an amount of capital in a long-term project, the
effect of time on the value of that capital needs to be considered. The effect of time on the value of
money can be illustrated by the following examples.
Consider a sum of $1000 that an individual has accumulated. If the $1000 were buried in a
can under a tree for some future need, the individual, one year later, would still have $1000. However,
if the $1000 were placed in an insured savings account earning 3% interest for one year, the amount
would have grown to $1030. Obviously, the length of time and the different investment opportunities
(represented by different interest rates) lead to varying amounts of money that the $1000 can yield
at some future date.
A second example deals with the same $1000 and its purchasing power as a function of time.
Suppose an individual has a choice of purchasing 1000 items now at a price of $1.00 per item or
waiting until a future date to make the purchase. If, over the course of one year, the price increased
to $1.03 per item, the $1000 will only be able to purchase 970 items. Thus, the value, in terms of
purchasing power, has decreased with time.
The longer the life of the project, the more important will be the considerations of the time
value of money. Other factors that affect the outcome of investment projects are inflation, taxes, and
risk. These will be discussed later in the text.

2.2 SOURCES OF CAPITAL


There are, in general, two sources of capital needed to make an investment. Capital can be obtained
either from the investors own funds or from a lender. Wherever capital is obtained, there is a cost
associated with the use of the funds. If they are obtained from a lender, the cost of capital is the
interest rate at which the funds are loaned to the investor. This interest rate reflects the current
state of the economy as a whole, the banks administrative costs, and, perhaps, the risk associated
with the particular loan as viewed by the lender. If the investor chooses to use his own funds for
the required capital, then the cost is called the opportunity cost of capital. The opportunity cost
reflects the income that could be generated from other opportunities the investor might have for
his funds. This opportunity cost is often referred to as the minimum acceptable rate of return
4 2. INTEREST AND THE TIME VALUE OF MONEY
(MARR). This minimum acceptable rate of return could be the interest rate obtained by placing
the funds in a certificate of deposit or savings account at a bank or it could be the rate of return on
another investment opportunity.The MARR is an important concept in the evaluation of investment
opportunities and will be discussed in Chapter 3. For now, the MARR will just be treated as an
interest rate, i.

2.3 INTEREST CONCEPTS


2.3.1 SIMPLE INTEREST
The amount of interest earned by an investment (for example, a single principal deposit in a savings
account) is called simple interest when the interest is found by Equation 2.1:
I = P in (2.1)

where, I = total interest, dollars


P = amount of principal, dollars
i = interest rate per interest period, fraction
n = number of interest periods.
Consider the following example. Individual A agrees to loan individual B $1000 for a time
period of 3 years. B agrees to pay A the $1000 at the end of the 3 years, plus an amount of interest
determined by applying a simple interest rate of 10% per year. The total interest charge will be:
I = (1000)(0.10)(3) = $300
Therefore, at the end of 3 years, B will pay a total of $1300 to A which would represent the $1000
initially borrowed plus $300 interest for the use of As money.

2.3.2 COMPOUND INTEREST


Simple interest concepts are used infrequently in todays business dealings, but they do provide the
basis for compounded interest rate concepts that are utilized. Compounded interest is computed
by applying the interest rate to the remaining unpaid principal plus any accumulated interest. One
could consider it as the interest earns interest. Referring back to the example presented above, the
total interest that B will pay A over 3 years would be calculated as the following:
Iyr 1 = (1000)(0.1) = $100, which would result in a balance at the end of year 1 of $1100
Iyr 2 = (1100)(0.1) = $110, which would result in a balance at the end of year 2 of $1210
Iyr 3 = (1210)(0.1) = $121, which would result in a balance at the end of year 3 of $1331
Therefore, at the end of 3 years, B will pay a total of $1331 to A which is $31 higher than for the
simple interest case. This difference results from compounding the interest. One should note that
the difference between these two methods will become larger as the interest rate and number of
interest periods increase.
2.3. INTEREST CONCEPTS 5
2.3.3 NOMINAL, EFFECTIVE, AND CONTINUOUS INTEREST RATES
The length of the interest period can and does vary from application to application. Common
interest rate periods are annually, semi-annually, quarterly, monthly, daily, and in the limiting case,
continuously. The amount of interest that is earned or charged to a principal will increase as the
compounding period becomes smaller.
Usually, a lending institution will quote a nominal annual percentage rate. However, payments
on the loan are made more often than annually. For example, consider a loan that is quoted at
10% nominal with semi-annual compounding (and, thus, semi-annual payments). The 10% annual
interest compounded semi-annually means that every one-half year, 5% interest is earned or charged
to the principal. This leads to the concept of effective yearly interest rate. The effective yearly interest
rate can be found by computing the value that the principal has grown to at the end of year one, F ,
subtracting the original principal, and then dividing by the principal:
F = 1000 + 1000(.05) + (1000 + 1000(.05))(.05) = 1000(1.05)2
Therefore, the effective rate is:
ie = (1000(1.05)2 1000)/1000 = 0.1025 or 10.25% per year
In general, the effective rate can be found by:
 
i m
ie = 1 + 1 (2.2)
m

where, m = number of interest periods per year


i = yearly nominal interest rate, fraction
ie = yearly effective interest rate, fraction.
In the limiting case of continuous compounding, the effective rate is given by:
ie = e i 1 (2.3)
Table 2.1 lists the effective rates for various compounding time periods for a 10% nominal rate.
As can be observed in Table 2.1, the difference between the effective rates generated by the
various compounding periods is relatively small. The differences can become insignificant when
considering the many uncertainties associated with analyzing most economic investments.
One should be careful with the term Annual Percentage Rate (APR) when dealing with
lending institutions. The APR is a yearly percentage rate that expresses the total finance charge on
a loan over its entire term. The APR includes the nominal interest rate, fees, points, and mortgage
insurance, and is therefore a more complete measure of a loans cost than the interest rate alone.
The loans nominal interest rate, not its APR, is used to calculate the monthly principal and interest
payment.
6 2. INTEREST AND THE TIME VALUE OF MONEY

Table 2.1: Example of effective interest rates for


compounding time periods

2.4 CASH FLOW DIAGRAMS


The construction of a cash flow diagram, sometimes referred to as a time line, will greatly aid in
the analysis of an investment opportunity. The cash flow diagram is a way of accounting for all cash
incomes and outflows at their appropriate position in time. That is, in general terms, the cash flow
for any particular period is the income received during that period minus the expenses incurred
during that same period. A good analogy to a cash flow diagram is ones checkbook. Deposits and
checks are written at specific points in time. These transactions could be consolidated on a monthly
basis to show the net cash flow in or out of the checkbook each month. Usually, once the cash flow
diagram is constructed properly, the economic analysis becomes relatively easy to complete.
There are several ways of constructing a cash flow diagram and the following method is
utilized by the authors. A horizontal line is drawn which represents the length of time (life) of the
investment opportunity (project). The interest periods are then marked off and labeled above the
line. At the extreme left of the time line is time zero (or, as will be defined in the next section,
the Present). Time zero represents the time when the first cash flow is made for this project. Time
zero is, therefore, defined by each project and not by a specific calendar date. Time zero can also be
interpreted as the beginning of time period 1. All cash flows are then placed beneath the time line,
corresponding to the position in time (or interest periods) in which they occurred. Negative cash
flows (expenses exceeding revenues) are given a minus sign. In the time line illustrated below, CF1 ,
CF2 , etc., represent the cash flows occurring at the end of interest period 1, 2, etc. The authors often
use a break in the time line for brevity.
When dealing with investments in engineering projects, the normal approach is to assume
that all investments for a particular year are made at the beginning of the year, while all revenues
and operating expenses occur at the end of the year. This will lead to a conservative evaluation of
the project using the techniques presented in Chapter 3.
2.4. CASH FLOW DIAGRAMS 7
0 1 2 3 n-2 n-1 n

CF0 CF1 CF2 CF3 CFn-2 CFn-1 CFn

Example 2.1
Consider the example of a 3-year auto loan from the view of the lender. The lender provides
$20,000 to the client (a negative cash flow for the lender) at month 0 at an interest rate of 0.5% per
month. In exchange, the lender receives $608 per month from the client over the next 36 months.
The resulting cash flow diagram would be:

0 1 2 3 34 35 36

-20,000 608 608 608 ... 608 608 608

Before equations can be developed that relate the time value of money, it is necessary to define
a set of notations that will be used throughout the text.

P = Present sum of money. The present (time zero) is defined as any point from which the
analyst wishes to measure time.
F = Future sum of money. The future is defined as any point n that is greater than time zero.
A = Annuity. This is a uniform set of equal payments that occur at the end of each interest
period from one to n.
G = Uniform gradient. This is a series of payments that uniformly increase or decrease over
the life of the project.
i = Compound interest rate per period.
n = Total number of compounding periods in the cash flow diagram.

The cash flow diagrams that follow should help to define these sums of money.
8 2. INTEREST AND THE TIME VALUE OF MONEY
Present, P :
0 1 2 3 n-2 n-1 n

P
Future, F :
0 1 2 3 n-2 n-1 n

F
Annuity, A:
0 1 2 3 n-2 n-1 n

A A A A A A
Gradient, G:
0 1 2 3 n-2 n-1 n

G 2G (n-3)G (n-2)G (n-1)G

2.5 INTEREST FORMULAS FOR DISCRETE


COMPOUNDING
The following section contains the derivation and sample calculations for nine interest formulas used
in most economic calculations. These formulas demonstrate the equivalency between the various
2.5. INTEREST FORMULAS FOR DISCRETE COMPOUNDING 9
sums of money described above at specific values of the interest rate, i, and the number of periods,
n. For example, in the example of the 3-year car loan, the $608 monthly payment is equivalent to
the $20,000 initial loan at an interest rate of 0.5% per month. These formulas are based on discrete
compounding, i.e., the interest is compounded at the end of each finite interest period. Formulas
used with continuous compounding will be presented later.

2.5.1 SINGLE PAYMENTS


The first formula to be derived allows the calculation of the equivalent future amount F , of a present
sum, P . Suppose P is placed in a bank account that earns i% interest per period. It will grow to a
future amount, F , at the end of n interest periods according to:

F = P (1 + i)n (2.4)

The derivation of Equation 2.4 is given by:

The factor (1 + i)n is frequently called the Single Payment Compound Amount Factor and
is symbolized in this text by (F /P )i,n . If one is given the amount of P , one uses the (F /P )i,n factor
to find the equivalent value of F . That is,

F = P (F /P )i,n (2.5)

Similarly, if a future amount, F , is known and it is desired to calculate the equivalent present amount,
P , then Equation 2.4 can be arranged as:

P = F (1 + i)n (2.6)

The factor (1 + i)n is frequently called the Single Payment Present Worth Factor and is symbolized
in this text by (P /F )i,n . If one is given the amount of F , one uses the (P /F )i,n factor to find the
equivalent value of P . That is,
P = F (P /F )i,n (2.7)
10 2. INTEREST AND THE TIME VALUE OF MONEY
2.5.2 UNIFORM SERIES (ANNUITIES)
It is often necessary to know the amount of a uniform series payment, A, which would be equivalent
to a present sum, P , or a future sum, F . In the following formulas that relate P , F , and A, it is
imperative that the reader understands that: 1) P occurs one interest period before the first value
of A; 2) A occurs at the end of each interest period; and 3) F occurs at the same time as the last A
(at time n). These relationships were illustrated in the previous cash flow diagrams that originally
defined each of them.
The value of a future sum, F , of a series of uniform payments, each of value A, can be found
by summing the future worth of each individual payment. That is, treat each A as a distinct present
value (but with a different time zero) and use (F /P )i,n to calculate its contribution to the total F :

F = A(1 + i)n1 + A(1 + i)n2 + A(1 + i)n3 + . . . + A(1 + i)1 + A (2.8)

Multiplying both sides of Equation 2.8 by (1 + i) yields

F (1 + i) = A(1 + i)n + A(1 + i)n1 + A(1 + i)n2 + . . . + A(1 + i)2 + A(1 + i) (2.9)

Subtracting Equation 2.8 from 2.9 yields

F (1 + i) F = A(1 + i)n A

Solving for F in terms of A results in:

F = A{[(1 + i)n 1]/i} (2.10)

The term in the {} brackets is called the Uniform Series Compound Amount Factor and is symbolized
by (F /A)i,n . If one is given the amount of A, one uses the (F /A)i,n factor to find the equivalent
value of F . That is,
F = A(F /A)i,n (2.11)
Rearranging Equation 2.10 and solving for A yields

A = F {i/[(1 + i)n 1]} (2.12)

The term in the { } brackets is called the Sinking Fund Factor and is symbolized by (A/F )i,n . If one
is given the amount of F , one uses the (A/F )i,n factor to find the equivalent value of A. That is,

A = F (A/F )i,n (2.13)

Substitution of Equation 2.10 into Equation 2.6 yields Equation 2.14 which contains the Uniform
Series Present Worth Factor, (P /A)i,n in the {} brackets:

P = A{[(1 + i)n 1]/[i(1 + i)n ]} (2.14)


2.5. INTEREST FORMULAS FOR DISCRETE COMPOUNDING 11
If one is given the amount of A, one uses the (P /A)i,n factor to find the equivalent value of P . That
is,
P = A(P /A)i,n (2.15)
Rearranging Equation 2.14 and solving for A yields
A = P {[i(1 + i)n ]/[(1 + i)n 1]} (2.16)
The term in the { } brackets is called the Capital Recovery Factor and is symbolized by (A/P )i,n . If
one is given the amount of P , one uses the (A/P )i,n factor to find the equivalent value of A. That
is,
A = P (A/P )i,n (2.17)

2.5.3 UNIFORM GRADIENT


In some applications, a series of cash flows will be generated from a project analysis which uniformly
increase or decrease from an initial value. The cash flow diagram is repeated here for clarity.

0 1 2 3 n-2 n-1 n

G 2G (n-3)G (n-2)G (n-1)G

Without derivation, Equations 2.18, 2.20, and 2.22 can be developed that relate the gradient,
G, to an equivalent annuity, an equivalent present sum, and an equivalent future sum:
A = G{1/i n/[(1 + i)n 1]} (2.18)
The term in the { } brackets is symbolized by (A/G)i,n . If one is given the amount of G, one uses
the (A/G)i,n factor to find the equivalent value of A. That is,
A = G(A/G)i,n (2.19)
P = G{[(1 + i)n 1]/[i 2 (1 + i)n ] n/[i(1 + i)n ]} (2.20)
The term in the { } brackets is symbolized by (P /G)i,n . If one is given the amount of G, one uses
the (P /G)i,n factor to find the equivalent value of P . That is,
P = G(P /G)i,n (2.21)
F = G{[(1 + i)n 1]/i 2 n/ i} (2.22)
The term in the { } brackets is symbolized by (F /G)i,n . If one is given the amount of G, one uses
the (F /G)i,n factor to find the equivalent value of F . That is,
F = G(F /G)i,n (2.23)
12 2. INTEREST AND THE TIME VALUE OF MONEY
The equations for the nine factors are given in Table 2.2 and numerical values are tabulated
in Appendix A for various values of interest rate, i, and number of periods, n so that the user can
look them up rather than use the actual formulas.
Rather than memorizing which factor is needed for a specific equivalency, think about the
formulas in terms of units conversion. That is, if the input to a system has units of X and the output
of that system has units of Y, the system provides a units conversion of (Y/X). Thus, if one is given
A (input) and wants to find G (output), the correct formula to use would be (G/A). Knowing the
value of the interest rate and the number of periods, one can look up or compute the value of the
formula.

Table 2.2: Formulas for discrete compounding


Factor Name Converts Symbol Formula
Single Payment
to F given P (F / P) i,n (1 + i) n
Compound Amount

Single Payment Present (1 + i) -n


to P given F (P / F) i,n
Worth

Uniform Series
to F given A (F / A) i,n (1 + i) n-1
Compound Amount i

Uniform Series Sinking i


to A given F (A / F) i,n
Fund (1 + i) n-1

Uniform Series Present


(P / A) i,n (1 + i) n-1
to P given A
Worth i (1 + i) n

i (1 + i) n
Capital Recovery to A given P (A/ P) i,n
(1 + i) n-1

Uniform Gradient (1 + i) n-1 n


to P given G (P/ G) i,n -
Present Worth i 2(1 + i) n i (1 + i) n

Uniform Gradient
(F / G) i,n (1 + i) n-1 n
to F given G -
Future Value i2 i
Uniform Gradient
1 n
Uniform Series to A given G (A / G)i,n -
i (1 + i) n-1
2.5. INTEREST FORMULAS FOR DISCRETE COMPOUNDING 13
2.5.4 THE USE OF FINANCIAL FUNCTIONS IN EXCEL
Many cash flow situations can be simulated by using a spreadsheet such as Microsoft Excel. This
will become more evident in future chapters, but this chapter presents the following useful financial
functions:

Future Value: =FV(rate, nper, pmt, pv, type)

Present Value: =PV(rate, nper, pmt, fv, type)

Annuity: =PMT(rate, nper, pv, fv, type)

Unfortunately, Excel does not have a built-in function for gradient-type cash flows. That
can, however, be overcome with functions that will be presented in later chapters.
In each of these functions, the variables are as follows:

rate is the interest rate (as a fraction) per period

nper is the number of interest bearing periods

pmt is an annuity (A) sum of money

pv is a present (P ) value sum of money (occurs at time = 0)

fv is a future (F ) value sum of money (occurs at time = nper)

type is 0 for end of period cash flows and 1 for beginning of period cash flows

It should also be noted that in order to use these functions as equivalents for (P /A), (P /F ),
etc., the values of pmt, pv, and fv need to be input as negative numbers.
An example of a simple Excel spreadsheet that computes the six functions given above is
shown for 10% annual interest rate for 10 years. The actual formulas are shown as well. Recall that
one needs to set type equal to zero to designate that the cash flows occur at the end of each period.
An explanation of the values in the various Excel formulas may be necessary. For example, in the
formula that computes F/A (cell B7), the values are as follows: B1 is the interest rate as a fraction,
B2 is for 10 periods, B3 is for an annual annuity payment of $1 per year, 0 represents the fact
that there is no present value payment, and B6 defines that the various payments are at the end of
the period. Since the formula finds the future value of a $1 annuity, we have effectively computed
(F/A). Some additional Excel financial functions that might be of some interest at this point are:
14 2. INTEREST AND THE TIME VALUE OF MONEY

A B A B
1 rate 0.1 1 rate 0.1
2 nper 10 2 nper 10
3 pmt(A) -1 3 pmt(A) -1
4 pv (P) -1 4 pv(P) -1
5 fv (F) -1 5 fv(F) -1
6 type 0 6 type 0
7 F/A 15.937 7 F/A =FV(B1,B2,B3,0,B6)
8 F/P 2.5937 8 F/P =FV(B1,B2,0,B4,B6)
9 P/A 6.1446 9 P/A =PV(B1,B2,B3,0,B6)
10 P/F 0.38554 10 P/F =PV(B1,B2,0,B5,B6)
11 A/P 0.16275 11 A/P =PMT(B1,B2,B4,0,B6)
12 A/F 0.062745 12 A/F =PMT(B1,B2,0,B5,B6)

Effective Interest Rate: =EFFECT(normal_rate, npery)

Number of periods: =NPER(rate, pmt, pv, fv, type)

The new variables are defined as follows:

normal_rate = the nominal annual interest rate (as a fraction)

npery = the number of compounding periods per year

The effective interest table for 10% nominal interest rate can be created in Excel as follows
(note that in the case of continuous compounding, npery=1,000,000 is close enough to give the
answer to the desired number of significant digits).
One can compare Table 2.3 with Table 2.1 to see consistency between the calculations in
Excel and those performed with the specific formula for ieff .
The NPER function is useful for determining how many compounding periods are necessary
to achieve a desired result. For example, one might want to determine how many years it will take
for an original investment to double in value if the interest rate is varied from 1% per year to 25%
per year. This is shown in Table 2.4.
The explanation of the values in the NPER formulas in Table 2.4 is as follows: A3/100
represents the interest rate as a fraction, 0 is for no annuity payment, -1 is for a present value
amount of $1, 2 is for a future value of $2, and 0 defines the amounts as end of year payments.
One can also note that the product of the interest rate (as a percentage) and the # of periods to
double the value of the investment varies from 70 to 75. This is commonly known as the Rule of
2.5. INTEREST FORMULAS FOR DISCRETE COMPOUNDING 15

Table 2.3: Using Excel to compute effective interest rates for a nominal 10%
interest rate.

Table 2.4: Using Excel to compute the number of years needed to double the value of
an initial investment.

72. If one takes 72 and divides by the interest rate (as a percentage), the resultant value is a close
approximation of how long it will take for an investment to double.

2.5.5 EXAMPLE PROBLEMS


At this point, it would be beneficial to examine some of the practical applications of these formulas.

Example 2.2
If $10,000 is invested in a fund earning 15% compounded annually, what will it grow to in 10
years?
Solution: F = P (F /P )i,n = 10, 000(F /P )15,10 = 10, 000(4.0456) = $40, 456
16 2. INTEREST AND THE TIME VALUE OF MONEY
Example 2.3
It is desired to accumulate $5,000 at the end of a 15-year period. What amount needs to be
invested if the annual interest rate is 10% compounded semi-annually? Assume the given interest
rate is a nominal rate and that the principal is compounded at 5% per period.
Solution: P = F (P /F )i,n = 5, 000(P /F )5,30 = 5000(0.23138) = $1, 157

Example 2.4
What interest rate, compounded annually, will make a uniform series investment (at the end
of each year) of $1,000 equivalent to a future sum of $7,442? The investment period is 5 years.
Solution: F = A(F /A)i,n 7, 442 = 1, 000(F /A)i,5 (F /A)i,5 = 7.442

Searching the various interest tables in Appendix A for n = 5 yields i = 20%

Example 2.5
An individual wishes to have $6,000 available after 8 years. If the interest rate is 7% com-
pounded annually, what uniform amount must be deposited at the end of each year?
Solution: A = F (A/F )i,n = 6, 000(A/F )7,8 = 6, 000(0.09747) = $585

Example 2.6
An individual wishes to place an amount of money in a savings account and, at the end of
one month and for every month thereafter for 30 months, draw out $1,000. What amount must be
placed in the account if the interest rate is 12% (nominal rate) compounded monthly?
Solution: i(monthly) = 0.12/12 = 0.01(1%)
P = A(P /A)i,n = 1, 000(P /A)1,30 = 1, 000(25.808) = $25, 808

Example 2.7
A principal of $50,000 is to be borrowed at an interest rate of 15% compounded monthly for
30 years. What will be the monthly payment to repay the loan?
Solution: i (monthly) = 0.15/12 = 0.0125(1.25%). Since Appendix A does not contain a
table for that interest rate, one must use the formulas.

A = P (A/P )i,n = 50, 000(A/P )1.25,360


= 50, 000{[(0.0125)(1 + 0.0125)360 ]/[(1 + 0.0125)360 1]}
= 50, 000(0.012644) = $632
2.5. INTEREST FORMULAS FOR DISCRETE COMPOUNDING 17
Example 2.8
An individual deposits $1,000 at the end of each year into an investment account that earns
8% per year compounded monthly. What is the balance in his account after 10 years?
Solution: Since the time frame of the deposits (annually) does not match the time frame of
the interest rate (monthly), one must convert to an effective annual interest rate before computing
the correct formulas.
   
i m 0.08 12
ie = 1 + 1= 1+ 1 = 0.0830
m 12
F = A(F /A)i,n = 1, 000(F /A)8.30,10 = 1, 000{[(1 + 0.0830)10 1]/0.0830}
= 1, 000(14.694) = $14, 694

Example 2.9
Calculate the future worth of the following 6-year cash diagram if the interest rate is 10%
compounded annually.

0 1 2 3 4 5 6

1000 1200 1400 1600 1800 2000

There are a number of ways to solve this economic problem, which is the case for most cash
flow evaluations. One technique might be shorter in terms of the number of formulas to look up or
calculate, but all will result in the same answer.
Solution 1:
Note that this series of cash flows can be broken into an annuity of $1,000 per year and a
gradient of $200 per year. One can compute the future value of each of these contributions separately
and then add to get the final result.

FAnnuity = A(F /A)i,n = 1, 000(F /A)10,6 = 1, 000(7.7156) = $7, 715.60


FGradient = G(F /G)i,n = 200(F /G)10,6 = 200(17.156) = $3, 431.22
F = 7, 715.60 + 3, 431.22 = $11, 147
18 2. INTEREST AND THE TIME VALUE OF MONEY
Solution 2:
Convert the gradient to an equivalent annuity, add this value to the $1,000 annuity and then
convert to the future.

AGradient = G(A/G)i,n = 200(A/G)10,6 = 200(2.2236) = $444.72


ATotal = 1, 000 + 444.72 = $1, 444.72
F = A(F /A)i,n = 1, 444.72(F /A)10,6 = 1, 444.72(7.7156) = $11, 147

Solution 3:
Treat each cash flow as an individual, single payment, find the future value of each individual
payment and then add to get the total future value.

FCF 1 = P (F /P )i,n = 1, 000(F /P )10,5 = 1, 000(1.6105) = $1, 610.50


FCF 2 = P (F /P )i,n = 1, 200(F /P )10,4 = 1, 200(1.4641) = $1, 756.92
FCF 3 = P (F /P )i,n = 1, 400(F /P )10,3 = 1, 400(1.3310) = $1, 863.40
FCF 4 = P (F /P )i,n = 1, 600(F /P )10,2 = 1, 600(1.2100) = $1, 936.00
FCF 5 = P (F /P )i,n = 1, 800(F /P )10,1 = 1, 800(1.1000) = $1, 980.00
FCF 6 = P (F /P )i,n = 2, 000(F /P )10,0 = 2, 000(1.0000) = $2, 000.00
F = 1.610.50 + 1, 756.92 + 1, 863.40 + 1, 936.00 + 1, 980.00 + 2, 000.00 = $11, 147

Example 2.10
Calculate the present worth of the following 10-year cash flow diagram if the annual interest
rate is 20% compounded annually.

0 1 2 3 8 9 10

2000 1900 1800 1300 1200 1100

Solution: Again, there are a variety of methods to solve this problem. One technique is to
recognize that the cash flow is made up of an annuity of $2,000 and a gradient of $100.

ATotal = 2, 000 100(A/G)20,10 = 2, 000 100(3.0739) = $1, 692.61


P = A(P /A)i,n = 1, 692.61(P /A)20,10 = 1, 692.61(4.1925) = $7, 096
2.6. INTEREST FORMULAS FOR CONTINUOUS COMPOUNDING 19

2.6 INTEREST FORMULAS FOR CONTINUOUS


COMPOUNDING
In the last section, the assumption was made that money was received or dispersed and interest rates
were compounded at the end of each discrete compounding period. In some projects (consider a
banking institution for example), money is received and dispersed on a nearly continuous basis. If
the evaluator wishes to consider the effect of continuous cash flow and/or continuous compounding
of interest, one needs to utilize a slightly different set of formulas that relate P , F , and A.

2.6.1 CONTINUOUS COMPOUNDING FOR DISCRETE PAYMENTS


The following formulas apply to the situation where payments (or withdrawals) to an account are
made at discrete points in time, while the account accumulates interest on a continuous basis:
(P /F )i,n = ein (2.24)
(P /A)i,n = (ein 1)/[ein (ei 1)] (2.25)
(F /A)i,n = (ein 1)/[(ei 1)] (2.26)

2.6.2 CONTINUOUS COMPOUNDING FOR CONTINUOUS PAYMENTS


The other application of continuous compounding is the case where the deposits or withdrawals to
an account are being made on a nearly continuous basis. One example of this situation would be a
credit card company that receives charges and payments on millions of cards throughout each day.
For this case, the following definitions need to be made:
P , F , A = the total amount of funds received over one period (present sum, future sum, or
annuity, respectively).
The following figures demonstrate these definitions:
20 2. INTEREST AND THE TIME VALUE OF MONEY

The appropriate formulas are:

(P /F )i,n = [i(1 + i)n ]/[ln(1 + i)] (2.27)


(F /P )i,n = [i(1 + i)n1 ]/[ln(1 + i)] (2.28)
i,n
(F /A) = (ein 1)/ i (2.29)
i,n
(P /A) = (ein 1)/[i(ein )] (2.30)

where, i is the nominal interest rate per period.

2.7 PROBLEMS
2.1. Given a nominal rate of 20%, what is the effective annual interest rate if the interest is
compounded under each of the following scenarios:

(a) Quarterly
(b) Monthly
(c) Daily
(d) Continuously

2.2. What is the percentage difference between the effective rates determined by annual and
continuous compounding for nominal interest rates of:

(a) 10%
(b) 20%
(c) 30%

2.3. A company has decided to invest in a project to make a product. The initial investment cost
will be $1,000,000 to be spread over the first two years with $700,000 in the first year and
$300,000 in the second. The plan calls for producing products at the following rates: 5,000
units in year 2; 10,000 in year 3; 30,000 in year 4; 30,000 in year 5; $10,000 in year 6; and
$5,000 in year 7. Products will be sold for $50 each throughout the life of the project and
cash operating expenses will be $60,000 per year for years 2 through 7. Construct a cash
flow diagram for the project.
2.7. PROBLEMS 21
2.4. Example 2.1 presented a cash flow diagram for an automobile loan as seen through the eyes
of the lender. Construct the corresponding cash flow diagram as seen through the eyes of
the borrower.

2.5. A $1,000 investment has grown to $2,476 in 8 years. What interest rate (compounded
annually) has it earned?

2.6. What present sum is equivalent to a future sum of $25,000 (after 5 years) at an interest rate
of 8% compounded annually?

2.7. If $200 is placed at the end of each year for 10 years in an account earning 7% interest
compounded annually, what amount will be accumulated at the end of 10 years?

2.8. What uniform series would be equivalent to a future sum of $10,000 if the series extends
for 10 years and earns 12% interest compounded semi-annually?

2.9. An annual deposit of $1,000 is placed in an account at the beginning of each year for 5 years.
What is the present value of that series if interest is 12% compounded annually? What is
the future value at the end of the 5th year?

2.10. What will be the future value, 10 years from the first payment, of the series of deposits in
problem 2.9?

2.11. What monthly car payments for the next 30 months are required to amortize a loan of
$4,000 if interest is 12% compounded monthly?

2.12. Payments of $1,000 are to be made at the end of each year for the next 3 years. What is the
present worth of the three payments if interest is 12% compounded monthly? What series
of monthly payments would be equivalent to the $1,000 year payments?

2.13. An individual agrees to lease a building to a firm with yearly payments shown on the cash
flow diagram below. What is the future worth of the payments if interest is 15% compounded
annually?

0 1 2 3 4 5 6 7 8 9 10

3000 3000 3000 3000 3300 3600 3900 4200 4500 4800

2.14. An engineer wishes to buy a house but can only afford monthly payments of $1500. 30-
year loans are available at 5.75% interest compounded monthly. If the engineer can make
a $20,000 down payment, what is the price of the most expensive house that the engineer
can afford to purchase?
22 2. INTEREST AND THE TIME VALUE OF MONEY
2.15. A young woman placed $200.00 in a savings account paying monthly interest. After one
year, her balance has grown to $214.00. What was the effective annual interest rate? What
was the nominal annual interest rate?

2.16. Find the value of cash flow X that will make the two cash flows equivalent. Interest is 10%
compounded annually. Time on the diagram is given in years.

0 1 2 3 4 5 6

100 120 140 160

0 1 2

X X X

2.17. It takes a full $10,000 to put on a Festival of Laughingly Absurd Walks (FLAW) each year.
Immediately before this years FLAW, the sponsoring committee finds that it has $40,000
in an account paying 15% interest compounded annually. After this year, how many more
FLAWs can be sponsored without raising more money?

2.18. If $10,000 is borrowed at 12% interest compounded monthly, what would the monthly
payments be if the loan is for 5 years? What would the annual payment be if the loan is for
5 years? Assume all payments occur at the end of a given period.

2.19. Calculate the value of the following cash flow diagram at the end of year 4. Interest is 10%
per year compounded annually.

0 1 2 3 4 5 6 7 8 9 10

1000 500 500 750 1000 800 600 400 2000


2.7. PROBLEMS 23
2.20. Calculate the future worth 5 years from now of a present sum of $2,000 if:

(a) Annual interest is 10% compounded annually


(b) Annual interest is 10% compounded quarterly
(c) Annual interest is 10% compounded continuously

2.21. Calculate the present value of 10 uniform $2,000 payments if:

(a) Annual interest is 10% compounded continuously and payments are received at the
end of each year
(b) Annual interest is 10% compounded continuously and payments are received contin-
uously over the year

2.22. A gas station sells $125,000 worth of gasoline over the course of a year. If this revenue is
collected and deposited continuously into an account that earns 8% interest, compounded
annually, how much money would the station have in its account at the end of the year?
2.23. Develop an Excel spreadsheet that computes the six functions (P /A), (P /F ), (F /A),
(F /P ), (A/P ), (A/F ) for a fixed interest rate and the number of periods ranging from
1 to 100.
2.24. Use the Excel NPER function to determine how long it will take for an investment to
triple in value at interest rates of 1%, 5%, 10%, 15%, 20%, and 25%. Can you determine
an approximate Rule for how to quickly calculate how long it takes for an investment to
triple in value?
25

CHAPTER 3

Project Evaluation Methods


3.1 INTRODUCTION
In order to make informed decisions on one or more potential investments, methods must be devel-
oped that provide a numerical evaluation of a project. Both equivalence and rate of return methods
will be developed in this chapter.
Consider the following cash flow diagrams that contain income generating streams.
A:
0 1 2 3 18 19 20

1,000,000
B:
0 1 2 3 18 19 20

100,000 100,000 100,000 100,000 100,000 100,000

Since there are no cash flows for A after period 0, the present value of cash flow A is simply
$1,000,000. For B, since the $100,000 occurs at the end of each period for 20 periods, multiplying
the $100,000 by (P /A)i,20 will yield a present value for the interest rate used in the formula. For
example, if the interest rate is 12% per year, the present value would be $746,944. If the question
which cash flow represents the largest present value? is asked, the answer is obviously cash flow A.
Now consider a different question. Suppose you have just won a lottery and you have a choice
of receiving $1,000,000 now or receiving $100,000 at the end of each year for 20 years. If interest
is expected to be constant at 12% for the next 20 years as in the previous paragraph, which set of
payments would you prefer? Since this question is represented by the cash flow diagrams shown
above and the interest rate of 12%, the choice can be made by analyzing the present values of the two
cash flow diagrams. Since cash flow A yields a larger present value than cash flow B at an interest
rate of 12%, the proper choice would be to accept option A.
26 3. PROJECT EVALUATION METHODS
However, what if the interest rate is expected to be 0% over the 20 year period? What would
the best choice be under that scenario? If interest is 0%, then money is worth the same no matter
when it occurs. At 0% interest, the present value of cash flow B becomes $2,000,000 and cash flow
B becomes the correct choice.
The discussion in the previous two paragraphs infer that at some interest rate between 0%
and 12%, the two cash flow diagrams are equivalent. A trial and error solution yields this interest
rate to be about 7.75%.
This discussion has just introduced two of the more popular techniques (equivalence methods
and rate of return methods) used to evaluate the financial value of projects and help the evaluator
choose between multiple projects. These will be discussed in more detail later in this chapter.

3.2 ALTERNATE USES OF CAPITAL


Investment analysis or project evaluation involves making a decision between alternative uses of
capital. A cash flow diagram is constructed for each alternative according to the specific parameters
of that alternative and evaluated using the concepts of time value of money that were discussed in
Chapter 2. The results of the evaluations are then compared and a decision is made as to which
alternative is the best option.
Several evaluation methods can be used in analyzing investment opportunities. Two general
types of calculations that will be introduced here are: (1) equivalence methods which involve the
determination of an equivalent present, annual, or future worth of a cash flow diagram given a
specific interest rate; and (2) rate of return methods which involve the determination of an interest
rate produced by the cash flow diagram.

3.3 MINIMUM ACCEPTABLE RATE OF RETURN (MARR)


When using either the equivalence method or the rate of return method for comparing alternatives,
a minimum acceptable rate of return, MARR, needs to be defined. The value of MARR is set as
the lower limit for investment which is acceptable to an individual or a company. The MARR may
vary from individual to individual, company to company, and even within the structure of a specific
company.
The lower bound for the MARR is generally set at the cost of capital, which reflects the
expense of obtaining funds for a given project. How much higher the MARR is above the cost
of capital depends on a particular companys or individuals position and the particular project. For
example, an individual who borrows money at 5% interest rate in order to invest in a profit-generating
project would have an MARR of at least 5%, but would probably want to set the MARR at, say, 10%
in order to generate a net increase in his/her personal worth based on the estimated profitability of
the project. Similarly, if individuals are using their own funds to invest, their cost of capital would
be the interest rate that their money is currently earning in a savings account, certificate of deposit,
or other investments. A companys MARR is usually set by the portfolio of projects in which the
3.4. EQUIVALENCE METHODS 27
company can invest. That is, what is the minimum interest that a company can earn by investing its
money in what it would consider to be a guaranteed success? For engineers performing economic
evaluations for their companies, the MARR will be provided by upper management so that they
will not have to make that determination.

3.4 EQUIVALENCE METHODS


In the equivalence methods to determine either the acceptability of a single project or to choose
the best project, the MARR is used as the interest rate in present, future, or annuity calculations.
A net present value, NPV (sometimes called the net present worth), net future value, NFV, or net
annual value, NAV, is calculated by one of the following equations:

NP V = Present Value of Cash Flows with i = MARR (3.1)

NF V = Future Value of Cash Flows with i = MARR (3.2)

N AV = Annuity Value of Cash Flows with i = MARR (3.3)
Since N P V , N F V , and N AV are related by the interest formulas developed in Chapter 2, any one
of the three calculations will yield the same conclusion (in terms of economic viability of the project)
as the other two. Because of this fact, most analysts concentrate on the NP V method, as do the
authors of this text.

3.5 NET PRESENT VALUE


3.5.1 ANALYSIS OF A SINGLE INVESTMENT OPPORTUNITY
For a single investment opportunity, the NP V would be calculated using the MARR as the interest
rate. A positive value for N P V indicates that the project which is represented by the cash flow
diagram earns an actual interest rate greater than the MARR, a negative value for NP V indicates
that it earns an actual interest rate less than the MARR, and an NP V value of zero indicates that it
earns the MARR. Since the MARR represents the decision point for determining the viability of
a project for a particular investor, a positive NP V would indicate that the project is an acceptable
one.
Example 3.1
Consider the project represented by the following cash flow diagram. The project requires an
initial investment of $1,000 that returns positive cash flows as shown. The MARR is 10%.

0 1 2 3 4 5

-1000 500 600 700 800 900


28 3. PROJECT EVALUATION METHODS

N P V = 1000 + 500(P /A)10,5 + 100(P /G)10,5


= 1000 + 500(3.7908) + 100(6.8618) = $1582

Since the N P V is greater than zero, this project would be an acceptable one to the investor.
An alternative method to calculate the NP V is to treat each individual cash flow as a future
value at various values of n. While this technique might require more formulas than recognizing
annuities and gradients in the cash flow diagram, it will always yield a correct value for NP V :

N P V = 1000 + 500(P /F )10,1 + 600(P /F )10,2 + 700(P /F )10,3


+ 800(P /F )10,4 + 900(P /F )10,5
N P V = $1582

In Excel , one can use the NP V function to make the same calculation. However, some
caution is necessary.
The function is: = NP V (rate, value1, value2, ).

where, rate = interest rate per period (as a fraction).


value1, value2, ... = cash flows that occur at the end of period 1, end of period 2, etc.

One can see that the NPV function does not include the investment period 0. Therefore, in order
to calculate the N P V of the entire cash flow diagram, one needs to include the initial investment.
For example, the complete Excel formula to compute the NP V of a series of cash flows would be
as shown in Figure 3.1:
= CF0 + NP V (rate, value1, value2,)

One can see that the results from Excel match the NP V calculations from the other two
methods.

Example 3.2
Consider the project represented by the following cash flow diagram. The project requires an
initial investment of $1,000 that returns positive cash flows as shown. The MARR is 10%.

0 1 2 3 4 5

-1000 150 200 250 300 350


3.5. NET PRESENT VALUE 29

A B A B
1 MARR = 10% 1 MARR = 0.1

3 Year CF 3 Year CF
4 0 -1000 4 0 -1000
5 1 500 5 1 500
6 2 600 6 2 600
7 3 700 7 3 700
8 4 800 8 4 800
9 5 900 9 5 900
10 10
11 NPV = 1582 11 NPV = =B4+NPV(B1,B5:B9)

Figure 3.1: Demonstration of the use of the N P V function in Excel .

N P V = 1000 + 150(P /A)10,5 + 50(P /G)10,5


= 1000 + 150(3.7908) + 50(6.8618) = $88

Since the N P V is negative, the project will not earn the MARR and, therefore, is not accept-
able to this investor. Now a question arises: What does the investor do with the $1000? Since the
time-line represents the only new investment opportunity available to the investor and the NP V
analysis suggests that it is not acceptable, the investor will choose to do nothing with the $1000.
The concept of the do nothing project will be defined in the next section.

3.5.2 DO NOTHING PROJECT


Example 3.2 indicates that there is always a choice to do nothing with investment funds. That is,
even if a project, like the one described in Example 3.2, is the only new investment available and
the financial analysis indicates that it is unacceptable, an investor can always choose to keep the
proposed funds, $1000 in the case of Example 3.2, where they currently are and do nothing with
those funds.
The do nothing project does not mean that the investment funds are going to be buried
in a can in the backyard where they earn nothing. The do nothing project means that the funds
are already invested in a project that is earning the MARR. As mentioned before, for individuals,
this could mean leaving their funds in their savings accounts. By definition, the NP V of the do
30 3. PROJECT EVALUATION METHODS
nothing project is zero.Thus, when a single investment opportunity is being evaluated, one is always
comparing it against a second opportunity which is to leave the money in the do nothing project.

3.5.3 ANALYSIS OF MULTIPLE INVESTMENT OPPORTUNITIES


For the purpose of this initial discussion of investing in multiple projects, assume that all of the
prospective projects to be evaluated require the same initial investment, that the investor only has
enough funds to invest in one of the projects, and that the decision will be based solely on NP V
analysis. These assumptions will be removed in subsequent chapters and discussed further. In addi-
tion, if at least one of the proposed projects has a positive NP V , then the do nothing project need
not be considered.

Example 3.3
Consider the following two investment opportunities. The investors MARR is 10% and the
investor only has enough funds to invest in one of the projects. Which one should be chosen?
Project A:

0 1 2 3 4 5

-800 215 215 215 215 215

Project B:

0 1 2 3 4 5

-800 100 100 100 100 900

N P V for Project A = 800 + 215(P /A)10,5 = $15.0


N P V for Project B = 800 + 100(P /A)10,5 + 800(P /F )10,5 = $75.8

Both projects show positive values of NP V . Therefore, both would be acceptable as long as
the investor had at least $800 to invest. In addition, the do nothing alternative does not need to be
considered. If the investor only has enough funds to invest in one of the projects, the NP V values
indicate that Project B is the best economic choice.
3.6. RATE OF RETURN METHODS 31
3.6 RATE OF RETURN METHODS
The second general type of project evaluation technique involves the determination of an unknown
interest rate for a given cash flow diagram. This interest rate is usually referred to as a rate of return.
There are several rates of return that can be calculated. Two will be presented in this chapter. The
first is called the Internal Rate of Return (IRR) which is also known as the Discounted Cash Flow
Rate of Return (DCFROR). The second is the External Rate of Return (ERR) which is also known
as the Growth Rate of Return. The I RR is the rate of return earned by a particular individuals or
companys investment.The ERR represents the overall growth of invested dollars for an individual or
a company. The differences will become apparent in the following discussion and example problems.

3.6.1 INTERNAL RATE OF RETURN (IRR)


The I RR is defined as the interest rate which discounts a series of cash flows to an NP V value of
zero: 
NP V = 0 = Present Value of Cash Flows with the interest rate equal to I RR (3.4)
The equation can also be written as:


n 
n
CFj
NP V = 0 = CFj (P /F )I RR,j = (3.5)
(1 + I RR)j
j =0 j =0

where, CFj = cash flow for period j


j = period of cash flow
n = total number of periods

It should be noted that one cannot normally solve explicitly for the I RR from Equation 3.5.
Therefore, a trial and error solution is usually required. Graphically, the relationship between NP V ,
interest rate, and I RR is demonstrated in Figure 3.2.
Once the I RR is calculated, it is then compared with the MARR. If the I RR is greater than
the MARR, the project is considered to be acceptable to the investor.
32 3. PROJECT EVALUATION METHODS

NPV vs Interest Rate


600
500
400
300
200 IRR = 0.125
NPV, $

100
0
-100 0 0.05 0.1 0.15 0.2 0.25
-200
-300
Interest Rate, fraon

Figure 3.2: General form of net present value as a function of interest rate. (Note, for this example, when
N P V = 0, the interest rate, or I RR, is 0.125.)

Example 3.4
Consider the two investment opportunities examined in Example 3.3. The investors MARR
is 10% and the investor only has enough funds to invest in one of the projects. What are the I RRs
for each project?
Project A:
0 1 2 3 4 5

-800 215 215 215 215 215


3.6. RATE OF RETURN METHODS 33
Project B:
0 1 2 3 4 5

-800 100 100 100 100 900

As noted, the calculation of I RR usually involves a trial and error approach. While the NP V
versus interest rate curve is not a straight line, it is generally accurate enough to bracket the I RR
solution within 5% and then linearly interpolate for the answer.
Project A: N P V for Project A = 800 + 215(P /A)i,5

 
Interpolating for I RR: I RR = 10.0 + 15.00
15.0(79.3) (15.0 10.0) = 10.8%

Project B: N P V for Project B = 800 + 100(P /A)i,5 + 800(P /F )i,5

Interest rate, % N P V
0.0 100.0
10.0 75.8
15.0 -67.0
 
Interpolating for I RR: I RR = 10.0 + 75.8(67.0)
75.80
(15.0 10.0) = 12.6%. It should be
noted that Figure 3.2 was generated with the cash flows from Project B. Thus, the true answer for
I RR is 12.5% compared to the interpolated value of 12.6%.
In this example, the I RRs of both projects are greater than the investors MARR, so both
projects are acceptable. It would appear that since the I RR of Project B is greater than the I RR of
Project A, then Project B is the best alternative. This is, indeed, the proper interpretation but only
because the initial investment values for both projects were the same. One must be very careful in
ranking projects by I RR values as will be shown in Chapter 5.

3.6.2 SPREADSHEET FORMULA FOR IRR


Excel has a built-in function to calculate Internal Rate of Return.
34 3. PROJECT EVALUATION METHODS
The function is: = I RR(values, guess)
where, values = cash flows that occur for the project
guess = initial estimate of the IRR (as a fraction)
This function automatically takes care of the year 0 cash flow without having to include it as a
separate term such as was necessary in the NP V calculation with Excel . One can see that the cash
flows in Figure 3.3 are the same as Project B in the previous example.

A B A B
1 MARR = 10% 1 MARR = 0.1

3 Year CF 3 Year CF
4 0 -800 4 0 -800
5 1 100 5 1 100
6 2 100 6 2 100
7 3 100 7 3 100
8 4 100 8 4 100
9 5 900 9 5 900
10 10
11 NPV = 75.8 11 NPV = =B4+NPV(B1,B5:B9)
12 IRR = 12.5% 12 IRR = =IRR(B4:B9,0.1)

Figure 3.3: Demonstration of the use of the N P V and I RR functions in Excel .

As in Figure 3.2, Excel provides the true value for I RR without the need for a trial and
error solution and without interpolating.

3.6.3 EXTERNAL RATE OF RETURN (ERR)


The External Rate of Return (ERR) or Growth Rate of Return is found by determining the interest
rate that will satisfy the following equation.
 
 n 
   n
 Cj (P /F )MARR,j  = Ij (F /P )MARR,nj (P /F )ERR,n (3.6)

j =0  j =0

where, Cj = negative cash flow at period j


Ij = positive cash flow at period j
n = life of project
3.6. RATE OF RETURN METHODS 35
The equation states that positive cash flows (Ij s) derived from the project are reinvested at the
MARR to generate a future value, which is called FI , at the end of the project life. All negative cash
flows (investments) are brought back in time at the MARR to generate a present value, which is
called PC , at year zero. The interest rate which will then discount FI to a value equal to the value of
PC is determined to be the ERR.
Another way of looking at the external rate of return is to set up a second project which is
called the reinvestment project. The negative cash flows for the reinvestment project are the positive
cash flows from the original project. A future value of the cash flows of the reinvestment project is
determined using the MARR as the interest rate (FI ). The original project and reinvestment project
are then added together to give a third project. The positive cash flows from the original project and
the costs from the reinvestment project should have netted out to zero. The remaining cash flows for
the third project will be the negative cash flow at year zero, any other negative cash flows from the
original project at the year of occurrence, and the future value determined for the second project. All
negative cash flows are brought back to time zero at the MARR to generate a present value (PC ).
The ERR is then determined by finding the interest rate which will bring the future value to a year
zero value equal to the present value of the negative cash flows.
The ERR method has a calculation advantage over the I RR method in that the ERR can
be solved for directly without a trial and error procedure. The steps in the calculation procedure are:
 
 n 
 
PC =  Cj (P /F )MARR,j  (3.7)
j =0 
 n
FI = Ij (F /P )MARR,nj (3.8)
j =0
ERR = (FI /PC )1/n 1 (3.9)

Example 3.5
Consider the two investment opportunities examined in Example 3.4. The investors MARR
is 10% and the investor only has enough funds to invest in one of the projects. What are the ERRs
for the projects?
Project A:
0 1 2 3 4 5

-800 215 215 215 215 215


36 3. PROJECT EVALUATION METHODS
Project B:
0 1 2 3 4 5

-800 100 100 100 100 900

Project A: PC = | 800| = 800


FI = 215(F /A)10,5 = 1312.6
ERR = (1312.6/800)1/5 1 = 0.104 = 10.4%

Project B: PC = | 800| = 800


FI = 100(F /A)10,5 + 800 = 1410.5
ERR = (1410.5/800)1/5 1 = 0.120 = 12.0%

In this example, the ERRs of both projects are greater than the investors MARR, so both
projects are acceptable. It would appear that since the ERR of Project B is greater than the ERR
of Project A, then Project B is the best alternative. This is, indeed, the proper interpretation but
only because the initial investment values for both projects were the same. Again, one must be very
careful in ranking projects by ERR values as will be shown in Chapter 5.
One additional observation can be made about the relationship between MARR, I RR, and
ERR. The ERR will always lie between the MARR and the I RR. Thus,

MARR ERR I RR or MARR ERR I RR

Example 3.6
Consider the investment opportunity below. The investors MARR is 10%. What are the
N P V , I RR, and ERR values for the project?
Project A:
0 1 2 3 4 5

-1000 500 500 -200 500 500


3.7. THE REINVESTMENT QUESTION IN RATE OF RETURN CALCULATIONS 37
NPV:

N P V = 1000 + 500(P /F )10,1 + 500(P /F )10,2 200(P /F )10,3


+ 500(P /F )10,4 + 500(P /F )10,5
N P V =$369.4

IRR:

Interest rate, %
NP V
10.0 369.4
20.0 90.2
30.0 -100.8
25.0 -13.8
 
Interpolating between 20% and 25%: I RR = 20.0 + 90.2(13.8)
90.20
(25.0 20.0) = 24.3%

ERR: PC = | 1000 200(P /F )10,3 | = $1150.3


FI = 500(F /P )10,4 + 500(F /P )10,3 + 500(F /P )10,1 + 500 = $2447.6
ERR = (2447.6/1150.3)1/5 1 = 0.163 = 16.3%

All three economic indicators show that this project is an acceptable one.

3.6.4 SPREADSHEET FORMULA FOR ERR


Excel has a built-in function that can be used to calculate the External Rate of Return.
The function is: = MI RR(values, finance_rate, reinvestment_rate)

where, values = cash flows that occur for the project


finance_rate = interest rate for discounting the negative cash flows to year 0 (as a fraction)
reinvestment_rate = interest rate for reinvesting the positive cash flows to year n (as a fraction)

One needs to set both the finance_rate and the reinvestment_rate to MARR. As with I RR, this
function automatically takes care of the year 0 cash flow without having to include it as a separate
term. Figure 3.4 demonstrates this formula (along with NPV and IRR) for the cash flows given in
Example 3.6.

3.7 THE REINVESTMENT QUESTION IN RATE OF RETURN


CALCULATIONS
The virtues of the I RR calculation have been argued for years by evaluators. When the I RR method
was first introduced, it was met with a great deal of enthusiasm and is still one of the most popular
38 3. PROJECT EVALUATION METHODS

A B A B
1 MARR = 10% 1 MARR = 0.1

3 Year CF 3 Year CF
4 0 -1000 4 0 -1000
5 1 500 5 1 500
6 2 500 6 2 500
7 3 -200 7 3 -200
8 4 500 8 4 500
9 5 500 9 5 500
10 10
11 NPV = 369.5 11 NPV = =B4+NPV(B1,B5:B9)
12 IRR = 24.3% 12 IRR = =IRR(B4:B9,0.1)
13 ERR = 16.3% 13 ERR = =MIRR(B4:B9,B1,B1)

Figure 3.4: Demonstration of the use of the N P V , I RR, and MI RR(ERR) functions in Excel .

evaluation methods used. Surveys have indicated that a vast majority of the companies polled use
I RR either by itself or in conjunction with other methods when evaluating projects. However, in
spite of the popularity of the I RR method, many evaluators still question its meaning and validity.
The basic question has to do with whether or not a reinvestment of incomes is implied in the
calculation procedure. That is, one argument is that in order for the original project investment to
earn the I RR, the positive cash flows generated by the project must be reinvested in another project
that earns the same I RR. The other argument is that reinvestment is not necessary to earn the
I RR. In fact, both arguments may be true depending on the evaluators perception of what is meant
by the phrase earning the IRR.
To begin the discussion of the reinvestment question, consider Example 3.7.
3.7. THE REINVESTMENT QUESTION IN RATE OF RETURN CALCULATIONS 39
Example 3.7
An investment of $5000 will yield $1931.45 at the end of each year for 4 years. What is the
value of the projects I RR? If the MARR is 15%, what is the projects ERR?

0 1 2 3 4

-5000 1931.45 1931.45 1931.45 1931.45

I RR : NP V = 5000 + 1931.45(P /A)i,4


For NP V = 0, (P /A)I RR,4 = 2.5887

Examining the interest tables in Appendix A, one can determine that the I RR is 20.0%.

ERR : PC = | 5000| = $5000


FI = 1931.45(F /A)15,4 = $9644
ERR = (9644/5000)1/4 1 = 0.178 = 17.8%

By definition, the calculation of ERR requires that the incomes be reinvested at the MARR of
15%. If the MARR had been higher, say 18%, the value of the ERR would have been higher. If
the MARR were 20%, one can show that the ERR is now equal to 20% (same as the I RR). Thus,
if the interest rate used for the reinvestment of incomes and for finding the present value of the
costs (negative cash flows) is the I RR, then the values of MARR, I RR, and ERR will be identical.
While not shown here, this can be demonstrated, mathematically, for any set of cash flows.

Now, lets expand on this example in order to determine the effect of different perceptions of
an investment earning a particular interest rate.

3.7.1 PERCEPTION #1
The first perception of an investment earning a particular interest rate parallels the concept of
investing money in a savings account for a specified period of time. In this perception, earning
means that an initial investment will yield a future value given by (F /P )i,n . Using the values from
this example, a $5000 investment earning 20% (the I RR) for 4 years should result in a future sum
of:
F = 5000(F /P )20,4 = $10, 368
However, if the individual cash flows of $1931.45 (recall that these cash flows yielded an I RR of
20%) were buried in a can under a tree (thus earning no interest), the total future accumulated
40 3. PROJECT EVALUATION METHODS
amount would be:
F = (4)(1931.45) = $7, 725.80
Since the four individual cash flows yield a future sum significantly less than $10,368, the initial
investment has not earned a 20% interest rate according to this perception of earning. In fact,
the actual rate of return would be:

i = (7725.80/5000)1/4 1 = 0.115 = 11.5%, not 20%!

However, if the individual cash flows were reinvested in an account that earned 20% interest,
the future sum accumulated in that account would be:

F = 1931.45(F /A)20,4 = $10, 368

and the earned interest rate would indeed be 20%. Thus, in this perception, in order to earn the
I RR (20%) interest rate on the entire initial investment ($5,000), any cash flows received before the
end of the project must be reinvested in another project that has the same I RR.

3.7.2 PERCEPTION #2
The second perception more closely parallels the concept of making a loan to a project and having
that loan be paid back at some interest rate. In this perception, interest is earned only on the portion
of the total loan that is still unpaid. The unpaid portion of the loan is also known as the unamortized
portion.
Again, consider the cash flows in Example 3.7. During the first year, interest is earned on
the entire $5000 investment (or loan). The required interest amount at the calculated I RR of 20%
would be:
I1 = 5000(0.20) = $1000
This means that $931.45 can be used to payback a portion of the original investment, leaving an
unamortized amount of $4,068.55. The required interest amount in the second year would then be:

I2 = 4068.55(0.20) = $813.71

The reminder of that years cash flow, $1,117.74, would be used to further reduce the unamortized
portion of the investment to $2,950.81. Table 3.1 summarizes this sequence for the entire project
life.
Note that the total interest earned is the same as would have been earned under perception
#1 if the cash flows were not reinvested. However, banking institutions agree that this repayment
scheme has indeed earned 20% on the original loan of $5,000.
In the opinion of the authors, the final conclusion is that the question of whether reinvestment
of the cash flows at the I RR must occur or not is really more of an issue of perceiving what is meant
by earning a return. Banking institutions readily invest in projects via loans to companies or
3.8. ACCELERATION PROJECTS 41

Table 3.1: Amortization table for a loan

individuals and receive the I RR as defined in perception #2 without automatic reinvestment at that
same rate. However, an individual or company that is expecting to generate a future sum of money
based on earning the I RR on the original investment for the entire life of the project must depend
on reinvestment of the cash flows at that specific I RR in order to actually have the desired future
sum.
It should be noted that, independent of the reinvestment question, I RR analysis still results
in a powerful economic evaluation tool.

3.7.3 FINAL COMMENTS ON ERR AND IRR RELATIONSHIPS


The ERR is a measure of the growth of the investment dollars. The I RR does not have the same
meaning since it is a measure of the project profitability only. If a company wants a true measure of
its growth based on a specific investment, then ERR analysis should be used.
Both the I RR and ERR are valid investment analysis techniques and, if applied correctly,
will yield the same conclusion regarding the viability of an investment to the company or individual.
It will be shown in the next section that the ERR method has some advantages in particular analysis
situations.

3.8 ACCELERATION PROJECTS


When a series of cash flows changes from a positive value to a negative value (or negative to positive)
more than once, the cash flows may generate multiple positive real solutions to the I RR equation.The
number of solutions is governed by Descartes rule. The rule states that if the terms of a polynomial
with real coefficients are ordered by descending variable exponent, then the number of positive
roots of the polynomial is equal to the number of sign differences between consecutive nonzero
42 3. PROJECT EVALUATION METHODS
coefficients. Since the I RR equation can be rearranged to form a polynomial of order n, this rule
will apply since the coefficients will be related to the cash flows.
A series of cash flows with more than one sign change is called an acceleration project.This type
of project is created when a second capital investment must occur after one or more years of positive
incomes. For example, consider a manufacturing facility that will require significant upgrading after
several years. The multiple values of I RR rates calculated when there are multiple sign changes are
difficult to interpret as to which might be the correct return on investment.
Since the ERR equation does not form a polynomial, it always has a unique answer and,
therefore, should be the rate of return technique of choice in acceleration projects. A modified I RR
calculation can be made by finding the present value of all of the negative cash flows by discounting
to year 0 at the MARR and then using the normal I RR equation. It should be noted that the investor
can always use the equivalence methods (NP V specifically) in this situation without difficulty.
In Example 3.6, a cash flow was presented that had sign changes between the 2nd and 3rd year
and the 3rd and 4th year. In this case, the analyst should be aware that multiple positive values of I RR
might exist. For that specific example, the nth order polynomial that is created by the NP V = 0
equation is developed as follows:

1000 + 500(P /F )I RR,1 + 500(P /F )I RR,2 200(P /F )I RR,3


+ 500(P /F )I RR,4 + 500(P /F )I RR,5 = 0

500 500 200 500 500


1000 + + + + =0
(1 + I RR) (1 + I RR) 2 (1 + I RR) 3 (1 + I RR) 4 (1 + I RR)5

I RR 5 + 4.5 I RR 3 + 7.5 I RR 3 + 5.7 I RR 2 + 1.4 I RR 0.8 = 0

Since the 5th order polynomial only has one sign change, there is only one positive value of
I RR for the cash flows in Example 3.6. Example 3.8 will demonstrate a situation where more than
one positive value exists.
3.8. ACCELERATION PROJECTS 43
Example 3.8
Given the following cash flow diagram, plot the NP V versus interest rate and determine the
two positive values of I RR that would be predicted by Descartes rule. Assume an MARR of 5%.

Solution: From the plot of NP V versus interest rate and the Excel spreadsheet, it can be
seen that there are two values for I RR: 9.1% and 57.2%. One can use Excel to find both rates of
return by adjusting the initial guess. An initial guess of 10% will yield the 9.1% value and an initial
guess of 50% will yield the 57.2% value. This creates an unfortunate situation in that one must have
an idea of the value of the larger root in order to have Excel compute it.
The 6th order polynomial that could be developed is:
I RR 6 + 5.1 I RR 5 + 9.3 I RR 4 + 5.4 I RR 3 2.7 I RR 2 3.1 I RR + 0.3 = 0
One can see that there are two sign changes in the list of terms and, therefore, two positive values
for I RR.
As mentioned before, the multiple values of IRR cause difficulties in interpretation. With a
total investment (without time value of money) of $320 and the total of the positive incomes (without
time value of money) of $290, one would be hard pressed to accept that this project earns 9.1%,
let alone 57.2%! Comparing 9.1% to the MARR of 5% would seem to indicate that this project is
acceptable.
44 3. PROJECT EVALUATION METHODS
Lets examine the ERR, NP V , and modified I RR for this project:

ERR : PC = | 100 90(P /F )5,4 80(P /F )5,5 50(P /F )5,6 | = $274.0


FI = 90(F /P )5,5 + 120(F /P )5,4 + 80(F /P )5,3 = $353.3
ERR = (353.3/274.0)1/6 1 = 0.043 = 4.3% .
NP V : The table or the figure show that the NP V at an interest rate of 5%
(the investors MARR) is -$10.4.

The modified I RR would be calculated by replacing the negative cash flows with PC calculated
above to create a new set of cash flows as follows:
0 1 2 3 4 5 6

-274 90 120 80 0 0 0

Using the trial and error solution technique or Excel , the modified I RR is 2.9%.
Thus, the ERR, the modified I RR, and the NPV indicate that this project is not an acceptable
project for the investor.
In summary, acceleration projects have the potential to add another level of complexity to
the calculation of I RR in that multiple positive rates may exist. The authors strongly suggest that
evaluators utilize N P V or ERR calculations to determine the economic viability of acceleration
projects.

3.9 PAYOUT
A supplementary evaluation technique that is frequently used is payout period or simply payout.
Payout may be calculated with or without discounting although it is usually calculated without
considering the time value of money. Payout refers to the time that it takes for a project to return its
initial investment. Thus, its a quick measure of how long the investment is at risk. Although this
time may be a very useful piece of information to compute for a particular project, payout analysis is
limited in its use as an evaluation criterion. It does not serve as a useful screening criterion since it
ignores any cash flows occurring past the payout period. Therefore, it must be used in conjunction
with one of the evaluation techniques that have already been presented.

Example 3.9
Given the following cash flow diagram, compute the undiscounted payout time and the
discounted payout time if MARR is 15%.
3.9. PAYOUT 45
0 1 2 3 4

-100 60 60 60 60

Undiscounted Payout:

Year Cash Flow Cumulave


Cash Flow
0 100 100
1 60 40
2 60 20

Interpolate between years 1 and 2 to find when the cumulative cash flow equals zero:
 
40 0
Payout = 1 + (2 1) = 1.67 years
40 (20)

Discounted Payout:

Year Cash Flow Discounted Cash Flow Cumulave Discounted


Cash Flow
0 100 100 100
1 60 60(P /F ) 15.1 = 52.2 47.8
2 60 60(P /F ) 15.2 = 45.4 2.4
3 60 60(P /F ) 15.3 = 39.4 37.0

Interpolate between years 2 and 3 to find when the cumulative cash flow equals zero:
 
2.4 0
Payout = 2 + (3 2) = 2.06 years
2.4 (37.0)

Discounted payout measures the time for the project to return the initial investment and a 15% rate
of return on that initial investment.
46 3. PROJECT EVALUATION METHODS
3.10 PROBLEMS

3.1. Calculate the present value and annual value of the following cash flow diagram. MARR
is 15%.

0 1 2 3 4 5 6 7

-2500 500 650 800 800 800 800 800

3.2. Calculate the I RR and ERR for the cash flow diagram given in Problem 3.1.

3.3. An individual is considering the purchase of a property that he believes he can resell for
$25,000 at the end of 10 years. The property will generate positive cash flows of $1,500 per
year for the 10 years. What is the maximum that the individual should pay for the property
if his MARR is 12%?

3.4. An investment of $10,000 will yield $33,000 at the end of 5 years with no other cash flows.
What is the I RR of this investment?

3.5. Calculate the I RR for the following cash flow diagram.

0 1 2 3 4 5

-2000 -500 1000 1000 1000 1000

3.6. A company invests $30,650 in a project which yields an income (positive cash flow) of
$10,000 in the first year, $9,000 in the second, $8,000 in the third, etc and $1,000 in
the tenth, along with an extra $10,000 income at the end of year 10. The companys MARR
is 10%. Determine the I RR and ERR of this project.
3.10. PROBLEMS 47
3.7. Determine the N P V , ERR, and modified I RR for the following cash flow diagram. Use
an MARR of 15%.

0 1 2 3

-50 100 100 -100

3.8. Determine the N P V , NAV , modified I RR, and ERR for the following cash flow diagram
if the MARR is 10%.

0 1 2 3 4

-75 50 50 -30 200

3.9. You are a project engineer and you have to make a choice between two contractors to perform
some rebuilding work on a manufacturing facility. One contractor proposes that he will do
the work for $1,300,000 payable immediately. The other contractor proposes that he will
perform the same job for $1,400,000 payable in eight equal quarterly payments, starting 3
months after the job begins. A nominal rate of 14% should be used as the MARR. What
equivalent annual interest rate is the second contractor offering? Which contractors offer
would you accept? Repeat the analysis with the NP V technique.

3.10. John Q. Customer has received his bill for the next 6 months premium on his auto insurance.
The bill allows him two methods to pay his premium of $189.00. He can either pay the
entire amount now, or he can pay $99.00 now, which includes half of the premium plus a
$4.50 prepaid service charge and $94.50 in two months, the other half of the premium.
The insurance company is, implicitly, offering John a loan. What is the effective annual
interest rate of the loan? Would you take the loan? Why or why not?
48 3. PROJECT EVALUATION METHODS
3.11. A project is expected to cost $2,000,000 and have the following net revenues:

Year Net Revenue


1 1,000,000
2 800,000
3 600,000
4 400,000
5 200,000
6 100,000

Calculate the undiscounted and discounted payout periods. The MARR is 15%.
3.12. Engineer A retires at the age of 65 with a retirement account worth $500,000. At what
interest rate would this amount need to be invested in order to withdraw $50,000 at the
end of each of the next 15 years?
3.13. Develop an Excel spreadsheet to compute NP V , NAV , NF V , I RR, and ERR for the
cash flow diagram given in Problem 3.1.
3.14. Develop an Excel spreadsheet to solve Problem 3.3 for MARR values of 5%, 10%, 12%,
15% and 20%.
3.15. Develop an Excel spreadsheet to solve Problem 3.4 for initial investments of $5000,
$10000 and $15000.
3.16. Develop an Excel spreadsheet to solve Problem 3.5 for initial investments of $2000, $1500,
and $1000.
3.17. Develop an Excel spreadsheet to solve Problem 3.6.
3.18. Develop an Excel spreadsheet to solve Problem 3.7.
3.19. Develop an Excel spreadsheet to solve Problem 3.8.
3.20. Develop an Excel spreadsheet to solve Problem 3.9.
3.21. Develop an Excel spreadsheet to solve Problem 3.10.
3.22. Develop an Excel spreadsheet to solve Problem 3.11 for MARR values of 5%, 10%, and
15%.
3.23. Develop an Excel spreadsheet to solve Problem 3.12.
49

CHAPTER 4

Service Producing Investments


4.1 INTRODUCTION
There are, in general, two types of investmentsone which produces income and one which produces
a service. A service producing investment is one that results in a cash flow diagram that normally
contains no positive cash flows with the exception of a possible salvage value of the service. Salvage
value is the estimated value of an asset at the end of its useful life. It is assumed that the asset can
be sold (as scrap metal for example) for this value as a positive cash flow to the project. The authors
use the symbol L to represent the positive cash flow due to salvage value.
An example of a service producing investment would be the consideration of either purchasing
a new vehicle for a field office or leasing the vehicle. The vehicle provides a necessary service for the
personnel in the field office but does not directly produce any income for the company. Generally, a
leased vehicle would not have any salvage value since it is just returned to the leasing agency at the
end of the lease period, while a purchased vehicle would have some salvage value since it could be
sold to another owner.
This chapter will discuss evaluation techniques for service producing investments for equal
and unequal life alternatives.

4.2 EQUAL LIFE ALTERNATIVES


Consider the following situation. An investment needs to be made by a company for a particular
service that is necessary for the company to conduct its business. Two or more alternatives have been
identified that provide the same service over the same time period. These alternatives are known
as equal life alternatives and they lend themselves to straight forward application of the evaluation
methods that were presented in Chapter 3.

4.2.1 EQUIVALENCE TECHNIQUES


The equivalence techniques, primarily NPV, are valid methods to choose the correct alternative.
However, since service producing investments deal primarily with costs, NPV is replaced with Net
Present Cost (NPC) which is the absolute value of the NPV. When the evaluator calculates NPC,
the simplest approach is to change the signs of all of the projects cash flows as will be demonstrated
in Example 4.1. The alternative with the lowest NPC would be the best economic choice. Similarly,
Net Annual Value (NAV) is replaced with Net Annual Cost (NAC).
50 4. SERVICE PRODUCING INVESTMENTS
Example 4.1
Two alternatives are being considered which provide the same service and which have the same
useful life of five years. Alternative A has an initial capital investment of $12,000, annual operating
costs of $3,500, and a salvage value of $5,000. Alternative B has an initial capital investment of
$20,000, annual operating costs of $1,500, and a salvage value of $10,000. If the companys MARR
is 15%, which alternative would be the best economic choice? Use NPC and NAC analysis.
Alternative A:
0 1 2 3 4 5

-12000 -3500 -3500 -3500 -3500 -3500


L = 5000
Alternative B:
0 1 2 3 4 5

-20000 -1500 -1500 -1500 -1500 -1500


L = 10000
NPC:
N P CA = 12000 + 3500(P /A)15,5 5000(P /F )15,5 = $21, 250
N P CB = 20000 + 1500(P /A)15,5 10000(P /F )15,5 = $20, 060

NAC:
N ACA = 12000(A/P )15,5 + 3500 5000(A/F )15,5 = $6, 340
N ACB = 20000(A/P )15,5 + 1500 10000(A/F )15,5 = $5, 980

Both NPC and NAC analysis indicate that Alternative B is the best economic choice since it
has the lowest cost under these conditions.

4.2.2 RATE OF RETURN METHODS


Rate of return methods need to be altered since there are generally no positive cash flows in a service
producing investment except, perhaps, a salvage value. Under that scenario, the definitions of IRR
and ERR dont make any sense and, in fact, generally do not result in positive values.
4.2. EQUAL LIFE ALTERNATIVES 51
When comparing two service producing investment alternatives, an incremental project rate
of return (either IRR or ERR) is determined and compared to the MARR. The cash flows for the
incremental project are found by taking the cash flows from the investment with the larger initial
capital cost and subtracting the cash flows from the investment with the lower initial capital cost. It
should be fairly obvious that if the alternative with the larger initial capital cost doesnt have lower
annual costs than the alternative with the lower initial capital cost, it will never be the economic
choice. Therefore, one would expect the incremental project cash flow diagram to be represented by
a negative initial investment, followed by positive cash flows that represent the savings generated by
choosing the alternative with the larger initial capital cost over the alternative with the lower initial
capital cost. Thus, another name for this incremental project is the savings project.
The rate of return (either IRR or ERR) can now be calculated for the savings project. If the
rate of return is larger than the MARR, this indicates that the savings project is an acceptable project
which thereby insinuates that the correct economic choice would be the alternative with the larger
initial capital cost. The net savings that occur by choosing the alternative with the larger initial
capital cost more than offset its additional initial capital cost. If the IRR or ERR is less than the
MARR, the savings project is not an acceptable project and, therefore, the alternative with the lower
initial capital cost will be the economic choice.
If there are more than two alternatives, all of the alternatives should first be listed in descending
order of initial capital cost and the various pairings of alternatives would be evaluated using one of
the techniques above. For example, if there were three alternatives (A, B, C) in order of initial capital
costs (with A having the highest and C having the lowest), one would first compare A to B. If A is
the better choice, one would then compare A to C to determine the best overall choice. However, if
B were the better choice, the next comparison would be B to C to determine the best overall choice.
Example 4.2
Compare Alternatives A and B given in Example 4.1 and determine the best economic choice
using IRR and ERR techniques. Recall that the MARR is 15%.
Since Alternative B has the highest initial capital cost, the savings project would be created
by subtracting the cash flows of Alternative A from those of Alternative B:
Savings Project, B-A:
0 1 2 3 4 5

-8000 2000 2000 2000 2000 2000


L = 5000
The N P V of this project is given by:
N P VBA = 8000 + 2000(P /A)i,5 + 5000(P /F )i,5
52 4. SERVICE PRODUCING INVESTMENTS

Interest Rate, % NP V
15 1190.1
20 -9.4

Interpolation yields an I RR = 20%. Since I RR > MARR, B is the best economic choice.

ERR : PC = | 8000| = 8000


FI = 2000(F /A)15,5 + 5000 = 18485
ERR = (18485/8000)1/5 1 = 0.182 = 18.2%

Again, the ERR would indicate that Alternative B is the best economic choice.

Example 4.3
Given the 3 alternatives below that provide the same service over a 4 year period, develop
an Excel spreadsheet that uses IRR analysis to determine which alternative is the best economic
choice. MARR is 10%.
Alternative A:

0 1 2 3 4

-1000 -300 -350 -400 -450


L = 200

Alternative B:

0 1 2 3 4

-800 -320 -380 -440 -500


L = 100
4.2. EQUAL LIFE ALTERNATIVES 53
Alternative C:
0 1 2 3 4

-700 -340 -410 -480 -550


L = 50

Spreadsheet and Results:

Incremental IRR calculations for Example 4.3:


A B C D E F G
1
2
3
4
5
6
7
8
9
10
11
12

The N P V and I RR functions are the same as presented in Chapter 3. The spreadsheet shows
the comparisons between all three projects. Since the initial investment of Alternative A is greater
than the initial investment of Alternative B and the initial investment of Alternative B is greater
than the initial investment of Alternative C, the alternatives are already correctly ordered by size
of initial investment. NPC analysis shows that Alternative B has the lowest net present cost and,
therefore, should be the alternative of choice.
The analysis of the incremental I RR calculations would be completed as follows:

1. Compare the first two alternatives.

2. Since the I RR of Incremental Project A-B (5.7%) is less than the MARR (10%), Alternative B
is a better choice than Alternative A.
54 4. SERVICE PRODUCING INVESTMENTS
3. Now compare Alternative B with Alternative C.
4. Since the I RR of Incremental Project B-C (23.5%) is greater than the MARR (10%), Alter-
native B is a better choice than Alternative C.
Therefore, Alternative B is the best economic choice (same as determined from the NPC
method).
Note that since Alternative B was a better choice than Alternative A, one never utilizes the incremen-
tal IRR that is calculated for the Incremental Project A-C. However, it is a necessary portion of the
Excel spreadsheet since one does not know, ahead of time, which Alternatives will be eliminated
during the analysis of the results.

4.3 UNEQUAL LIFE ALTERNATIVES


The analysis of service producing investments that have alternatives which provide the same service
but have unequal project lives cannot be completed without modifications to the alternatives. A
common evaluation life for each alternative must be found before a proper economic decision can
be made. This is because the definition of two alternatives providing the same service includes the
assumption that they provide this service for the same length of time. For example, one cannot
compare an alternative to purchase a vehicle, keep it for 5 years, and then sell it for its salvage value
to a three-year lease option for the same vehicle. Both options are providing the service of a vehicle,
but the service is provided for different lengths of time.
There are, in general, two methods employed by evaluators to find common evaluation lives
in these situations. The first method requires the determination of a least common multiple of
service lives for the alternatives being considered. The second method involves the determination of
a common study period which will be either the life of the shortest or longest alternative. In both
methods, cost assumptions must be made that will impact the final analysis.

4.3.1 LEAST COMMON MULTIPLE METHOD


The least common multiple method of finding a common service life utilizes the same principles
that are involved in determining the common denominator when adding or subtracting fractions.
Consider the example of two alternatives having useful lives of 3 and 4 years. The least common
multiple in this case would be 12 since there is not a smaller number which is divisible by 3 and 4
without leaving partial years as a remainder. The alternative having a useful life of 3 years would be
repeated 4 times on a time line to reach the least common multiple of 12 years. The other alternative
would be repeated 3 times.
A couple of disadvantages of this method should immediately come to mind. First, costs do
not stay constant over time, so one would need to predict the future cost of each alternative. Cost
escalation will be discussed in Chapter 6, but even this approach requires a number of assumptions.
Secondly, one or more of the alternatives may be rendered obsolete by the development of new
4.3. UNEQUAL LIFE ALTERNATIVES 55
technology before the end of the time period that corresponds to the least common multiple is
reached.

4.3.2 COMMON STUDY PERIOD


The common study period method of finding a common service life utilizes either the life of the
shortest alternative or the life of the longest alternative as the common study period. To determine
which of these to use, the length of the common study period should be, if possible, the length of
time that the service is actually required.
If the life of the shortest alternative is used, the extra years of the longer life alternative are
neglected and a new salvage value is assigned at the end of the common study period. The new
salvage value will typically be larger than the original salvage value since it should reflect the value
of the extra years that are neglected.
If the life of the longest alternative is used, the shorter project needs to be extended via one
of two methods. The project can be extended by either estimating the cost involved to repair the
service to get additional years of service from it or by purchasing a new unit of service. Both of these
require some assumptions with regard to future cost.

Example 4.4
The cash flows shown below represent two alternatives which can provide the same service.
Assume that the MARR is 15%. Use both methods described above to determine which alternative
is the best economic choice. (Numbers are in $1,000.)
Alternative A:
0 1 2 3 9 10

-150 -3 -3 -3 -3 -3
L = 10

Alternative B:
0 1 2 3 4 5

-50 -18 -18 -18 -18 -18


L=8
56 4. SERVICE PRODUCING INVESTMENTS
Least Common Multiple Technique: The least common multiple of 5 and 10 is 10. Therefore,
one needs to extend Alternative B from 5 to 10 years. It will be assumed that there is no escalation
in the costs for Alternative B for the second 5 year period. In Chapter 6, we will consider this same
problem with cost escalation. Therefore, Alternative B extended to 10 years would be:
Alternative B (extended to 10 years):
0 1 2 3 5 6 9 10

-50 -18 -18 -18 -18


L=8
-50 -18 -18 -18
L=8

N P C Analysis:

N P CA = 150 + 3(P /A)15,10 10(P /F )15,10 = $162.6


N P Cextended B = 50 + 18(P /A)15,10 + 42(P /F )15,5 8(P /F )15,10 = $159.2

N P C analysis indicates that Alternative B is the best economic choice under the assumptions
that were made (e.g., no increase in costs for the second 5 years). If costs increase or if technology
makes Alternative B obsolete, then this analysis will be inaccurate and one may need to consider
other non-economic factors in making this decision.
Common Study Period Technique: Lets shorten Alternate A to 5 years by neglecting the costs
in the final 5 years and by increasing the salvage value that could be received at year 5 to $80,000.
Alternative A (shortened to 5 years):
0 1 2 3 4 5

-150 -3 -3 -3 -3 -3
L = 80
N P C Analysis:

N P Cshortened A = 150 + 3(P /A)15,5 80(P /F )15,5 = $120.3


NP CB = 50 + 18(P /A)15,5 8(P /F )15,5 = $106.4
4.4. PROBLEMS 57
N P C analysis indicates that Alternative B is the best economic choice under these set of
assumptions (e.g., the new estimated salvage value for Alternative A and the assumption that one
can actually sell Alternative A for salvage at the end of 5 years).

4.4 PROBLEMS

4.1. A mining company is in need of four trucks. Suppliers will offer the options of purchasing
or leasing the trucks. The purchase price is $200,000. Maintenance, insurance, and general
operating costs (payable at the end of each year) will be $30,000 in year 1, $40,000 in year 2,
and $50,000 in year 3 with an expected salvage value of $70,000 at the end of year 3. The
lease price is $80,000 per year for the 3 years (payable at the beginning of each year). The
lease covers maintenance costs, but insurance and general operating costs will be $25,000
per year (payable at the end of each year). If the companys MARR is 20%, determine the
best economic choice.

4.2. A natural gas producing company is considering two engine systems for use in driving a
small compressor. System A can be purchased for $120,000 and is expected to have a life of 4
years. Annual diesel fuel consumption is estimated to be 60 gallons per day of use. System B
can be purchased for $150,000 and is expected to have a life of 4 years. Annual propane fuel
consumption is estimated to be 40 gallons per day of use. Both engines have salvage values
equal to 15% of initial cost and both will accomplish the needed requirements. Estimates
of fuel costs for each system and expected days of use each year are as follows:

Assume that MARR is 8% and that all other costs besides fuel will be the same for both
systems. Which system is the best economic choice?
58 4. SERVICE PRODUCING INVESTMENTS
4.3. Use ERR analysis to determine which alternative would be the best economical choice.
Verify your decision with NPC analysis. Assume the MARR equals 10%.
Alternative A:
0 1 2 3 4

-500 -25 -25 -25 -25


L = 100

Alternative B:

0 1 2 3 4

-300 -50 -50 -50 -50


L = 25

Alternative C:
0 1 2 3 4

-250 -75 -60 -45 -30


L = 10

Alternative D:

0 1 2 3 4

-450 -35 -35 -35 -35


L = 100
4.4. PROBLEMS 59
4.4. Consider the two service producing projects described below. They will provide the same
service but they do not have equal lives. Use NPC, IRR, and ERR analyses to determine
which alternative should be chosen. For the least common multiple method, assume no
increases in future costs for either project. For the common study period method, assume
that the salvage value for Alternative B will increase to $4,000 at the end of year 3. The
MARR is 10%.
Alternative A:

0 1 2 3

-15000 -1000 -1000 -1000


L=0

Alternative B:

0 1 2 3 4

-10000 -3000 -3000 -3000 -3000


L = 2000

4.5. Use Excel to solve Problem 4.1 for values of MARR of 10%, 15%, 20%, and 25%.
4.6. Use Excel to solve Problem 4.2 for values of MARR of 5%, 8%, and 12%.
4.7. Use Excel to determine what initial cost of Alternative A in Problem 4.2 would make the
two systems equal at an MARR of 8%.
4.8. Use Excel to solve Problem 4.3.
4.9. Use Excel to solve Problem 4.4.
61

CHAPTER 5

Income Producing Investments


5.1 INTRODUCTION
In the previous chapter, investments were considered that only provided a service of some kind for the
investor. In this chapter, investments that generate income (or profit) are discussed. The evaluation
techniques to be used will be identical to those introduced in Chapter 3. However, one additional
concept needs to be introduced when an investor is faced with making decisions between multiple
alternatives. This concept is the fact that income producing investment situations can be classified
as being either mutually exclusive, independent, or contingent as defined in later sections of this
chapter.

5.2 INVESTMENT IN A SINGLE PROJECT


If an investor is being offered the opportunity to invest in a single project (that is, without considering
any other alternatives other than the do nothing alternative), he needs to consider the following
two economic issues:
Does he have enough money to invest in this project?

Is the project profitable enough?


If one does not consider the option of the investor borrowing money from a lending institution, the
answer to the first question should be a clear yes or no. If the answer is no, then the investor
cannot invest in the project. Chapter 7 will cover financial leverage which will allow for the borrowing
of money.
If the answer to the first question is yes, then project profitability needs to be considered in
order to answer the second question. Utilizing the analysis techniques presented in Chapter 3, this
would mean one of the following:
The N P V of the project, calculated at the investors MARR, is greater than zero. (Similarly,
N AV or N F V would be greater than zero.)

The I RR of the project is greater than the investors MARR.

The ERR of the project is greater than the investors MARR.


Of these three options, the authors strongly suggest the NPV method. This will become clearer as
this chapter proceeds.
62 5. INCOME PRODUCING INVESTMENTS
Example 5.1
An investor with MARR of 15% has been presented with the opportunity to invest in the
following income producing project. Assume that he has $20,000 to invest. Should he invest in this
project based on economic considerations?

0 1 2 3 9 10

-20000 7500 7500 7500 7500 7500


L = 10000

Using the NPV, IRR, and ERR techniques described in Chapter 3:

NP V : N P V = 20000 + 7500(P /A)15,10 + 10000(P /F )15,10 = $20, 115


I RR : N P V = 20000 + 7500(P /A)I RR,10 + 10000(P /F )I RR,10 = 0
Trial and error solution yields I RR = 36.7%
ERR : PC = | 20000| = $20, 000
FI = 7500(F /A)15,10 + 10000 = $162, 300
ERR = (162300/20000)1/10 1 = 0.233 = 23.3%

Since N P V > 0, I RR > MARR, and ERR > MARR, this project would be acceptable to
the investor.

5.3 MUTUALLY EXCLUSIVE ALTERNATIVES


When considering two or more alternatives in an economic analysis situation in which only one
alternative may be chosen, the alternatives are said to be mutually exclusive. Examples of mutually
exclusive alternatives would include the choice between two or more ways to develop a physical
property location (for example, build a gas station or a laundromat, but not both) or the choice
between two or more projects when faced with limited investment capital.
To evaluate choices in mutually exclusive situations, it is necessary to first determine if each
alternative is economically acceptable using the same questions as listed above. Any alternatives that
are not acceptable will be discarded. The remaining alternatives can then be ranked by a couple of
methods and the project at the top of the ranking is the best economic choice.

5.3.1 EQUIVALENCE TECHNIQUES


Equivalence techniques are those that use NPV, NAV, or NFV calculations. As explained earlier,
for a given project, if one of these values is greater than zero then the others will be also. Recall that
5.3. MUTUALLY EXCLUSIVE ALTERNATIVES 63
values greater than zero indicate that the alternative is an acceptable one. Obviously, if the value
is zero, the project earns exactly the MARR. Thus, the evaluation approach, using NP V as the
calculation choice, is as follows:
1. Calculate the N P V for each alternative.
2. Eliminate any alternative with NP V < 0.
3. If all alternatives have N P V < 0, then the investors decision should be the do nothing
alternative.
4. If one or more alternatives have NP V 0, the alternative with the largest positive NP V is
the best economic choice.

Example 5.2
In addition to the alternative given in Example 5.1, consider the situation where an investor
with an MARR of 15% has the choice between that alternative and the two additional ones given
below. Assume that the investor has $80,000 to invest. Also assume that the three alternatives are
mutually exclusive projects. This may occur because they represent alternatives in which only one
can actually be built or may occur because the investor only has $80,000 to invest so he only has
enough capital to invest in one.
Lets call the project in Example 5.1 Alternative A. Thus, new alternatives are Alternative B
and Alternative C.
Alternative B:
0 1 2 3 9 10

-80000 20000 20000 20000 20000 20000


L = 25000

Alternative C:

0 1 2 3 9 10

-70000 17500 17500 17500 17500 17500


L = 21875
64 5. INCOME PRODUCING INVESTMENTS
Using the NPV technique described in Chapter 3:

N P V : N P VB = 80000 + 20000(P /A)15,10 + 25000(P /F )15,10 = $26, 560

N P VC = 70000 + 17500(P /A)15,10 + 21875(P /F )15,10 = $23, 240

Since N P VA , N P VB , and NP VC are all greater than zero, all three alternatives would be
acceptable to the investor. However, since these are mutually exclusive alternatives, Alternative B is
the overall best economic choice because its NP V is the largest.
One might think that the evaluator should directly compare any two projects (such as A
and B in the previous example) by using incremental NPV analysis. The following calculations will
demonstrate that this approach is not necessary because the NPV of an incremental project such as
B-A is governed by the following relationship:

NP V BA = NP V B NP V A

From Example 5.1, N P V A = $20, 115 and from Example 5.2, NP V B = $26, 560. Using the
relationship above, N P V BA should be $6,445. The following cash flow diagram represents the
incremental project B-A:
Alternative B-A:
0 1 2 3 9 10

-60000 12500 12500 12500 12500 12500


L = 15000

NPV: N P V BA = 60000 + 12500 (P A)15,10 + 15000 (P F )15,10 = $6, 445
Note that the NPV of the incremental project, B-A, is, indeed, numerically equal to the
difference between the NPV values of alternative B and A (B minus A).

5.3.2 RATE OF RETURN TECHNIQUES


One can use both the internal rate of return (IRR) and external rate of return (ERR) methods to
find the best alternative from a list of mutually exclusive alternatives. However, unlike N P V , it will
be shown that the alternative with the highest I RR or ERR is not necessarily the best economic
choice. One must be very careful not to simply rank the projects by I RR or ERR.
The process to determine the best alternative using I RR or ERR is as follows:

1. Calculate the I RR or ERR for each alternative.


5.3. MUTUALLY EXCLUSIVE ALTERNATIVES 65
2. Eliminate any alternative with I RR or ERR < MARR.

3. If all alternatives have I RR or ERR < MARR, then the investors decision should be the do
nothing alternative.

4. If one or more alternatives have I RR or ERR MARR, then those alternatives should be
rank ordered from the one with the highest initial investment to the one with the lowest initial
investment.

5. A comparison is made between the alternatives with the two largest initial investments. Create
an incremental project cash flow diagram by subtracting the cash flows of the lower initial
investment from those of the higher initial investment.

6. Calculate the I RR or ERR of the incremental project. If this I RR or ERR is MARR,


then the alternative with the larger initial investment is the better economic choice. Similarly,
if this I RR or ERR is < MARR, then the alternative with the lower initial investment is the
better economic choice. Keep the best alternative and discard the other one.

7. If additional alternatives are still available, return to step 5 and compare the alternative that
was kept from step 6 with the one with the next lower initial investment.

8. If no additional alternatives remain, the best economic choice is the alternative that was kept
from step 6.

Example 5.3
Consider the three alternatives A, B, and C introduced in the earlier example problems. Use
IRR and ERR analysis to determine the best economic choice. The MARR is 15%.
Alternative A:
0 1 2 3 9 10

-20000 7500 7500 7500 7500 7500


L = 10000

I RR : N P V = 0 = 20000 + 7500(P /A)I RR,10 + 10000(P /F )I RR,10

Trial and error solution yields I RR = 36.7%


66 5. INCOME PRODUCING INVESTMENTS

ERR : PC = | 20000| = $20, 000


FI = 7500(F /A)15,10 + 10000 = $162, 300
ERR = (162300/20000)1/10 1 = 0.233 = 23.3%

Alternative B:
0 1 2 3 9 10

-80000 20000 20000 20000 20000 20000


L = 25000

I RR : N P V = 0 = 80000 + 20000(P /A)I RR,10 + 25000(P /F )I RR,10

Trial and error solution yields I RR = 22.7%

ERR : PC = | 80000| = $80, 000


FI = 20000(F /A)15,10 + 25000 = $431, 100
ERR = (431100/80000)1/10 1 = 0.183 = 18.3%

Alternative C:
0 1 2 3 9 10

-70000 17500 17500 17500 17500 17500


L = 21875

I RR : N P V = 0 = 70000 + 17500(P /A)I RR,10 + 21875(P /F )I RR,10

Trial and error solution yields I RR = 22.7%


5.3. MUTUALLY EXCLUSIVE ALTERNATIVES 67

ERR : PC = | 70000| = $70, 000


FI = 17500(F /A)15,10 + 21875 = $377, 200
ERR = (377200/70000)1/10 1 = 0.183 = 18.3%

As one can see, all three alternatives have I RR and ERR MARR. Therefore, all three
alternatives are acceptable. Putting them in ranked order by initial investment yields:

Ini al
Alterna ve Investment I RR ERR
B $80,000 22.7% 18.3%
C $70,000 22.7% 18.3%
A $20,000 36.7% 23.3%

At this point, one cannot simply choose the alternative with the highest I RR or ERR as the
best overall economic choice.
First, compare Alternative B to Alternative C:
Alternative B-C:
0 1 2 3 9 10

-10000 2500 2500 2500 2500 2500


L = 3125

Using the techniques described in Chapter 3:

I RR : N P V = 0 = 10000 + 2500(P /A)I RR,10 + 3125(P /F )I RR,10

Trial and error solution yields I RR = 22.7%

ERR : PC = | 10000| = $10, 000


FI = 2500(F /A)15,10 + 3125 = $53, 900
ERR = (53900/10000)1/10 1 = 0.183 = 18.3%

Since both the I RR and ERR are greater than the MARR, this indicates that Alternative B
is better than Alternative C. Eliminate Alternative C from further consideration and compare
Alternative B to the next alternative.
68 5. INCOME PRODUCING INVESTMENTS
Comparing Alternative B to Alternative A:
Alternative B-A:
0 1 2 3 9 10

-60000 12500 12500 12500 12500 12500


L = 15000

Using the techniques described in Chapter 3:

I RR : N P V = 0 = 60000 + 12500(P /A)I RR,10 + 15000(P /F )I RR,10

Trial and error solution yields I RR = 17.6%

ERR : PC = | 60000| = $60, 000


FI = 12500(F /A)15,10 + 15000 = $268, 800
ERR = (268800/60000)1/10 1 = 0.162 = 16.2%

Since both the I RR and ERR are greater than the MARR, this indicates that Alternative B
is better than Alternative A. Since the list of mutually exclusive alternatives has been exhausted,
Alternative B is the best overall economic choice.
In summary, one cannot use the values of the I RR and ERR from individual alternatives to
determine the best economic choice. If one were to do that, the results shown in the table for this
example would indicate that Alternative A is the best economic choice since it has the largest values
of I RR and ERR. However, both NP V and incremental rate of return analyses clearly show that
Alternative B is the best economic choice.

Example 5.4
To further reinforce the fact that one should not rank investments through the use of rate
of return, consider the following example. You are an investor with only $10 in your pocket. Two
friends offer you the following opportunities: Friend #1 needs $1 from you, but will give you $2 back
at the end of the day. Friend #2 needs all $10 of your money, but will give you $12 back at the end
of the day. Which opportunity is better for you from an economic point of view?
Examine this using NPV and incremental IRR approaches. Since the time frame is short (1
day), your daily MARR can be considered to be very close to 0%.
5.3. MUTUALLY EXCLUSIVE ALTERNATIVES 69
Friend #1 Alternative:

0 1

-1 2

NP V : N P V = 1 + 2(P /F )0,1 = $1

I RR: N P V = 1 + 2(P /F )I RR,1 = 0


Trial and error solution yields I RR = 100%

Friend #2 Alternative:

0 1

-10 12

NP V : N P V = 10 + 12(P /F )0,1 = $2

I RR: N P V = 10 + 12(P /F )I RR,1 = 0


Trial and error solution yields I RR = 20%

NPV analysis indicates that Friend#2 Alternative is the best economic choice, but IRR analysis
appears to indicate that Friend#1 Alternative is the best.
Friend #1 is offering a 100% rate of return and Friend #2 is offering a 20% rate of return. One
might think that Friend #1s offer is the best. However, at the end of the day, you only have $11 in
your pocket if you invest with Friend #1, but $12 if you invest with Friend #2. It is clear, therefore,
that you should invest with Friend #2 even though that friend is offering a lower rate of return. The
reason that the higher rate of return option is not the best option in this case is that the other $9
in your pocket is earning 0% rate of return. Combining 0% rate of return on $9 and 100% rate of
return on $1 ends up yielding a 10% overall rate of return if you invest with Friend #1.
70 5. INCOME PRODUCING INVESTMENTS
Incremental Alternative of Friend#2 Friend#1:
0 1

-9 10

I RR: N P V = 9 + 10(P /F )I RR,1 = 0


Trial and error solution yields I RR = 11.1%

Since the incremental I RR is greater than your MARR, this indicates that Friend #2 Alter-
native is, indeed, the best economic choice.
This example also introduces the notion of risk in an investment. Obviously, the mathematical
analysis has shown that loaning Friend #2 is the better investment. But, it requires you, the lender,
to give up all of your money. If there was a chance that neither friend could come through with
their repayment, then it might be better to keep the $9 in your pocket and invest in Friend #1. In
the event that neither friend could provide their repayment, at least you would still have $9 left of
your money. The concept of risk in investments will be discussed much more in Chapter 9.

5.3.3 USING EXCEL


As shown in the previous chapters, Excel can be used to choose the best alternative among a group
of mutually exclusive alternatives. Since Excel offers the ability to quickly calculate incremental
rates of return, there is no need to manually choose the pairs of alternatives to be evaluated. However,
this requires that one needs to evaluate each possible pair of alternatives (starting with all alterna-
tives ordered from highest to lowest initial investment) and then analyze the results table rather
than analyze specific pairs one at a time. For example, for Alternatives A, B, and C presented in
Examples 5.1 through 5.3, an Excel spreadsheet might look like what is shown in Table 5.1. Recall
that within the original alternatives, Alternative B had the largest initial investment, C had the next
highest initial investment, and A had the lowest initial investment. Thus, the pairs of interest are
B-C, B-A, and C-A.
One would use Table 5.1 as follows:

If using NPV analysis:

1. Note that the values of NP V given in cells B17, C17, and D17 are all positive. This
indicates that all three alternatives are acceptable.
2. Note that Cell C17 contains the largest value of NP V . This would indicate that Alter-
native B is the best economic choice.
71
5.3. MUTUALLY EXCLUSIVE ALTERNATIVES

Table 5.1: Excel solution of Examples 5.1 through 5.3


72 5. INCOME PRODUCING INVESTMENTS
If using IRR or ERR analysis (we will use IRR for this analysis):

1. Note that the values of I RR given in cells B18, C18, and D18 are all greater than the
MARR. This indicates that all three alternatives are acceptable.
2. Examine cell F18, which is the result of comparing the first pair of projects: B and C.
Since this value (22.7%) is larger than the MARR, this would indicate that Alternative B
is better than Alternative C. Alternative C is thus removed from further consideration
and the next viable pair would be B-A.
3. Examine cell G18, which is the result of comparing projects B and A. Since this value
(17.6%) is larger than the MARR, this would indicate that Alternative B is better than
Alternative A.
4. Since all necessary pairs have been examined, Alternative B is the best economic choice.
5. While column H is required to calculate the NP V , I RR, and ERR of the C-A pair, it
is not utilized in this example since Alternative C was removed from consideration after
its comparison against Alternative B. However, when developing this spreadsheet, one
does not know the result of the incremental analyses and, thus, all possible pairs must be
included. In addition, depending on the value of MARR, column H might be utilized
in other scenarios.

5.4 UNEQUAL LIFE ALTERNATIVES


Recall in Chapter 4 that if one was comparing service producing investments that have unequal
lives, one must choose one of two methods to force the projects to the same length of time. This is
because, to be comparable, the service must be offered for the same length of time.
In income producing investments, creating a common life is not required for NPV analysis.
However, for NAV, NFV, IRR, or ERR analysis, one must make the lives the same. Usually the life
of the longest alternative is used as the common evaluation life. If should be noted, however, that to
extend an income producing investment, one does not extend the positive cash flows. Instead, zero
cash flows are used to extend the life of the project. This is the case because one is assuming that the
cash flows from the income producing investment have already been estimated out to the full life of
the project and the project will be shut down at that time.
When conducting incremental rate of return analyses on unequal life alternatives, the evaluator
may find that the incremental project has multiple changes in sign of the yearly cash flows. This
was described in Chapter 3 as an acceleration project. Since the alternating signs may yield multiple
I RR values, either the modified IRR or ERR technique will need to be applied in the analysis.

Example 5.5
Use NPV, NAV, NFV, IRR, and ERR analyses to evaluate the unequal life alternatives below.
MARR is 12%.
5.4. UNEQUAL LIFE ALTERNATIVES 73
Alternative A:

0 1 2 3 4

-200 100 100 100 100

Alternative B:

0 1 2 3 4 5 6

-300 90 90 90 90 90 90

NPV analysis:

N P VA = 200 + 100(P /A)12,4 = $103.7

N P VB = 300 + 90(P /A)12,6 = $70.0

Alternative A is the best economic choice based on NPV analysis.

NAV, NFV, IRR, and ERR Analyses:


From the NPV analysis above, both alternatives are acceptable. Now, perform the incremental
analysis for NAV, NFV, IRR and ERR. Use six years as the common evaluation life by extending
Alternative A for two additional years with zero cash flows:
Alternative A extended to six years:

0 1 2 3 4 5 6

-200 100 100 100 100 0 0


74 5. INCOME PRODUCING INVESTMENTS
The incremental project is then:
Alternative B-A:
0 1 2 3 4 5 6

-100 -10 -10 -10 -10 90 90

N AV :

N AVBA = 100(A/P )12,6 10(P /A)12,4 (A/P )12,6 + 90(F /A)12,2 (A/F )12,6
= $ 8.20

Since the incremental NAV is less than zero, Alternative A is the best economic choice.

NF V :

N F VBA = 100(F /P )12,6 10(P /A)12,4 (F /P )12,6 + 90(F /A)12,2 = $ 66.5

Since the incremental NF V is less than zero, Alternative A is the best economic choice.

I RR:

N P VBA = 0 = 100 10(P /A)I RR,4 + 90(P /A)I RR,2 (P /F )I RR,4

Trial and error solution yields I RRBA = 5.4%


Since I RRBA is less than the MARR, Alternative A is the best economic choice.

ERR:

PCBA =| 100 10(P /A)12,4 | = $130.4


FIBA =90(F /A)12,2 = $190.8
ERRBA =(190.8/130.4)1/6 1 = 0.0655 = 6.55%

Since ERRBA is less than the MARR, Alternative A is the best economic choice.
In summary, each of the analysis techniques of NPV, NAV, NFV, incremental IRR, and
incremental ERR, indicate that Alternative A is the best economic choice. However, of these five
options, the authors strongly suggest the NPV method because it usually involves the least amount
of calculations and never requires the use of incremental analyses.
5.5. INDEPENDENT AND CONTINGENT INVESTMENTS 75
5.5 INDEPENDENT AND CONTINGENT INVESTMENTS
5.5.1 INDEPENDENT INVESTMENTS
Consider the case when an investor is faced with the choice of investing in one or more projects (rather
than just one from a list of mutually exclusive alternatives) depending upon how much investment
capital is available. These alternatives are said to be independent alternatives. The final decision of
which projects to invest in will be based on maximizing the NP V for the given investment dollars.
This could mean that several combinations of projects will need to be evaluated.

5.5.2 CONTINGENT INVESTMENTS


A contingent project is a project that is conditional on the choice of one or more other projects. For
example, in the discipline of petroleum engineering, consider that the regional office of a large oil
company must make a decision to invest in one of the following projects for a particular producing
field: a series of well workovers to increase production from the existing wells; a polymer flood to
capture more oil from the field; or drilling a number of new wells within the field to expedite the oil
recovery from the field. Unfortunately, prior to investing in a full-scale polymer flood, the regional
office must also invest in a pilot polymer flood that will, most likely, not be an economic success
by itself. However, if the pilot is technically successful, then the full-scale polymer flood could be
considered. Therefore, the full-scale polymer flood would be considered a contingent project because
it could not be implemented without also choosing to invest in the pilot flood.
Example 5.6
Projects A, B, and C are being considered as investments. List the combinations that will need
to be considered under each of the following scenarios:
(a) The projects are mutually exclusive
(b) The projects are independent
(c) Projects A and B are mutually exclusive, but project C is contingent on project B.
(a) If the projects are already mutually exclusive, then the investor can only invest in one project.
Therefore, the list of combinations would be:

Mutually Projects Possible


Exclusive Included Combinaons
Alternave A B C

1 0 0 0 None
2 1 0 0 A
3 0 1 0 B
4 0 0 1 C
76 5. INCOME PRODUCING INVESTMENTS
(b) If the projects are independent, then the investor can invest in any or all projects. Therefore, the
list of combinations would be:

Mutually Projects Possible


Exclusive Included Combinaons
Alternave A B C

1 0 0 0 None
2 1 0 0 A
3 0 1 0 B
4 0 0 1 C
5 1 1 0 A,B
6 1 0 1 A,C
7 0 1 1 B,C
8 1 1 1 A,B,C

(c) For the contingencies given, the list of combinations would be:

Mutually Projects Possible


Exclusive Included Combinaons
Alternave A B C

1 0 0 0 None
2 1 0 0 A
3 0 1 0 B
4 0 1 1 B,C

Of the list from (b) in this example, the following combinations are missing for the following
reasons:

1. C only Since C is contingent on project B, it cannot stand by itself

2. A,B Since A and B are mutually exclusive, they cannot be combined together

3. A,C Since C is contingent on project B and project B is not in this combination, A and C
cannot be combined

4. A,B,C Since A and B are mutually exclusive, they cannot be combined together
5.5. INDEPENDENT AND CONTINGENT INVESTMENTS 77
5.5.3 LIMITED INVESTMENT CAPITAL
When investment capital is unlimited and more than one project may be chosen, the analysis simply
requires the determination of which project(s) will earn more than the MARR. This can be done
with any of the analysis techniques discussed previously. Once the list of acceptable alternatives has
been generated, the economic choice is to invest in all of them.
When investment capital is limited, the analysis approach is a bit more complicated. The
basic approach is to determine all possible combinations of projects in which the total investment
is within the capital constraints and then to analyze each of the combinations as being mutually
exclusive. The combination with the highest NPV will represent the set of projects in which one
should invest.

Example 5.7
The cash flow diagrams of six projects, A through F, are shown below. For these projects,
determine what combination of projects is the best economic choice using NPV analysis and a
MARR of 10%. Projects B, C, and E are mutually exclusive. Projects A and D are mutually exclusive
but both are contingent on the acceptance of C. Project F is contingent on the acceptance of either
B or E. Consider two separate scenarios:
(a) Assume unlimited capital
(b) Assume limited capital of $30,000
0 1 2 3
A:

-5000 2500 2500 2500


0 1 2 3
B:

-30000 13500 13500 13500


0 1 2 3
C:

-15000 10000 10000 10000


78 5. INCOME PRODUCING INVESTMENTS
0 1 2 3
D:

-10000 6000 6000 6000


0 1 2 3
E:

-20000 10000 10000 10000


0 1 2 3
F:

-15000 11000 11000 11000

It can be shown that the individual projects have the following NP V s:

Project NP V
A $1,220
B $3,570
C $9,870
D $4,920
E $4,870
F $12,360
5.6. RANKING ALTERNATIVES 79
The table of mutually exclusive alternatives would be:

Mutuall y I n v es t m e nt
Exclusive Projects Possible Capital
Alterna ve A B C D E F Combina ons Needed NP V
1 0 0 0 0 0 0 None 0 0
2 0 1 0 0 0 0 B $30,000 $3,570
3 0 0 1 0 0 0 C $15,000 $9,870
4 0 0 0 0 1 0 E $20,000 $4,870
5 1 0 1 0 0 0 A,C $20,000 $11,090
6 0 0 1 1 0 0 C,D $25,000 $14,790
7 0 1 0 0 0 1 B,F $45,000 $14,930
8 0 0 0 0 1 1 E,F $35,000 $17,230

(a) There are eight mutually exclusive alternatives that result from the original six individual
projects and their interrelationships. When capital is unlimited, the correct economic choice is
the alternative that maximizes the NPV. In this case, alternative #8, which consists of investing
in projects E and F is the correct economic choice because it has the largest NPV.

(b) When capital is limited to $30,000, alternatives #7 and #8 are no longer considered. With
those removed, the correct economic choice will be alternative #6 since it will maximize the
NPV for those projects whose total investment is less than or equal to $30,000.

5.6 RANKING ALTERNATIVES


As mentioned earlier, one can always correctly rank alternatives according to their NP V values.
The combination of projects that is within any constraint of investment capital and has the highest
NP V will be the alternative of choice. However, one cannot correctly rank alternatives by I RR or
ERR values unless one utilizes incremental analyses. To illustrate this further, another example is
presented below.

Example 5.8
For the six projects listed in Example 5.7, use the IRR and ERR techniques to choose the
best mutually exclusive alternative.
It can be shown that mutually exclusive alternatives 1 through 8 have the following I RRs and
ERRs:
80 5. INCOME PRODUCING INVESTMENTS

Alternave I RR ERR
1 10.0% 10.0%
2 16.7% 14.2%
3 44.6% 30.2%
4 23.4% 18.3%
5 39.5% 27.4%
6 41.3% 28.4%
7 29.2% 21.7%
8 36.3% 25.7%

Direct ranking by I RR or ERR would indicate that Alternative 3 (project C alone) would
be the best economic choice. This is, of course, inconsistent with the previous NPV analysis. To
overcome this inconsistency, the evaluator must perform incremental IRR or incremental ERR
analyses. For example, the incremental IRR technique is shown below:

1. Order the alternatives from the largest investment to the smallest:

Alternave Capital Annual


Investment Cash Flow
7 $45,000 $24,500
8 $35,000 $21,000
2 $30,000 $13,500
6 $25,000 $16,000
4 $20,000 $10,000
5 $20,000 $12,500
3 $15,000 $10,000
1 $0 $0

2. Calculate the I RR of the incremental project 7-8:

0 1 2 3

-10000 3500 3500 3500

N P V78 = 0 = 10000 + 3500(P /A)I RR,3


Trial and error solution yields I RR78 = 2.5%
5.6. RANKING ALTERNATIVES 81
Since the incremental I RR is less than the MARR, Alternative #8 is better than Alternative #7.
Keep Alternative #8, discard Alternative #7, and compare Alternative #8 with the next one on
the list (#2).

3. Calculate the I RR of the incremental project 8-2:

0 1 2 3

-5000 7500 7500 7500


N P V82 = 0 = 5000 + 7500(P /A)I RR,3
Trial and error solution yields I RR78 = 139.0%

Since the incremental I RR is greater than the MARR, Alternative #8 is better than Alter-
native #2. Keep Alternative #8, discard Alternative #2, and compare Alternative #8 with the
next one on the list (#6).

4. Continue in this manner (comparing the best choice with the next one on the list) until one
has exhausted all of the alternatives. Alternative #8 will be the last remaining alternative and,
thus, will be the best economic choice. This result is now consistent with the one from NPV
analysis. A similar method for ERR will yield the same ultimate results of Alternative #8 being
the best economic choice.
Another way to complete the incremental IRR technique is to compare each alternative with
all other alternatives that have a lower capital investment and compute the incremental IRR. This
would result in a table of incremental IRRs as given below:

Alterna ve with
Row # higher capital Alterna ve with lower capital investment
investment
8 2 6 4 5 3 1
1 7 2.5* 52.8 6.1 33.8 20.7 21.2 29.2
2 8 139 23.4 52.8 32.1 29.9 36.3
3 2 -1 7 4 2.5 -4 2 . 4 - 15.9 1 6.6
4 6 106 48.7 36.3 41.3
5 4 23.4
6 5 23.4 3 9 .5
7 3 44.6
*IRR7-8
82 5. INCOME PRODUCING INVESTMENTS
The use of this table would be as follows (see the arrows):

1. Start in row 1. Compare incremental alternative 7-8. Since the incremental I RR (2.5%) is less
than the MARR (10%), choose Alternative #8. Drop to row 2 (that belongs to Alternative #8).

2. Compare incremental alternative 8-2. Since the incremental I RR (139%) is greater than the
MARR (10%), choose Alternative #8. Stay in row 2.

3. Compare incremental alternative 8-6. Since the incremental I RR (23.4%) is greater than the
MARR (10%), choose Alternative #8. Stay in row 2.

4. Compare incremental alternative 8-4. Since the incremental I RR (52.8%) is greater than the
MARR (10%), choose Alternative #8. Stay in row 2.

5. Compare incremental alternative 8-5. Since the incremental I RR (32.1%) is greater than the
MARR (10%), choose Alternative #8. Stay in row 2.

6. Compare incremental alternative 8-3. Since the incremental I RR (29.9%) is greater than the
MARR (10%), choose Alternative #8. Stay in row 2.

7. Compare incremental alternative 8-1. Since the incremental I RR (36.3%) is greater than the
MARR (10%), choose Alternative #8. Since there are no more alternatives to be compared
with Alternative #8, then Alternative #8 is the best economic choice.

For the case of limited capital ($30,000), omit Alternatives #7 and #8 from the table. Follow
the arrows to show that Alternative #6 is the best economic choice.

Alterna ve with
higher capital Alterna ve with lower capital investment
investment
6 4 5 3 1
2 -174 2.5 -42.4 -15.9 16.6
6 106 48.7 36.3 41.3
4 - - 23.4
5 23.4 39.5
3 44.6

In summary, once the incremental I RR table has been created, start with the alternative
with the largest initial investment and compare it to the alternative with the second largest initial
investment. If the incremental I RR is less than the MARR, drop to the row of the lower initial
investment and proceed to compare with the next alternative. If the incremental I RR is greater than
the MARR, stay on the same row and proceed to compare with the next alternative. Eventually, one
will exit from the table on the best economic choice.
5.7. PROBLEMS 83
5.7 PROBLEMS
5.1. Projects A and B below are mutually exclusive alternatives. The cash flow diagrams are
given. Determine which project is the best economic choice using NPV, IRR, and ERR
analyses. Use a value of 15% for MARR.
Project A:

0 1 2 3 9 10

-8000 5000 5000 5000 5000 5000


L = 8000

Project B:

0 1 2 3 9 10

-12000 6000 6000 6000 6000 6000


L = 12000

5.2. Two mutually exclusive, but unequal life, investment projects A and B are shown below.
Project A:

0 1 2 3 4 5

-100 40 40 40 40 140

Project B:

0 1 2

-120 60 180
84 5. INCOME PRODUCING INVESTMENTS
(a) Determine the best economic choice using NPV, IRR, and ERR analyses. Use an
MARR of 20%.
(b) What value of MARR would reverse the ranking of projects A and B found in part
(a)?

For Problems 5.3 and 5.4.

The following projects are utilized in Problems 5.3 and 5.4. Projects A and B are indepen-
dent. Projects C and D are mutually exclusive and both are dependent on the acceptance of
B. Project E is dependent on the acceptance of A.

B:

C:

D:

-
5.7. PROBLEMS 85

E:

5.3. For the projects described above, do the following:

(a) List all mutually exclusive alternatives.


(b) Which alternative should be chosen if the MARR equals 10% and one has unlimited
capital?
(c) Which alternative should be chosen if the MARR equals 10% and investment capital
is limited to $80?

5.4. For the projects described above, do the following:

(a) List all mutually exclusive alternatives.


(b) Develop the incremental I RR table.
(c) Use the table to determine which alternative should be chosen if the MARR equals
10% and one has unlimited capital.
(d) Use the table to determine which alternative should be chosen if the MARR equals
10% and investment capital is limited to $80.

5.5. Use NPV and ERR analyses to determine which of the following two mutually exclusive
projects is the best economic choice. Use MARR of 15%.
Project A:

0 1 2 3 4

-500 200 200 200 200


L = 500
86 5. INCOME PRODUCING INVESTMENTS
Project B:

0 1 2

-200 100 100


L = 200

5.6. Suppose you are considering two independent sets of two mutually exclusive projects each
plus a fifth project. The fifth project is contingent on two of the first four occurring. Make a
table that shows all of the mutually exclusive alternatives that are possible and the projects
that each alternative contains.
5.7. Projects A through E are being considered by an investor. They all are ten-year projects and
the MARR is 10%. Projects A and B are mutually exclusive. Projects C and D are mutually
exclusive and contingent on the acceptance of B. Project E is contingent on the acceptance
of A.

Project NP V ERR Capital Investment


A $5,000 $20,000
B 8% $15,000
C $20,000 $30,000
D $15,000 $22,000
E $10,000 $15,000

(a) List all of the possible mutually exclusive alternatives.


(b) Which alternative is the best economic choice with unlimited capital?
(c) Which alternative is the best economic choice with a capital constraint of $40,000?

5.8. Use Excel to solve Problem 5.1 for values of MARR of 5%, 15%, 25%, 35%, and 45%.
5.9. Use Excel to solve Problem 5.2 for values of MARR of 10%, 20%, and 30%.
5.10. Use Excel to solve Problem 5.3 for values of MARR of 10%, 20%, 25%, and 30%.
5.11. Use Excel to develop the incremental I RR table for Problem 5.4. Use the table to deter-
mine which alternative should be chosen if the MARR equals 10% and one has unlimited
capital.
5.12. Use Excel to solve Problem 5.5 for values of MARR of 5%, 15%, 25%, and 35%.
87

CHAPTER 6

Determination of Project Cash


Flow
6.1 INTRODUCTION
This chapter contains a discussion of escalation, depreciation, income taxes, and the subsequent
generation of cash flows when considering taxes. This chapter is not meant to be a detailed presenta-
tion on all of the ramifications of taxes. Most companies will use tax consultants and/or tax lawyers
instead of engineers to handle complicated tax questions. This chapter is meant to provide a basic
working knowledge of taxes so that the engineer can develop a stream of before- and after-tax cash
flows for a particular project.

6.2 ESCALATION AND INFLATION


When considering the effects of escalation on cash flows, it is necessary to define three types of
dollars with which evaluators work. The first is what is called today dollars. Today dollars simply refer
to the situation where all of the cash flows are calculated without any consideration for changes in
prices and costs as a function of time. This, of course, is not consistent with what actually occurs in
real life. A second type is escalated or actual dollars. When an evaluator attempts to estimate price
and cost changes and subsequently incorporates these changes into the cash flow calculations, then
the dollars are said to be escalated. The final type is constant dollars. When inflation is removed from
escalated dollars, then the resulting cash flows are said to be in constant dollars.
In order to more fully understand what is meant by these various types of dollars, the terms
inflation and escalation need to be defined.
Inflation refers to the general increase of prices with time due to an expanded money supply
with no hard assets to support the additional money. By definition, inflation affects prices of all
commodities by the same percentage amount. If the money supply decreases, there could be deflation
or the decrease in prices. There are as many causes of inflation as there are people who talk about it.
It is not the intent of the authors to discuss these causes. Using the Consumer Price Index (CPI)
that is published by the Bureau of Labor Statistics (http://www.bls.gov/data/), the average
inflation rate for 2000 to 2010 was 2.39% per year. The values of the CP I are shown in Figure 6.1
and Table 6.1 for various time periods. Published values are available back to 1913.
One can determine the average inflation rate for a given period of time by using the (F /P )i,n
formula. Consider the CP I values from two different years, n and m (with n > m). The CP I from
88 6. DETERMINATION OF PROJECT CASH FLOW

CPI Since 1960


250.0

200.0

150.0
CPI
100.0

50.0

0.0
1960 1970 1980 1990 2000 2010
Year

Figure 6.1: Consumer price index values for 1960-2010from http://www.bls.gov/data/.

year n will be considered as a future value and the CP I from year m will be considered a present
value. Thus:

CP In = CP Im (1 + f )nm which can be solved for the inflation rate, f , as


 [1/(nm)] 
f = CP Iyr n /CP Iyr m 1 100 (6.1)

For example, the average inflation rate between 1980 and 1990 was:

(130.7/82.4)[1/10] 1 100 = 4.72%

Similarly, the average inflation rate between 2009 and 2010 was:

(218.056/214.537)[1/1] 1 100 = 1.64%

Escalation, on the other hand, refers to the total change in the price of a specific commodity or
service over a period of time. Prices of individual commodities can change due to supply and demand,
as well as many other factors. While the inflation rate is a single numerical value for all commodities,
the escalation rate may be different for each commodity. For example, for 2000 to 2010, the price
of food increased an average of 2.69% per year (similar to inflation), the price of unleaded gasoline
increased an average of 6.32% per year, and the price of computers actually dropped an average of
6.2. ESCALATION AND INFLATION 89

Table 6.1: Consumer price index values for 1980-2010from http://www.bls.gov/


data/
Year CPI Year CPI Year CPI
1980 82.4 1990 130.7 2000 172.2
1981 90.9 1991 136.2 2001 177.1
1982 96.5 1992 140.3 2002 179.9
1983 99.6 1993 144.5 2003 184.0
1984 103.9 1994 148.2 2004 188.9
1985 107.6 1995 152.4 2005 195.3
1986 109.6 1996 156.9 2006 201.6
1987 113.6 1997 160.5 2007 207.342**
1988 118.3 1998 163.0 2008 215.303
1989 124.0 1999 166.6 2009 214.537
2010 218.056

** Star ng in 2007, the Bureau of Labor Sta s cs began publishing the CPI with three
decimal places instead of one

16.4% per year over that same time frame. It should be pointed out that escalation includes the effect
of inflation. Figure 6.2 shows the price of unleaded gasoline ($/U.S. gallon) from 1976 to 2010.
One should notice that the CPI curve in Figure 6.1 is relatively smooth, but the price of any
one commodity may fluctuate significantly over the same time frame as shown in Figure 6.2. This
is due to the fact that the CPI measures the average change in prices paid for a market basket of
goods and services. (U.S. Department of Labor)
The escalated or actual dollar type of analysis referred to above includes both the effect of
inflation and escalation. This type of analysis attempts to predict the future prices of those elements
that are part of the cash flow calculation. One can either let all income and expenses rise at the
average inflation rate or one can attempt to isolate each commodity and use various escalation rates
for each income or expense item.
The constant dollar analysis reflects the purchasing power of money over the life of the project
by factoring out the effect of inflation. For example, in constant dollars, the price of unleaded gasoline
has increased an average of 3.84% per year and the price of food only increased an average 0.29%
per year from 2000 to 2010. This calculation will be shown later in this chapter.
The today dollar analysis simply uses the current prices for the commodities that are part of
the cash flow calculation for the project and maintains them at this level throughout the life of the
project. Thus, there is no consideration of the effects of inflation and escalation.
The authors believe that one should use either an escalated dollar analysis or a constant dollar
analysis when attempting to determine the economic viability of a project. Today dollar analyses
90 6. DETERMINATION OF PROJECT CASH FLOW

Price of Unleaded Gasoline


3.500
3.000
2.500
2.000
$/US gal
1.500
1.000
0.500
0.000
1970 1980 1990 2000 2010 2020
Year

Figure 6.2: Price of unleaded gasoline from 1976 to 2010 (Government Accounting Office analysis of
Bureau of Labor Statistics (BLS) data).

should only be used for projects that have short enough lives that the costs of the commodities that
are part of the cash flow calculation do not change substantially.
Two rules should be kept clearly in mind when incorporating the effects of inflation and
escalation. The first is that the dollar types, constant or escalated, should never be mixed within
a single cash flow diagram. The second is that the MARR that is used in the evaluation must be
consistent with the type of dollars used. A rate of return calculated from a set of cash flows that are
based on constant dollars should be compared with an MARR that is also based on constant dollars.
Similarly, consistency between escalated dollar cash flows and an MARR that is based on escalated
dollars is necessary. It should be noted that bank interest rates and investment bond interest rates
are based on escalated dollars. Thus, if an investors MARR is derived from those types of interest
rates, it should also be considered to be an escalated dollar MARR.
The relationship between interest rates in escalated and constant dollars can be obtained by
comparing the corresponding P /F factors:

(P /F )i,n = (P /F )f,n (P /F )ii,n

where, i = escalated dollar interest rate, fraction per period


f = inflation rate, fraction per period
ii = constant dollar interest rate, fraction per period
6.2. ESCALATION AND INFLATION 91
If one substitutes the definition of (P /F ) and does some basic algebra, one can show:

ii = (1 + i)/(1 + f ) 1 (6.2)

This equation was utilized earlier to factor out the effect of inflation from escalation. For
example, for the period of 2000-2010, the inflation rate was 2.39% per year and the escalation rate
for unleaded gasoline was 6.32% per year. Using Equation 6.2, the constant dollar growth of this
commodity is:
ii = (1 + 0.0632)/(1 + 0.0239) 1 = 0.0384 = 3.84%
Note that just subtracting the inflation rate from the escalation rate (a difference of 3.93% in this
example) is not the correct way to factor out inflation from escalation.
The relationships between today dollars, escalated dollars, and constant dollars are shown
below:

Escalated $ price at the end of the year n = (Today $) (1 + i)n (6.3)


Constant $ price at the end of the year n = (Escalated $)/(1 + f )n (6.4)
Constant $ price at the end of the year n = (Today $) ((1 + i)/(1 + f ))n (6.5)

where, i = escalation rate, fraction per year


f = inflation rate, fraction per year

Example 6.1
Cash flow diagrams for projects A and B are shown below. Assume that the cash flows are
in escalated dollars and that the escalated dollar MARR is 15%. (a) Calculate the NP V of each
project as given and (b) calculate the NP V if one assumes a 5% inflation rate.

0 1 2 3
A:

-100 55 60.5 66.55


0 1 2 3
B:

-100 40 60 80
92 6. DETERMINATION OF PROJECT CASH FLOW

(a): N P VA = 100 + 55(P /F )15,1 + 60.5(P /F )15,2 + 66.55(P /F )15,3


= $37.33
N P VB = 100 + 40(P /F )15,1 + 60(P /F )15,2 + 80(P /F )15,3
= $32.75

(b):
For part (b), one needs to factor out the effect of inflation from the escalated cash flows. In
addition, the MARR will have to be adjusted to a constant dollar basis. The cash flows are adjusted
by using the (P /F ) factor at 5% for the corresponding number of years. For example, the 55 (year 1
cash flow for Project A) is multiplied by (P /F )5,1 to yield 52.38. When this is done, the cash flows
become:
0 1 2 3
A:

-100 52.38 54.88 57.49


0 1 2 3
B:

-100 38.10 54.42 69.11

The constant dollar MARR will be:

ii = (1 + i)/(1 + f ) 1 = (1 + 0.15)/(1 + 0.05) 1 = 0.0952 = 9.52%

The N P V s then become:

N P VA = 100 + 52.38(P /F )9.52,1 + 54.88(P /F )9.52,2 + 57.49(P /F )9.52,3


= $37.34
N P VB = 100 + 38.10(P /F )9.52,1 + 54.42(P /F )9.52,2 + 69.11(P /F )9.52,3
= $32.66

Note that within numerical round off, the NP V s are the same for either escalated or constant dollar
analysis. This will always be the case.
6.2. ESCALATION AND INFLATION 93
Example 6.2
A five-year life project has an initial capital expenditure of $250,000 and annual operating
costs beginning at the end of the year 1 of $100,000. At the end of the years 3, 4, and 5 the project
receives $500,000 as income. Calculate the I RR for the following cases:
(a) Assume the cash flows given are in escalated dollars and the escalated dollar MARR is 25%.
(b) Assume the cash flows given are in today dollars and that incomes are escalated at 6% and
costs are escalated at 10%.
(c) Assume inflation is 4% and rework part (b) in terms of constant dollars.
(a) (numbers are in $1000):
0 1 2 3 4 5

-250 -100 -100 400 400 400

N P V = 0 = 250 100(P /A)I RR,2 + 400(P /A)I RR,3 (P /F )I RR,2


Trial and error solution yields I RR = 34.3%. This value would be compared to the escalated
MARR of 25% to indicate that its an economically acceptable project.
(b) (numbers are in $1000):
Use Equation 6.3 to convert today dollars to escalated dollars:
94 6. DETERMINATION OF PROJECT CASH FLOW

NP V = 0
= 250 110(P /F )I RR,1 121(P /F )I RR,2 + 463(P /F )I RR,3
+ 485(P /F )I RR,4 + 508(P /F )I RR,5

Trial and error solution yields I RR = 39.9%. This value would be compared to the escalated
MARR of 25% to indicate that its an economically acceptable project.
(c) (numbers are in $1000):
Use Equation 6.4 to convert the today dollars to constant dollars:

Year Constant $ income Constant $ costs Constant $ CF


0 -250 (1.10)0 / (1.04)0 = -250 -250
1 -100 (1.10)1 / (1.04)1 = -106 -106
2 -100 (1.10)2 / (1.04)2 = -112 -112
3 500 (1.06)3 / (1.04)3 = 529 -100 (1.10)3 / (1.04)3 = -118 411
4 500 (1.06)4 / (1.04)4 = 540 -100 (1.10)4 / (1.04)4 = -125 415
5 500 (1.06)5 / (1.04)5 = 550 -100 (1.10)5 / (1.04)5 = -132 418

NP V = 0
= 250 106(P /F )I RR,1 112(P /F )I RR,2 + 411(P /F )I RR,3
+ 415(P /F )I RR,4 + 418(P /F )I RR,5

Trial and error solution yields I RR = 34.5%. This value would be compared to the constant
dollar MARR that is calculated according to Equation 6.1:

ii = (1 + 0.25)/(1 + 0.04) 1 = .202 = 20.2%

This value would still indicate that its an economically acceptable project.

6.3 DEPRECIATION
Certain capital assets of a company lose their value with use and/or with time. A building or an
item of equipment are examples of such assets. These assets have an initial value that is equal to
the original cost of the asset. However, they may lose value over time due to physical deterioration,
development of improved facilities by technological advances, or different demands of their use. The
reduction in value is called depreciation.
6.3. DEPRECIATION 95
One also needs to recognize that most governments (including the United States) do not
allow companies, for tax purposes, to deduct the entire cost of an asset against their income in
the year that the asset is purchased. Since the asset retains at least some portion of its value over
its life, companies must prorate the deduction of the original asset cost over the usable life of the
asset. Governments will specify particular techniques for this proration. These techniques are called
depreciation methods.
Therefore, there are two interpretations of a depreciation account for a capital asset. Under the
first, a company would set aside actual cash in a depreciation account in order to have the necessary
funds to replace the asset at the end of its useful life. Under the second, rather than setting aside actual
cash in the depreciation account, the company would simply establish depreciation accounts for tax
purposes. That is, the depreciation account represents the allowable annual deduction of the asset
against the projects income. The second interpretation represents reality. Thus, the depreciation
account that is maintained does not involve real dollars and depreciation expenses are known as
paper expenses in that they reduce the tax liability of the project but do not represent actual cash
expenditures. This chapter contains information on how to handle these paper expenses in the
calculation of after-tax cash flows for a project.
The most popular depreciation methods used in the United States are straight-line, sum-
of-the-years-digits, declining-balance, and the accelerated-cost-recovery-system. All four of these
methods will be discussed in this chapter.
In addition to the depreciation account, one also maintains a book value account that represents
the remaining value of the asset. Book value is simply the initial cost of the asset minus all accumulated
depreciation up to a specific point in time.
Depreciation calculations are based on the initial cost of the asset, P , any salvage value of the
asset at the end of its useful life, L, and the length of its useful life, N. The quantity P L represents
the total allowable depreciation of the asset if it is held for the entire time period N.

6.3.1 STRAIGHT-LINE DEPRECIATION (SL)


When using the straight-line depreciation method, the yearly amount of depreciation is given by
Equation 6.6:
Dn = (P L)/N (6.6)

where, Dn = depreciation amount in year n, $


n = year of depreciation
P = initial cost of the asset, $
L = salvage value of the asset at the end of its useful life, $
N = length of the assets useful life, years

It should be evident that the depreciation is constant with time when using the straight-line
method.
96 6. DETERMINATION OF PROJECT CASH FLOW
At the end of any given year, the book value of the asset is given by Equation 6.7:

Bn = P n(P L)/N (6.7)

where, Bn = book value of the asset at the end of year n, $

Excel has a built-in function called SLN that computes straight-line depreciation:

= SLN(Initial_Cost, Salvage, Life)

where, Initial_Cost = initial cost of the asset (P )


Salvage = salvage value of the asset (L)
Life = assets useful life (N)

6.3.2 DECLINING-BALANCE DEPRECIATION


Unlike straight-line depreciation, the annual depreciation amount determined using the declining-
balance method is not constant with time. The declining-balance method provides for a larger
depreciation deduction in the early years of an assets life than when using straight-line depreciation.
In this method, the depreciation amount is a fixed percentage of the remaining book value of the
asset.
The equations to calculate the annual depreciation amount and the book value at the end of
each year are given in Equations 6.8 and 6.9:

Dn = f (1 f )n1 P (6.8)
Bn = (1 f )n P (6.9)

where, f = a fixed percentage as a fraction

It should be noted that while the salvage value, L, is not utilized in the equations, one must
be careful that the total depreciation does not exceed the amount (P L).
Limits have been placed on the value of f that can be used in the declining-balance method.
The value of f cannot exceed 2/N. When the value of 2/N is used, the method is referred to as the
double-declining-balance (DDB) method.
Excel has a built-in function called DDB that computes double-declining balance depreci-
ation:

= DDB(Initial_Cost, Salvage, Life, Period, Factor)


6.3. DEPRECIATION 97

where, Initial_Cost = initial cost of the asset (P )


Salvage = salvage value of the asset (L)
Life = assets useful life (N )
Period = the period of interest
Factor = 2 (or omitted) for double-declining balance

6.3.3 SUM-OF-THE-YEARS-DIGITS (SYD) DEPRECIATION


This method, like the declining-balance method, provides for an accelerated depreciation deduction
in the early years of the useful life of an asset.
The equations to calculate the annual depreciation amount and the book value at the end of
each year are given in Equations 6.10 and 6.11:

Dn = [N (n 1)](P L)/S (6.10)


n
Bn = P Dj (6.11)
j =1

where, S = sum of the digits of the useful life of the asset = N(N + 1)/2
Excel has a built-in function called SYD that computes sum-of-the-years-digits deprecia-
tion:

= SYD(Initial_Cost, Salvage, Life, Period)

where, Initial_Cost = initial cost of the asset (P )


Salvage = salvage value of the asset (L)
Life = assets useful life (N )
Period = the period of interest

When calculating depreciation amounts for the determination of after-tax cash flows, it is
advantageous to use the most accelerated depreciation schedule possible. The sum-of-the-years-
digits and the declining-balance methods give larger depreciation amounts in the early years of an
asset. The straight-line method may, however, be more advantageous in later years.

Example 6.3
A device costs $5000 and has a salvage value of $800 after its useful life of 7 years. Calculate
the depreciation deduction that can be taken each year and the book value at the end of each year
for the useful life of the asset. Use the following depreciation methods:
98 6. DETERMINATION OF PROJECT CASH FLOW
(a) Straight-Line (SL)
(b) Double-Declining-Balance (DDB)
(c) Sum-of-the-Years-Digits (SYD)
(a) For Straight-Line:

Dn = (P L)/N = (5000 800)/7 = $600 which remains constant over the 7 years
Bn = P n(P L)/N = 5000 600 n

(b) For Double-Declining Balance:

f = 2/N = 2/7 = 0.28571


Dn = f (1 f )n1 P = 0.28571(0.71429)n1 5000 = 1428.55(0.71429)n1
Bn = (1 f )n P = (0.71429)n 5000 = 5000(0.71429)n

0 5000
1 1429 3571
2 1020 2551
3 729 1822
4 521 1301
5 372 929
6 129* 800
7 0** 800

*D6 would have been calculated as $266, but it was limited to $129 because the book value
cannot go below the salvage value.
**D7 would have been calculated as $190, but it was limited to $0 because the book value had
already reached the salvage value at the end of year 6.
6.3. DEPRECIATION 99
(c) For Sum-of-the-Years-Digits:

S = N (N + 1)/2 = (7)(8)/2 = 28
Dn = [N (n 1)](P L)/S = [7 (n 1)](5000 800)/28 = 150(8 n)
n n
Bn = P Dj = 5000 Dj
j =1 j =1

The depreciation and book values are shown in Figures 6.3 and 6.4 below to further demon-
strate the differences between these three methods.

Solution with Excel :

A B C D
1 P= 5000
2 L= 800
3 N= 7
4
5 Year SL DDB SYD
6 1 $600 $1,429 $1,050
7 2 $600 $1,020 $900
8 3 $600 $729 $750
9 4 $600 $521 $600
10 5 $600 $372 $450
11 6 $600 $130 $300
12 7 $600 $0 $150
100

A B C D
1 P= 5000
2 L= 800
3 N= 7
4
5 Ye a r SL DDB SYD
6 1 =SLN($B$1,$B$2,$B$3) =DDB($B$1,$B$2,$B$3,A6) =SYD($B$1,$B$2,$B$3,A6)
7 2 =SLN($B$1,$B$2,$B$3) =DDB($B$1,$B$2,$B$3,A7) =SYD($B$1,$B$2,$B$3,A7)
6. DETERMINATION OF PROJECT CASH FLOW

8 3 =SLN($B$1,$B$2,$B$3) =DDB($B$1,$B$2,$B$3,A8) =SYD($B$1,$B$2,$B$3,A8)


9 4 =SLN($B$1,$B$2,$B$3) =DDB($B$1,$B$2,$B$3,A9) =SYD($B$1,$B$2,$B$3,A9)
10 5 =SLN($B$1,$B$2,$B$3) =DDB($B$1,$B$2,$B$3,A10) =SYD($B$1,$B$2,$B$3,A10)
11 6 =SLN($B$1,$B$2,$B$3) =DDB($B$1,$B$2,$B$3,A11) =SYD($B$1,$B$2,$B$3,A11)
12 7 =SLN($B$1,$B$2,$B$3) =DDB($B$1,$B$2,$B$3,A12) =SYD($B$1,$B$2,$B$3,A12)
6.3. DEPRECIATION 101

Depreciaon Values
1600
1400
1200
1000
D(n) $ 800 SL
600 DDB
400 SYD
200
0
1 2 3 4 5 6 7
Year

Figure 6.3: Comparison of depreciation values for straight-line, double declining balance, and sum-of-
the-years-digits methods.

Book Values
5000
4500
4000
3500
3000
B(n) $ 2500 SL
2000 DDB
1500
1000 SYD
500
0
0 1 2 3 4 5 6 7
Year

Figure 6.4: Comparison of book values for straight-line, double declining balance, and sum-of-the-
years-digits methods.
102 6. DETERMINATION OF PROJECT CASH FLOW
6.3.4 MODIFIED ACCELERATED COST RECOVERY SYSTEM (MACRS)
In 1981, the United States government passed the Economic Recovery Tax Act which made sig-
nificant changes in depreciation calculations. The act was further modified in 1986 which led to
the Modified Accelerated Cost Recovery System (MACRS) for assets that were placed after 1980.
MACRS generally simplified the calculation of depreciation by (a) removing any reference to the
salvage value of the asset at the end of its useful life by assuming that L = 0 and (b) using various
combinations of the three previously presented depreciation methods to calculate annual deprecia-
tion values that are simply percentages of the original asset cost. As in the other methods, the assets
book value is the original cost minus all accumulated depreciation. That is:

Dn = P Depreciation Rate(depreciable life, n) (6.12)


n
Bn = P Dj (6.13)
j =1

To determine what depreciation rate to use, one must first determine the depreciable life of the
asset. The MACRS method created the following classifications: 3-year property, 5-year property,
7-year property, 10-year property, 15-year property, 20-year property, and 25-year property. IRS
publication 946 (http://www.irs.gov/pub/irs-pdf/p946.pdf) defines the types of assets that
fit in each classification. Table 6.2 shows a summary of this publication. One should note that any
property that doesnt specifically fit in another category is automatically classified as 7-year property.
Table 6.3 shows the depreciation rates that are used for various classifications, assuming a
half-year convention (most common assumption). A half-year convention simply recognizes that
assets are put into service at various times during any one year. Rather than beginning to depreciate
the asset on the actual day that it is put into service, the U.S. government allows a half year of
depreciation in the first year of use, a full year of depreciation from year two until year N and a half
year of depreciation in year N+1. Thus, depreciation for assets that fit in the 7-year depreciation
category, are actually spread over a total of 8 years.
6.3. DEPRECIATION 103

Table 6.2: Various classifications of depreciable propertyfrom http://www.irs.gov/


pub/irs-pdf/p946.pdf

Property
Classification Examples
3-year property Tractor units for over-the-road use
Qualified rent-to-own property
5-year property Automobiles, taxis, buses, and trucks
Computer and peripheral equipment
Office machinery
Certain geothermal, solar, and wind energy property
7-year property Office furniture and fixtures
Agricultural machinery and equipment
Any property that does not have a class life and has not been designated
by law as being in any other class
Any natural gas gathering line placed in service after April 11, 2005
10-year property Vessels, barges, tugs and similar water transportation equipment
Qualified small electric meter and qualified smart electric grid system
placed in service after Oct 3, 2008
15-year property Any municipal wastewater treatment plant
Any qualified restaurant property placed in service before Jan 1, 2012
Electric transmission property used in transmission at 69 or more kilovolts
of electricity placed in service after April 11, 2005
Any natural gas distribution line placed in service after April 11, 2005
20-year property Farm buildings
Municipal sewers
25-year property Water utility property that is not included as 20-year property
104 6. DETERMINATION OF PROJECT CASH FLOW

Table 6.3: Depreciation rates for various property livesfrom http://www.irs.gov/pub/


irs-pdf/p946.pdf
6.3. DEPRECIATION 105

Although Excel does not have a built-in function that can be used to directly compute
MACRS depreciation, one can use the VDB function if one recognizes that MACRS is defined as
DDB depreciation, using 1/2 year convention, and then switching to straight-line depreciation:
=VDB(Initial_Cost, Salvage, Life, Start_Period, End_Period, Factor,no_Switch)

where, Initial_Cost = initial cost of the asset (P )


Salvage = salvage value of the asset (L) set to zero for MACRS
Life = assets useful life (N )
Start_Period to End_Period = the period of interest (which can be fractional time periods).
For 1/2 year convention in year 1, use Start_Period = 0 and End_Period = 0.5.
For full year convention for years 2 through N , use Start_Period = (n 1.5) and
End_Period = (n 0.5).
For 1/2 year convention in year N + 1, use Start_Period = (N 0.5) and
End_Period =N .
Factor = 2 (or omitted for DDB)
no_Switch = FALSE (or omitted for automatic switching to straight-line)

For example, for 7-year property:


106 6. DETERMINATION OF PROJECT CASH FLOW
Example 6.4
Determine the yearly depreciation for the device described in Example 6.3 if it fits in the
7-year life category. Recall that P = $5000.

6.4 CASH FLOW COMPUTATION


As described in Chapter 2, cash flow is simply the net change (+ or -) in a companys or individuals
cash balance relative to a given project. That is, a positive project cash flow for a period would
indicate that the company had more cash due to that project at the end of that period than it did at
the beginning. A negative project cash flow would indicate just the opposite.
The following discussion lists the major considerations in determining cash flow for a project.
Cash flows can be calculated as before-tax or after-tax (where the tax is state and federal income tax).
It should be noted that the implications of a specific project on the companys overall tax situation
will ultimately be determined by the companys accountants and/or tax attorneys. Therefore, most
engineering economic analyses will be conducted on before-tax cash flows. However, sometimes it
is necessary or informational to evaluate after-tax cash flows. Therefore, both types are covered in
this discussion.

6.4.1 CAPITAL INVESTMENT


Capital investment is the cash that is expended by the company or the individual necessary to get
the project underway. That is, it is money used by the company or individual to purchase fixed assets
such as land, machinery, or buildings rather than money used for day-to-day operations. While
cash expenditures for fixed assets will generally occur over the length of a specific time period, it is
assumed that, for economic evaluation purposes, it all occurs at the beginning of that time period.
Thus, if it takes $500,000 over 6 months to construct a manufacturing facility, one would consider
all $500,000 to be spent at the beginning of year 1 (which, recall, is year 0 on the cash flow diagram).
Capital investment will include all costs associated with the fixed assets that are being purchased.
6.4. CASH FLOW COMPUTATION 107
For example, labor costs, materials, services, etc. that are part of the construction of a manufacturing
facility are considered capital investment.

6.4.2 GROSS REVENUE


Gross revenue is all revenue that is generated through the sale of a product or service. In most cases,
revenue for each product stream can be computed with Equation 6.14:

{Gross Revenue} = {#of items sold during a period} {price per item} (6.14)

It should be noted that, for economic evaluation purposes, the periods gross revenue will be assumed
to occur at the end of the particular time period in which it is generated.

6.4.3 OPERATING EXPENSES


Operating expenses are all cash outlays that are necessary to produce and sell the product or service.
These expenses may include, but are not limited to, items such as labor costs, building rent, utility
costs, raw materials, supplies, interest on loans, etc. Operating expenses are normally classified as
either fixed costs or variable costs. Fixed costs represent costs that are independent of the number of
units produced (for example building rent), whereas variable costs are proportional to the number
of units produced (for example raw materials). Equation 6.15 shows how to compute operating
expenses:

{Operating Expenses} ={Fixed Costs during a period} +


{#of items sold during a period} {variable cost per item} (6.15)

It should be noted that, for economic evaluation purposes, the periods operating expenses will be
assumed to all occur at the end of the particular time period in which it is spent. The assumption
that all capital investment will occur at the beginning of each year and that the income and operating
expenses will occur at the end of each year is known as end-of-year convention.

6.4.4 BEFORE-TAX PROFIT COMPUTATION


For the computation of before-tax profit, one only needs to consider gross revenues and operating
expenses:

{Before tax Profit} = {Gross Revenue} {Operating Expenses} (6.16)

6.4.5 BEFORE-TAX CASH FLOW COMPUTATION


For the computation of before-tax cash flows, one needs to have information on capital investment,
gross revenue, and operating expenses for each time period, n:

{Before tax cash flow} = {Gross Revenue} {Operating Expenses}


{Capital Investment} (6.17)
108 6. DETERMINATION OF PROJECT CASH FLOW
Example 6.5
Create the cash flow diagram for the following project. $300,000 is to be expended over 6
months to build a bicycle manufacturing facility. It is assumed that the facility will build 500 bicycles
the first year and 1000 bicycles in years two through five.The bicycles will be sold for $500 in the first
year with an estimated 4% escalation rate in years two through five. In the first year, fixed operating
costs will be $20,000 and variable operating costs will be $100 per bicycle. Assume an estimated 3%
escalation rate in years two through five for both operating costs.
The table below shows the detailed cash flow calculations for each year that results in the
following cash flow diagram ($ in thousands):

0 1 2 3 4 5

-300 180 396.4 413.5 431.3 449.8

Year Capital Gross Revenue Opera ng Expenses Before-tax cash


Investment ow
0 300,000 0 0 -300,000
1 0 500*500 = 250,000 20,000 + 500*100 = 70,000 180,000
2 0 1000*500*(1.04) = (20,000 + 1000*100)*(1.03) 396,400
520,000 = 123,600
3 0 1000*500*(1.04)2 = (20,000 + 1000*100)*(1.03)2 413,500
540,800 = 127,300
4 0 1000*500*(1.04)3 = (20,000 + 1000*100)*(1.03)3 431,300
562,400 = 131,100
5 0 1000*500*(1.04)4 = (20,000 + 1000*100)*(1.03)4 449,800
584,900 = 135,100

6.4.6 DEPRECIATION
As mentioned above, depreciation costs are paper expenses that result from the depreciation of a
capital item. That is, there is no actual cash expenditure for this category. The cost does, however,
reduce the companys income tax burden as will be shown. One can pick any of the methods given
above to calculate the depreciation expenses.
6.4. CASH FLOW COMPUTATION 109
6.4.7 TAXABLE INCOME
Taxable income is the income (or sometimes called gross profit) that is subject to taxation by the
United States government:

{Taxable Income} = {Gross Revenue} {Operating Expenses} {Depreciation} (6.18)

6.4.8 STATE AND FEDERAL INCOME TAX


As shown in Table 6.4, U.S. companies compute their U.S. federal income tax (FIT) as a percentage
of their taxable income. (United States Code: Title 26, Subtitle A, Chapter 1, Part II, 11) Even
though the FIT rate varies as the taxable income increases, it is common for engineering economic
analyses to use a flat tax rate of 35% on all taxable income. In addition, many states in the U.S. have a
state income tax of a few percent (0 to 12% with a U.S. average of 6.56%). For engineering economic
calculations, it is sufficiently accurate to add the state and federal income tax rates together to arrive
at an effective tax rate.

Table 6.4: United States corporate income tax (FIT) ratesfrom United States Code: Title 26,
Subtitle A, Chapter 1, Part II, 11.

Therefore,

{FIT} = {Taxable Income} {Tax Rate} (6.19)

In some circumstances, FIT can be allowed to be a negative value. That is, if the taxable
income is negative (a loss), multiplying any tax rate by that taxable income would yield a negative
value for FIT. This would be the same as the government paying the project for losing money!!
However, this computation can be defensible if the project that is being evaluated is only one of
many for a large company. Since the company only pays taxes on its total taxable income (that is,
from all projects taken together), a loss from one project will reduce the taxes that would be paid by
a profitable project. Thus, the project that generates a negative taxable income does indeed yield a
negative tax. Allowing negative FIT values is known as a corporate analysis.
If the project is a stand alone project (that is, its profit or loss will not be combined with any
other project), then any negative values of FIT must be changed to zero for that year. However, the
loss in that year may be carried forward into the future to reduce taxes from a profitable year that
occurs later. This is an area where consultation with a corporate tax expert would be necessary.
110 6. DETERMINATION OF PROJECT CASH FLOW
6.4.9 NET PROFIT
Net Profit is computed as the taxable income minus the income tax:

{Net Profit} = {Taxable Income} {FIT}


= {Taxable Income} (1 Tax Rate) (6.20)

6.4.10 CASH FLOW


The values defined above can now be combined in order to compute the cash flow (or net cash flow)
for a particular period:

{Cash Flow} = {Net Profit} + {Depreciation} {Capital Investment} (6.21)

As mentioned before, since depreciation is only a paper expense (that is, no actual cash
payment is made for depreciation), it must be added back into the cash flow calculation. Depreciations
only effect, therefore, is to reduce the income tax that is paid.
Any capital investment (cash spent on depreciable assets) made during the particular period
is subtracted after all other cash flow considerations are taken into account.

Example 6.6
Determine the after tax cash flows for the ten years of the following projects life:

Initial capital investment: $1,000,000

Use 7-year MACRS depreciation

Total tax rate of 40%

Corporate tax analysis

Sales Schedule:

Year # of units sold Price per unit


1 5,000 $100
2 5,000 $110
3 7,000 $120
4 7,000 $120
5 10,000 $140
6-10 10,000 $140

Fixed Costs: $200,000 per year

Variable Costs: $30 per unit


6.4. CASH FLOW COMPUTATION 111
Solution:
Year 0

For evaluation purposes, assume that the initial capital investment occurs at the beginning of
year 1 (which, by definition, is year 0).
CF0 = 1, 000, 000

Year 1

Gross Revenue = 5,000 * 100 = $500,000


Operating Costs = 200,000 + 5000 * 30 = $350,000
Depreciation = 0.143 * 1,000,000 = $143,000
Taxable Income = 500,000 - 350,000 - 143,000 = $7,000
FIT = 0.40 * 7,000 = $2,800
CF 1 = 7,000 - 2,800 + 143,000 = $147,200
The remaining nine years are calculated in a similar manner and are shown in the following
cash flow table:

Gross Opera ng Taxable Capital


Year Revenue Costs Deprecia on Income FIT Investment Cash Flow
0 1,000,000 -1,000,000
1 500,000 350,000 143,000 7,000 2,800 147,200
2 550,000 350,000 245,000 -45,000 -18,000 218,000
3 840,000 410,000 175,000 255,000 102,000 327,000
4 840,000 410,000 125,000 305,000 122,000 308,000
5 1,400,000 500,000 89,000 811,000 324,000 576,000
6 1,400,000 500,000 89,000 811,000 324,000 576,000
7 1,400,000 500,000 89,000 811,000 324,000 576,000
8 1,400,000 500,000 45,000 855,000 342,000 558,000
9 1,400,000 500,000 0 900,000 360,000 540,000
10 1,400,000 500,000 0 900,000 360,000 540,000

At a value of MARR of 20%, the NP V of this project can be shown to be $518,000 (after
tax).
One might wish to generate an Excel spreadsheet to allow additional analysis of this problem
if any or all of the given numerical values change. Such a spreadsheet is shown on the next page.
From the formulas it can be seen that key numerical values can be easily changed and the remainder
of the spreadsheet will change accordingly.
The formulas and/or values in each column are shown on the next pages.
112 6. DETERMINATION OF PROJECT CASH FLOW
6.4. CASH FLOW COMPUTATION 113
114 6. DETERMINATION OF PROJECT CASH FLOW
6.5. PROBLEMS 115
6.5 PROBLEMS

6.1. Using the CP I , compute the average inflation rate from 1992 to 2009.

6.2. Cash flow diagrams for projects A and B are shown below. Assume that the cash flows are
in escalated dollars and that the escalated dollar MARR is 10%.

(a) Calculate the N P V of each project as given.

(b) Calculate the N P V if one assumes a 5% inflation rate.

0 1 2 3

-80 40 45 50

0 1 2 3

-120 100 80 60

6.3. An eight-year life project has an initial capital expenditure of $450,000, annual income of
$300,000 beginning at the end of year 1, and annual operating costs of $80,000 beginning
at the end of year 1. Calculate the I RR for the following cases:

(a) Assume the cash flows given are in escalated dollars and the escalated dollar MARR
is 20%.

(b) Assume the cash flows given are in today dollars and that incomes are escalated at 7%
and costs are escalated at 6%.

(c) Assume inflation is 4% and rework part (b) in terms of constant dollars.
116 6. DETERMINATION OF PROJECT CASH FLOW
6.4. An investment related to developing a new product is estimated to have the following costs
and revenues in today dollars. Do not consider any tax issues.

0 1 2 3 4 5

Investment: 50,000 150,000


Income: 200,000 200,000 200,000 200,000
Oper Costs: 100,000 100,000 100,000 100,000
Salvage: 0

(a) Evaluate the projects escalated dollar I RR if both capital costs and operating costs
are estimated to escalate at 15% per year from time zero and income is estimated to
escalate at 10% per year from time zero.
(b) Evaluate the projects escalated dollar I RR assuming a washout of escalation of
income and operating costs with a 15% escalation of capital costs per year. Washout
means any operating cost escalation is offset by the same dollar escalation of revenue
(not the same percentage escalation) so that the before-tax profit remains uniform.
(c) Compute the constant dollar I RR of case (b) assuming that the rate of inflation will
be 10% per year.
6.5. Determine the breakeven escalated dollar selling price per unit, X, required in each of years
1 and 2 to achieve a 15% constant dollar project I RR, assuming a 12% per year inflation
rate. All values are given in today dollars.

0 1 2

Investment: 100,000
Income: 1000(X) 1000(X)
Oper Costs: 50,000 50,000
Income escalation = 10% per year from time zero when selling price is $X per unit.
Operating Cost escalation = 15% per year from time zero.
1,000 units are to be produced and sold per year.
6.5. PROBLEMS 117
6.6. Equipment has been purchased for $2,000,000 and put into service with an expected salvage
value at the end of 10 years of $200,000. Calculate the annual depreciation using:

(a) 10-year straight-line method

(b) 10-year double-declining balance method

(c) 10-year sum-of-the-years-digits method

(d) 10-year MACRS

6.7. Consider a mining and processing project for an oil tar sands project. From the data given
below, calculate the after-tax cash flows for a 30-year life of the project and the NP V for
an MARR of 15%.

Initial capital expenditures totaled $415.5 million and were distributed over four years
(10% in year 0, 30% in year 1, 40% in year 2, and 20% in year 3).

Beginning in year 4:

17.666 million tons of ore will be mined per year


Bitumen production rate will be 7.347 million barrels per year
Product yield will be 0.841 barrels of oil per barrel of bitumen
Product selling price will be $80 per barrel
Operating costs:
$10.47 per barrel of bitumen for plant and upgrading costs
$9.02 per ton of ore for mining costs
10-year straight-line depreciation
40% tax rate (state and federal)

6.8. The XYZ oil company owns several natural gas wells and is negotiating a 10-year contract
to sell the gas from these wells to another company. They are negotiating on the price of the
gas in the first year, in dollars per thousand cubic feet ($/MCF), including a 4% escalation
clause. XYZ expects the wells to produce 33,000 MCF the first year and to decline at the
rate of 15% every year thereafter. Operating costs are estimated to be $2/MCF and escalate
at 3% per year. XYZ has agreed to spend $500,000 now to lay pipelines from each well to
the second companys processing plant. What should the minimum price be the first year
for this to be acceptable to XYZ? Assume an end-of-year convention and an MARR of
15%.
118 6. DETERMINATION OF PROJECT CASH FLOW
6.9. An investment of $80,000 is projected to generate escalated dollar net revenues (income
minus costs) of $10,000 in year 1, $30,000 in year 2, and $40,000 in year 3 with a $40,000
salvage value at the end of year 3.

(a) Calculate the escalated dollar I RR for an escalated dollar MARR of 20%. Is this an
acceptable investment?
(b) Calculate the equivalent constant dollar I RR assuming that inflation will be 8% in
year 1, 10% in year 2, and 12% in year 3. Is this an acceptable investment?

6.10. The projected cost of the Alaskan oil pipeline was $900 million in 1969 dollars. The final
cost estimate was nearly $8.5 billion in 1977. What was the average yearly escalation rate
for the pipeline?

6.11. Bostons Big Dig is one of the most expensive highway projects in the U.S. The projects
original estimated cost was $2.6 billion in 1982 dollars. The costs in 2005 had risen to over
$14.6 billion.

(a) What is the value of the $14.6 billion in 1982 dollars?


(b) What was the average yearly escalation rate for the project?

6.12. Using Excel and the CP I values given in Table 6.1, calculate the annual inflation rate for
each year from 1980 to 2010.

6.13. Use Excel to solve Problem 6.2 for all 9 combinations of the following:

Values of MARR of 5%, 10%, and 15%


Inflation rates of 2%, 5%, and 7%

6.14. Use Excel to solve the following problem. An eight-year life project has an initial capital
expenditure of $450,000, annual income of $300,000 beginning at the end of year 1, and
annual operating costs of $80,000 beginning at the end of year 1. Calculate the I RR for
the following cases:

(a) Assume the cash flows given are in escalated dollars and the escalated dollar MARR
is 10%, 20%, and 30%.
(b) Assume the cash flows given are in today dollars and pairs of escalation rates are:
a. Incomes are escalated at 7% and costs are escalated at 6%
b. Incomes are escalated at 3% and costs are escalated at 5%
c. Incomes are escalated at 4% and costs are escalated at 4%
(c) Assume inflation is 4% and rework all portions of part (b) in terms of constant dollars.
6.5. PROBLEMS 119

6.15. Use Excel to solve Problem 6.6. Create a line graph that shows the values generated by
all four of the methods.
6.16. Use Excel to solve Problem 6.7. The spreadsheet should allow for the user to easily change
any of the numerical values given.
6.17. Use Excel to solve Problem 6.8. The spreadsheet should allow for the user to easily change
any of the numerical values given.
121

CHAPTER 7

Financial Leverage
7.1 INTRODUCTION
Earlier in this text, a brief description of the financial aspects involved in economic analyses was
presented. It was pointed out that one of the important financial aspects had to do with obtaining
the funds required to initiate the project. These funds are referred to as the investment capital. As
a source for this investment capital, a company could use its own internal funds (what is known as
equity funds), borrow funds from an external source (known as debt funds), or use a combination
of the two. The ratio of total borrowed funds to the total capital investment is called the financial
leverage factor. The ratio of borrowed funds to equity funds is called the debt to equity ratio. The
degree of financial leverage for any given project will affect the economic analysis of the project.

7.2 FINANCIAL LEVERAGE AND ASSOCIATED RISK


Under the correct conditions, financial leverage will allow an investor (company or individual) to
obtain a higher rate of return on its equity capital than it could achieve with no leverage. However,
there is often a good deal of added risk associated with leveraged projects. This additional risk is
due to the fact that when projects are financed with borrowed funds, those funds must be repaid to
the lender, independent of the ultimate success or failure of the project. If a leveraged project is only
marginally successful during any particular time period, the borrowed funds must be repaid to the
lender before any funds are used to pay a return on the equity portion of the investment.

7.3 ADJUSTMENT TO CASH FLOW EQUATIONS


Equations 6.14 and 6.15, as well as 6.18 through 6.21, allow the analyst to compute the after-tax cash
flows from a project. Some of these equations need to be modified for the case where the project is
leveraged. These modifications will account for the fact that (a) interest paid on the debt is a pre-tax
deduction while (b) the principal paid on the debt is not a pre-tax deduction.
Equation 6.18 is modified as follows:
{Taxable Income} ={Gross Revenue} {Operational Expenses}
{Depreciation} {Interest paid on debt} (7.1)
Equation 6.21 is modified as follows:
{Cash Flow} ={Net Profit} + {Depreciation}
{Equity Investment} {Principal paid on debt} (7.2)
122 7. FINANCIAL LEVERAGE
It should be noted that the investor is allowed to compute depreciation on the total value of
each asset in the project independent of the source of funds. Despite the source of funds, the investor
owns the full value of the depreciable assets that it procures for the project.

Example 7.1
A company is considering a one year investment which will cost $1000.The companys before-
tax MARR is 10%. The $1000 will purchase assets that will be fully depreciated in the one year
of operation. There are three possible economic conditions that the company needs to investigate.
Details of these conditions are shown below. In addition, the company will consider three different
leverage factors: 0.0, 0.4, and 0.7. Interest on any borrowed funds will be 10% over the one year of
operation. Use a 40% corporate tax rate and determine the after-tax I RR on the equity funds for
each combination of the three economic conditions and the three leverage factors. Note that, for
economic condition A, the before-tax IRR on total assets (in this case $1000) is less than the interest
rate that will be charged on the loan. For economic condition B, the before-tax I RR on total assets
is equal to the interest rate to be charged on the loan and, for economic condition C, the before-tax
I RR is greater than the loan interest rate.

Economic Conditions
A B C
Revenue Oper Costs $1050 $1100 $1200
Depreciation 1000 1000 1000
Taxable income without leverage 50 100 200
I RR on total assets before taxes 5% 10% 20%

Before-tax cash flow diagrams for each economic condition:

0 1 0 1 0 1
A: B: C:

-1000 1050 -1000 1100 -1000 1200

Table 7.1 shows the cash flows and the computed after-tax I RRs for the 9 different combi-
nations. Figure 7.1 shows the after-tax IRR on equity as a function of the leverage factor for the
three different economic conditions.
7.3. ADJUSTMENT TO CASH FLOW EQUATIONS 123

Table 7.1: Effect of leverage and economic conditions on the after-tax I RR on equity for
Example 7.1
124 7. FINANCIAL LEVERAGE

Economic Condition A
Economic Condition B
Economic Condition C

Figure 7.1: Effect of leverage factor for various economic conditions for Example 7.1.

From the results of Example 7.1, the following observations can be made:

1. Figure 7.1 shows that when the projects before-tax I RR on assets is less than the interest rate
charged on the loan (economic condition A), the after-tax I RR on equity decreases as the
leverage factor increases. This makes sense because the project must pay the lender a higher
rate of interest than it will be able to pay the owner in rate of return.

2. Figure 7.1 shows that when the projects before-tax I RR on assets is equal to the interest rate
charged on the loan (economic condition B), the after-tax I RR on equity is not affected as
the leverage factor increases. This makes sense because the project pays the lender the same
rate of interest as it will be able to pay the owner in rate of return.

3. Figure 7.1 shows that when the projects before-tax I RR on assets is greater than the interest
rate charged on the loan (economic condition C), the after-tax I RR on equity increases as the
leverage factor increases. This makes sense because the project pays the lender a lower rate of
interest than it is able to pay the owner in rate of return.

4. There is more risk to equity capital when projects are leveraged with borrowed money. If the
economic conditions are poorer than originally predicted (such as condition A occurring when
condition C was predicted when the decision to invest was made), the after-tax I RR on equity
will decrease.

5. If enough equity capital exists, companies should not borrow money to fund a project unless
the interest rate paid on the debt is less than the before-tax I RR on the projects total assets.

Leverage factors vary from company to company and even within a company from project
to project. In general, for most companies other than public utilities (who typically have very high
7.3. ADJUSTMENT TO CASH FLOW EQUATIONS 125
leverage factors of 0.6 or greater), leverage factors usually run from 0.3 to 0.5. A highly leveraged
project can do very well in a favorable economic climate, but may run into some hard times as
economic conditions go from good to bad. Many companies have used this principle to expand
rapidly during thriving business conditions.

Example 7.2
Consider the following five-year project with different methods of financing. A company has
the opportunity to invest in a five-year project that has an initial capital investment of $100,000.
The entire capital investment (total assets) will be depreciated over the five-year life of the project
using straight-line depreciation. Annual incomes and operating costs are expected to be $50,000
and $10,000, respectively. Interest on borrowed money will be 10% compounded annually. Calculate
the after-tax IRR on equity for the following cases and assuming a corporate tax rate of 40%. Use
an after-tax MARR of 12%.

(a) 100% equity.

(b) Leverage factor of 0.4. The principal payments will be constant for each of the five years and
the interest paid each year will be based on the outstanding debt balance.

(c) Leverage factor of 0.7. The principal payments will be constant for each of the five years and
the interest paid each year will be based on the outstanding debt balance.

(d) Leverage factor of 0.4. The principal and interest will be paid with a constant annual payment
as calculated according to: P &I payment = Debt (A/P )10%,5 .

(e) Leverage factor of 0.4. Interest payments are made each year but the principal is paid back
in one lump sum at the end of the project. This is known as yearly interest with a balloon
payment of the principal at the end.
126 7. FINANCIAL LEVERAGE
First, solve for the before-tax I RR on assets. This would be represented by a 0.0 leverage
factor and a 0% FIT rate.

Therefore, the before-tax I RR on assets for this project is 28.6%. Since the interest rate on
borrowed funds is less than this value, leveraging the project should increase the after-tax I RR on
equity.
7.3. ADJUSTMENT TO CASH FLOW EQUATIONS 127
(a) This solution will show the effect of the 40% FIT rate compared to the before-tax solution shown
previously.

The 40% FIT tax rate reduces the after-tax I RR on total assets to 18.0%.
128 7. FINANCIAL LEVERAGE
(b) This solution will show the effect of a leverage factor of 0.4.

0.4
40000

4000 3200 2400 1600 800

16000 16800 17600 18400 19200


6400 6720 7040 7360 7680
9600 10080 10560 11040 11520

8000 8000 8000 8000 8000


60000
-60000 21600 22080 22560 23040 23520

40000 32000 24000 16000 8000

25.1%
18691

The after-tax I RR on an equity investment of $60,000 has increased to 25.1%. This increase
is as expected. Also, the after-tax NP V has increased from a leverage factor of 0.0.
7.3. ADJUSTMENT TO CASH FLOW EQUATIONS 129
(c) This solution will show the effect of a leverage factor of 0.7.

Increasing the leverage factor to 0.7 further increases the after-tax I RR on equity and the
after-tax N P V .
130 7. FINANCIAL LEVERAGE
(d) This solution will show the effect of paying constant annual principal and interest payments.

Using a more conventional method to repay the debt, the after-tax I RR and after-tax NP V
both increase slightly from the first repayment method.
7.3. ADJUSTMENT TO CASH FLOW EQUATIONS 131
(e) This solution will show the effect of paying annual interest and then a balloon payment for the
principal.

** Modified I RR using an after-tax MARR of 12%.


The balloon repayment method further increases the after-tax I RR and after-tax NP V .

From the results of Example 7.2, the following observations can be made:

1. If the results from parts (a), (b), and (c) are compared, it is again found that, under these
economic conditions, when the amount of borrowed funds is increased, a higher rate of return
is obtained on the equity investment. It should be stressed that this higher rate of return is on
a smaller amount of equity dollars compared to financing the project with 100% equity funds.

2. It can also be seen that after-tax NP V increases as the leverage factor is increased. NPV
analysis would further emphasize that, under the economic conditions of the before-tax I RR
on assets being greater than the interest rate paid on the debt, the best option is to maximize
132 7. FINANCIAL LEVERAGE
the amount of leverage. By using maximum leverage on each project, a company can invest in
more projects and grow more rapidly.

3. Parts (b), (d), and (e) compare three different, but acceptable, methods of repaying the debt
portion of the investment. Since the interest on the borrowed money is less than the I RR on
assets, it is better to push the repayment of the principal as far forward in time as possible
in order to increase I RR on equity and NP V . The balloon payment technique provides the
highest I RR and N P V .

7.3.1 LEVERAGE AND MUTUALLY EXCLUSIVE PROJECTS


When applying leverage concepts to the evaluation of several projects to determine which one is
best, the leverage factor is an important variable. It has been shown in the example problems, that the
project I RR on equity is a function of the leverage factor. In order to compare projects, the degree
of leverage must be the same on all projects. Many companies have a policy that the comparison of
projects is done without considering any leverage for all of the projects. Once a project is chosen, then
various methods of financing, including different amounts of leverage and repayment techniques,
can be investigated as to their effect upon the project.

7.3.2 EXCEL SPREADSHEET


As shown on the next page, the project spreadsheet generated for Example 6.6 can be easily modified
to include the effect of leverage. The only assumptions in the spreadsheet are that the loan will be
paid with constant principal payments over the first five years and the interest paid each year will be
based on the outstanding debt balance. This could be modified for other repayment options.
7.3. ADJUSTMENT TO CASH FLOW EQUATIONS 133
134 7. FINANCIAL LEVERAGE
7.4 PROBLEMS
7.1. A used piece of heavy equipment is available for purchase at $300,000. A rental company is
deciding whether or not to purchase the equipment. The company estimates the equipment
will create annual incomes of $110,000 and have annual operating costs of $20,000. The
equipment can be depreciated in five years with straight-line depreciation. Based on the
results from part (a) below, should the rental company purchase the equipment if their
corporate tax rate is 35%? Consider a five-year life project and an after-tax MARR of 15%.

(a) Determine the return on equity for each of three different leverage factors of 0, 0.4,
and 0.7. Assume an interest rate on borrowed funds to be 10% compounded annually.
The principal payments will be constant for each of the five years and the interest paid
each year will be based on the outstanding debt balance.
(b) Assume two additional economic conditions: (i) annual income increases to $125,000
and (ii) annual income decreases to $95,000. Repeat part (a) for these two economic
conditions. Prepare a plot of the I RR on equity versus the leverage factor.

7.2. A corporations tax rate is 40%. An outlay of $35,000 is being considered for a new asset.
Estimated annual revenues are $30,000 and estimated annual operating costs are $10,000.
The useful life of the asset is 5 years and has no salvage value. Use the SYD method of
depreciation. A lending institution has offered to loan the corporation 50% of the initial
investment cost at an annual interest rate of 12.5%. The principal and interest will be
paid with a constant annual payment as calculated according to: P &I payment = Debt
(A/P )12.5%,5 . If the corporations after-tax MARR is 15%, should it accept the loan?
7.3. Solve Problem 6.7 using a leverage factor of 0.2.
7.4. Use Excel to solve Problem 6.8 for leverage factors of 0.2 and 0.4.
135

CHAPTER 8

Basic Statistics and Probability


8.1 INTRODUCTION
In previous chapters of this text, it was assumed that all of the information needed to make an eco-
nomic analysis was known without any uncertainty. In practice, this is a rare situation. Nearly always,
an evaluator will need to include a measure of the uncertainty pertaining to one or more variables
in the analysis. This uncertainty may, in turn, add significant uncertainty about the profitability of
an investment. For example, with one set of economic assumptions, the projects NPV might be
greater than zero which would indicate an acceptable investment. However, with a different set of
economic assumptions, the projects NPV might be negative, thereby indicating that the investor
should pass on this opportunity. This range of uncertainty about the projects profitability is one
way to define the risk in a project. Having a basic understanding of statistics and probability will
allow an evaluator to incorporate various risk factors into the analyses that are to be completed for a
project. Some techniques that are available to incorporate uncertainty into project variables, and that
apply the ideas of statistics and probability presented in this chapter, will be presented in Chapter 9.

8.2 STATISTICS
8.2.1 MEASURES OF CENTRAL TENDENCY
Averages are often used to represent a set of data. Several different types of averages can be calculated.
These include the arithmetic mean, the median, the mode, and the geometric mean.These are known
as measures of central tendency as they tend to be centrally located within the data.

Arithmetic Mean
The arithmetic mean of a set of data is calculated with Equation 8.1. The arithmetic mean is also
known as the expected value of the data.
N 

= xi /N (8.1)
i=1

where, xi = the ith value of the data


N = total number of data points
= arithmetic mean
136 8. BASIC STATISTICS AND PROBABILITY

Excel has a built-in function to calculate the arithmetic mean:


= AVERAGE(number1, number2,)

where, number1, number2, ... = list of data points

Median
When a set of data is arranged in order of magnitude, the median of the set is found by taking the
middle value (when there is an odd number of values) or the arithmetic mean of the two middle
values (when there is an even number of values).
Excel has a built-in function to calculate the median:

= MEDIAN(number1, number2,)

Mode
The mode is the value which occurs with the greatest frequency. A set of data can have a single
mode, several modes, or no modes.
Excel has two built-in functions to calculate the mode:

single mode: = MODE.SNGL(number1, number2, )


multiple modes: = MODE.MULT(number1, number2, )

Geometric Mean
The geometric mean of a set of data is calculated with Equation 8.2:

N

G=  N
xi (8.2)
i=1

where, xi = the ith value of the data


N = total number of data points
N
= (x1 )(x2 )(x3 ) . . . (xN 1 )(xN )
i=1

Excel has a built-in function to calculate the geometric mean:


= GEOMEAN(number1, number2,)
8.2. STATISTICS 137
Example 8.1
Consider 100 exam scores from a college-level class as shown below:

75 67 96 73 78 53 51 47 31 42
76 91 81 87 91 55 88 90 53 74
57 88 94 59 97 81 78 90 83 65
77 38 93 76 35 78 94 88 67 89
54 86 94 60 88 75 86 38 67 99
84 91 67 79 78 73 78 79 39 90
51 77 88 79 46 97 80 90 78 84
94 46 99 85 79 34 85 39 91 82
95 46 48 79 85 87 76 56 48 32
85 87 83 98 74 88 51 57 95 61

(a) Calculate the arithmetic mean:

= (75 + 67 + 96 + + 95 + 61)/100 = 73.7

(b) Calculate the median:

First, order the 100 scores from high to low. Since there is an even number of values, the
median is the average of 50th (79) and 51st (78) values, or 78.5.

(c) Calculate the mode:

Again, order the 100 scores from high to low and find the value that occurs most often. In this
case, the value of 78 occurs six times. Therefore, the mode is 78.

(d) Calculate the geometric mean:


100
G= 75 67 96 95 61 = 70.9
138 8. BASIC STATISTICS AND PROBABILITY

Using Excel :

A B C D E F G H I J

1 75 67 96 73 78 53 51 47 31 42
2 76 91 81 87 91 55 88 90 53 74
3 57 88 94 59 97 81 78 90 83 65
4 77 38 93 76 35 78 94 88 67 89
5 54 86 94 60 88 75 86 38 67 99
6 84 91 67 79 78 73 78 79 39 90
7 51 77 88 79 46 97 80 90 78 84
8 94 46 99 85 79 34 85 39 91 82
9 95 46 48 79 85 87 76 56 48 32
10 85 87 83 98 74 88 51 57 95 61
11
12 Average = 73.7
13 Median = 78.5
14 Mode = 78
15 Geo Mean = 70.9

A B C D E F G H I J
1 75 67 96 73 78 53 51 47 31 42
2 76 91 81 87 91 55 88 90 53 74
3 57 88 94 59 97 81 78 90 83 65
4 77 38 93 76 35 78 94 88 67 89
5 54 86 94 60 88 75 86 38 67 99
6 84 91 67 79 78 73 78 79 39 90
7 51 77 88 79 46 97 80 90 78 84
8 94 46 99 85 79 34 85 39 91 82
9 95 46 48 79 85 87 76 56 48 32
10 85 87 83 98 74 88 51 57 95 61
11
12 Average = =AVERAGE(A1:J10)
13 Median = =MEDIAN(A1:J10)
14 Mode = =MODE.SNGL(A1:J10)
15 Geo Mean = =GEOMEAN(A1:J10)
8.2. STATISTICS 139
8.2.2 MEASURES OF DISPERSION
It is frequently desired to determine how a set of data is dispersed or spread about its average.
Measures of dispersion which will be discussed in this chapter include the range, the mean deviation,
the standard deviation, and the variance.

Range
The range of a set of data is simply the difference between the largest and the smallest values of the
data.
In order to compute the range of a set of data with Excel , use the MAX and MIN functions:
=MAX(number1, number2,) MIN(number1, number2,)

Mean Deviation
The mean deviation (or average deviation) is the mean of the distances between each value and the
mean. It is computed with Equation 8.3:
N 

M.D. = |xi | /N (8.3)
i=1

where, xi = the ith value of the data


= the arithmetic mean of the data
N = total number of data points

Excel does not have a built-in function to calculate the mean deviation. To use Excel , do
the following:
1. Place the data in a single column (for example, assume 10 data points in cells A1 through
A10).

2. Use the formula =AVERAGE(A1:A10) in cell A11 to compute the average of this column of
data. This is the mean of the data.

3. In the adjacent column B, use the formula =ABS(A1$A$11) in cell B1.

4. Copy this formula to cells B2 through B10.

5. Use the formula =AVERAGE(B1:B10) in cell B11 to compute the average of this column of
data. This is the mean deviation of the data.

Standard Deviation
Standard deviation is another measure of the variability of a data set about its mean. Its origins are
associated with the normal distribution that is discussed later in this chapter, but it has meaning
140 8. BASIC STATISTICS AND PROBABILITY
for any set of data. A small value of standard deviation indicates that the data points are clustered
more closely to the mean than a larger value of standard deviation. If the entire population has been
sampled (that is, N equals the total possible number of data points in the population), the standard
deviation is calculated with Equation 8.4:

N

= (x )2 /N
i (8.4)
i=1
where, xi = the i value of the data
th

= the arithmetic mean of the data


N = total number of data points
If one was calculating the standard deviation of 100 exam scores in a particular college-level
class with 100 students, then N would be 100 in Equation 8.4. However, if only a subset of the
population is being sampled, N should be replaced with N 1. It can be noted that when N gets
larger than about 30, there is very little error introduced by using N instead of N 1. As an example
of a sample, assume that one wanted to measure the mean and standard deviation of the age of
the population in a city of 20,000 people. It would be difficult to get the age of all 20,000 people,
so a subset of the population is sampled (perhaps 1,000 people). One would use Equation 8.1 to
determine the mean age of the population and Equation 8.4 (with N 1 instead of N ) to determine
the standard deviation of the populations age.
Excel has two built-in functions to calculate the standard deviation:
=STDEV.P(number1, number2,) for the entire population or
=STDEV.S(number1, number2,) for a sample of the population.
Per Equation 8.4, STDEV.P contains a division by N , whereas STDEV.S contains a division
by N 1.

Example 8.2
Consider 100 exam scores from a college-level class as shown below (same as Example 8.1):
75 67 96 73 78 53 51 47 31 42
76 91 81 87 91 55 88 90 53 74
57 88 94 59 97 81 78 90 83 65
77 38 93 76 35 78 94 88 67 89
54 86 94 60 88 75 86 38 67 99
84 91 67 79 78 73 78 79 39 90
51 77 88 79 46 97 80 90 78 84
94 46 99 85 79 34 85 39 91 82
95 46 48 79 85 87 76 56 48 32
85 87 83 98 74 88 51 57 95 61
8.2. STATISTICS 141
(a) Calculate the range:

Order the numbers from high to low. The range is then given by the highest value minus the
lowest value. Range = 99-31 = 68.

(b) Calculate the mean deviation:

M.D. = (|75 73.7| + |67 73.7| + |96 73.7| + + |95 73.7| + |61 73.7|) /100
= 15.4

(c) Calculate the standard deviation:


(75 73.7)2 + (67 73.7)2 + + (95 73.7)2 + (61 73.7)2
=
100
= 18.5

Using Excel :

A B C D E F G H I J
1 75 67 96 73 78 53 51 47 31 42
2 76 91 81 87 91 55 88 90 53 74
3 57 88 94 59 97 81 78 90 83 65
4 77 38 93 76 35 78 94 88 67 89
5 54 86 94 60 88 75 86 38 67 99
6 84 91 67 79 78 73 78 79 39 90
7 51 77 88 79 46 97 80 90 78 84
8 94 46 99 85 79 34 85 39 91 82
9 95 46 48 79 85 87 76 56 48 32
10 85 87 83 98 74 88 51 57 95 61
11
12 Average = 73.7
13 Median = 78.5
14 Mode = 78
15 Geo Mean = 70.9
16 Range = 68
17 Mean Dev = 15.4
18 StdDev = 18.5
142 8. BASIC STATISTICS AND PROBABILITY

A B C D E F G H I J
1 75 67 96 73 78 53 51 47 31 42
2 76 91 81 87 91 55 88 90 53 74
3 57 88 94 59 97 81 78 90 83 65
4 77 38 93 76 35 78 94 88 67 89
5 54 86 94 60 88 75 86 38 67 99
6 84 91 67 79 78 73 78 79 39 90
7 51 77 88 79 46 97 80 90 78 84
8 94 46 99 85 79 34 85 39 91 82
9 95 46 48 79 85 87 76 56 48 32
10 85 87 83 98 74 88 51 57 95 61
11
12 Average = =AVERAGE(A1:J10)
13 Median = =MEDIAN(A1:J10)
14 Mode = =MODE.SNGL(A1:J10)
15 Geo Mean = =GEOMEAN(A1:J10)
16 Range = =MAX(A1:J10)-MIN(A1:J10)
17 Mean Dev = =B121**
18 StdDev = =STDEV.P(A1:J10)

**This assumes that the 100 data points are copied to cells A21:A120 and the procedure listed
above under Mean Deviation is followed.

8.2.3 FREQUENCY DISTRIBUTIONS


The creation of a frequency distribution is another technique to summarize large numbers of raw data.
When the raw data are summarized, they begin to take on more meaning and utility. A frequency
distribution is made by grouping the raw data into classes and counting the number of items that
fall into each class. This number is referred to as the class frequency. A table is then formed which
contains a column for the class, a column for the class frequency, and a column for the cumulative
class frequency. The resulting table is the frequency distribution. The size and number of classes will
depend upon the particular application that is being considered. Typically, frequency distributions
contain five to ten classes, all of equal size. However, some data might lend themselves to classes of
unequal size or even classes that might be open ended (normally, the first class or the last class or
both).
The cumulative frequency distribution, Fi , is the summation of the frequency distribution.
8.2. STATISTICS 143
Example 8.3
Consider 100 exam scores from a college-level class as shown below (same as Example 8.1):
75 67 96 73 78 53 51 47 31 42
76 91 81 87 91 55 88 90 53 74
57 88 94 59 97 81 78 90 83 65
77 38 93 76 35 78 94 88 67 89
54 86 94 60 88 75 86 38 67 99
84 91 67 79 78 73 78 79 39 90
51 77 88 79 46 97 80 90 78 84
94 46 99 85 79 34 85 39 91 82
95 46 48 79 85 87 76 56 48 32
85 87 83 98 74 88 51 57 95 61

Using ten classes from 0100, develop the frequency distribution for the data.
Solution:

Within the frequency distribution, the range of numbers that is used to define the class is
called a class interval. The smaller number is the lower class limit and the larger number is the upper
class limit. Note that in Example 8.3, the upper class limit of one class is the same as the lower class
limit of the next class. If a value is exactly equal to one of the class limits, one needs to decide in
which class it belongs. It doesnt matter if it is placed in the higher range or the lower range as long
as the evaluator remains consistent. In Example 8.3, any value that is equal to a class limit is placed
in the lower range (e.g., a value of 90 is placed in the 80-90 class). If this convention is used, then
one can define true class limits for a range. In this case, the true class limits would be 90.5-100.5,
80.5-90.5, 70.5-80.5, etc. Excel utilizes this convention as well.
There are two other terms that need to be defined for frequency distributions. The class size
is the difference between the upper true class limit and the lower true class limit. The true class mark
144 8. BASIC STATISTICS AND PROBABILITY
is the midpoint of each true class interval or the average between the upper true class limit and the
lower true class limit. In Example 8.3, the class size is ten for all ten classes and the true class marks
are 95.5, 85.5, 75.5, etc.

True Class True Class Frequency Cumula ve


Limits Mark fi Frequency, Fi
0.5-10.5 5.5 0 0
10.5-20.5 15.5 0 0
20.5-30.5 25.5 0 0
30.5-40.5 35.5 8 8
40.5-50.5 45.5 7 15
50.5-60.5 55.5 12 27
60.5-70.5 65.5 6 33
70.5-80.5 75.5 23 56
80.5-90.5 85.5 27 83
90.5-100.5 95.5 17 100
Total 100

Frequency distributions are often represented graphically. Graphical representations include


histograms, frequency polygons, and relative and cumulative relative frequency diagrams.
A histogram consists of a set of rectangles, where a rectangle is drawn for each class interval
with the width of each rectangle equal to the class size and the height of the rectangle is the class
frequency. The histogram is constructed so that the center of each rectangle lies at its true class mark.
Figure 8.1 is the histogram for the data presented in Example 8.3.
A frequency polygon can be generated by creating a line graph of the frequency of each class
as a function of the true class marks. Figure 8.2 is the frequency polygon for the data presented in
Example 8.3. The first and last points of the polygon are found on the x-axis at what would be the
true class marks associated with class intervals before the first actual class interval (located at 4.5)
and after the last actual class interval (located at 105.5).
8.2. STATISTICS 145

Histogram
30

25

20

Freq 15

10

0
5.5 15.5 25.5 35.5 45.5 55.5 65.5 75.5 85.5 95.5
Exam Scores

Figure 8.1: Histogram for Example 8.3.

Frequency Polygon
30

25

20

Freq 15

10

0
-4.5 5.5 15.5 25.5 35.5 45.5 55.5 65.5 75.5 85.5 95.5 105.5
Exam Scores

Figure 8.2: Frequency polygon for Example 8.3.


146 8. BASIC STATISTICS AND PROBABILITY
8.2.4 RELATIVE FREQUENCY DISTRIBUTION
The relative frequency distribution is constructed by dividing the number of occurrences in each
class interval by the total number of points in the data set. The following shows the relative fre-
quency distribution for Example 8.3 while Figures 8.3 and 8.4 show the graphical versions of these
distributions.

True Class True Class Rela ve Cumula ve


Limits Mark Frequency, Rela ve
fi Frequency, Fi
0.5-10.5 5.5 0.00 0.00
10.5-20.5 15.5 0.00 0.00
20.5-30.5 25.5 0.00 0.00
30.5-40.5 35.5 0.08 0.08
40.5-50.5 45.5 0.07 0.15
50.5-60.5 55.5 0.12 0.27
60.5-70.5 65.5 0.06 0.33
70.5-80.5 75.5 0.23 0.56
80.5-90.5 85.5 0.27 0.83
90.5-100.5 95.5 0.17 1.00

Relave Frequency Distribuon


0.3

0.25

0.2

Rel Freq 0.15

0.1

0.05

0
-4.5 5.5 15.5 25.5 35.5 45.5 55.5 65.5 75.5 85.5 95.5 105.5
Exam Scores

Figure 8.3: Relative frequency distribution for Example 8.3.


8.2. STATISTICS 147

Cumulave Relave Frequency


1
0.9
0.8
0.7
Cumul 0.6
Relave 0.5
Freq 0.4
0.3
0.2
0.1
0
-4.5 5.5 15.5 25.5 35.5 45.5 55.5 65.5 75.5 85.5 95.5 105.5
Exam Scores

Figure 8.4: Cumulative relative frequency for Example 8.3.

If the data are presented in frequency distribution form, items such as the mean, mean devi-
ation, and standard deviation can be determined from the following equations, respectively.


M
 = fj xj (8.5)
j =1
M  
 
M.D. = fj xj   (8.6)
j =1

M
  2
=

fj xj  (8.7)
j =1

where, fj = the relative frequency of the j th class


xj = the true class mark of the j th class
M = total number of classes
 = the arithmetic mean of the data based on the distribution
M.D. = the mean deviation of the data based on the distribution
 = the standard deviation of the data based on the distribution
148 8. BASIC STATISTICS AND PROBABILITY
Example 8.4
Calculate the mean, mean deviation, and standard deviation for the data in Example 8.1, using
the relative frequency distributions found in Example 8.3:

Mean:

 = 0.00 5.5 + 0.00 15.5 + + 0.27 85.5 + 0.17 95.5 = 73.3

This value compares favorably to 73.7 computed with all 100 data points.
Mean Deviation:

M.D. =0.00 |5.5 73.3| + 0.00 |15.5 73.3| + +


0.27 |85.5 73.3| + 0.17 |95.5 73.3|
=15.1

This value compares favorably to 15.4 computed with all 100 data points.
Standard Deviation:

0.00 (5.5 73.3)2 + 0.00 (15.5 73.3)2 + +
 =
0.27 (85.5 73.3)2 + 0.00 (95.5 73.3)2
= 18.3

This value compares favorably to 18.5 computed with all 100 data points.
8.3. PROBABILITY 149
8.3 PROBABILITY
8.3.1 CLASSICAL DEFINITION
The classical definition of probability involves an event occurring from a group or a set of equally
likely outcomes. That is, when a fair coin is tossed, the specific outcome of that event is either a
heads or a tails with each outcome equally likely to occur. Suppose that a particular event occurs
a certain number of times out of a total possible number of other events. The probability that the
desired event will occur is given by Equation 8.8:
P (A) = nA /n (8.8)

where, P (A) = the probability of event A occuring


nA = the number of times event A could occur
n = the total number of possible events

Example 8.5
Consider the probability of drawing an ace from a fair deck of cards. Since there are four aces
in a total of 52 cards and the chances of drawing any specific card is the same, the probability of
drawing an ace would be:
P (Ace) = 4/52 = 1/13 = 0.0769 = 7.69%
Note that in the above example, the probability of 4/52 is only correct for any given attempt
to draw an ace if, when an undesired card is drawn (not an ace), it is returned to the deck before
the next card is drawn. This is referred to as sampling with replacement. If the undesired card is not
returned to the deck, known as sampling without replacement, the probability changes to 4/51, then
4/50, etc.

Example 8.6
Consider the probability of rolling two fair die and getting a total of 8. When rolling two fair
die, the possible outcomes are:

Die 1 Die 2 Total Die 1 Die 2 Total Die 1 Die 2 Total


1 1 2 2 1 3 3 1 4
1 2 3 2 2 4 3 2 5
1 3 4 2 3 5 3 3 6
1 4 5 2 4 6 3 4 7
1 5 6 2 5 7 3 5 8
1 6 7 2 6 8 3 6 9
150 8. BASIC STATISTICS AND PROBABILITY

Die 1 Die 2 Total Die 1 Die 2 Total Die 1 Die 2 Total


4 1 5 5 1 6 6 1 7
4 2 6 5 2 7 6 2 8
4 3 7 5 3 8 6 3 9
4 4 8 5 4 9 6 4 10
4 5 9 5 5 10 6 5 11
4 6 10 5 6 11 6 6 12

As shown in the table, there are 36 possible outcomes of the dice rolls, five of which have a
value of 8. Therefore, the probability of getting exactly 8 is:

P (8) = 5/36 = 0.139 = 13.9%

Example 8.7
Consider the flipping of a fair coin (50% probability of a head and 50% probability of a tail).
The coin will be flipped three times. What is the probability that 0, 1, 2, and 3 heads occurred in
the three flips?

First Flip Second Flip Third Flip Total # of Heads


T T T 0
T T H 1
T H T 1
T H H 2
H T T 1
H T H 2
H H T 2
H H H 3

As shown in the table, there are eight possible outcomes of the three flips. Therefore,

P (0Heads) = 1/8 = 0.125 = 12.5%


P (1Heads) = 3/8 = 0.375 = 37.5%
P (2Heads) = 3/8 = 0.375 = 37.5%
P (3Heads) = 1/8 = 0.125 = 12.5%
8.3. PROBABILITY 151
8.3.2 RELATIVE FREQUENCY DEFINITION
The classical definition of probability uses the concept of equally likely outcomes to aid in the
definition. Since the words equally likely themselves imply some notion of probability, the definition
would appear to be a bit circular in nature. To get around this, the concept of relative frequency
probability was introduced. If an experiment or trial is going to be repeated a large number of times,
the probability that a particular event of the experiment will occur is given by the relative frequency
shown in Equation 8.9:
P (A) = lim (nA /n) (8.9)
n

Example 8.8
Consider the tossing of a coin. One would normally assume that the probability of getting a
head on any one toss is 50%. However, consider an experiment where a coin is tossed 100 times and
heads occurs 52 times and tails 48 times. For this case, the probability of getting a head would be
predicted to be 52%. If the coin is a fair one, as the number of experimental data points gets larger
and larger, the probability of getting a head will approach 50%.
In most cases throughout engineering, one does not have the luxury of performing an infinite
number of experiments in order to determine the true probability of an event occurring. For example,
if an engineer is doing failure tests on a particular manufactured component, one can only do a limited
number of failure experiments in order to determine the probability of failure.

8.3.3 SUBJECTIVE DEFINITION


A third type of probability definition is known as subjective probability. This type of probability
is not determined from theoretical or experimental work, but rather from the experience that an
individual or group of individuals has gained during their career in a particular area. This experience
is then used to predict the probability of future events. For example, a civil engineer who does road
design will gain, over time, a feeling or estimation of the probability that a road surface will begin
to fail within a certain time range based on weather conditions, quantity of traffic, type of surfacing
materials used, etc.
In summary, for economic evaluations, it is necessary that probabilities of certain outcomes be
assigned. This allows the evaluator to incorporate risk factors into economic analysis situations. The
difficulty is often in the assigning of the actual probabilities. One or more of the above definitions
may assist the evaluator in this task.

8.3.4 PROBABILITY DISTRIBUTIONS


For a given event or set of events, if probability or frequency distributions can be established, the
statistical concepts discussed earlier can be applied to calculate means and standard deviations for
each event.
152 8. BASIC STATISTICS AND PROBABILITY
Two different types of probability distributions, discrete and continuous, will be discussed
below.

Discrete Distribution
A discrete distribution is one which involves an experiment with a finite number of possibilities.
For example, as described earlier, when a fair die is thrown, a 1, 2, 3, 4, 5, or 6 will occur. Thus,
the outcome of a throw is a discrete value. Using the classical probability definition, each possible
outcome would have a probability of 1/6. Note that the sum of the probabilities of all possible
outcomes will always equal 1.0.

Example 8.9
Consider that a certain discrete random variable, x, has a discrete probability distribution as
follows:

(a) Graph the distribution

(b) Find the mean

(c) Find the standard deviation

(a) Graphically, the distribution is shown in Figure 8.5.

Discrete Probability Distribuon


0.3

0.25

0.2

P(x) 0.15

0.1

0.05

0
-3 -1 0 1 2 3 5 8
x

Figure 8.5: Probability distribution for the random discrete variable x in Example 8.9.
8.3. PROBABILITY 153
For this distribution, it would be useful to calculate the mean and standard deviation using
Equations 8.5 and 8.7:

(b)  = 0.1 (3) + 0.2 (1) + 0.15 (0) + 0.25 (1) + 0.1 (2) + 0.1 (3)
+ 0.05 (5) + 0.05 (8) = 0.90

 0.1 (3 0.9)2 + 0.2 (1 0.9)2 + 0.15 (0 0.9)2 + 0.25 (1 0.9)2
(c) =
+ 0.1 (2 0.9)2 + 0.1 (3 0.9)2 + 0.05 (5 0.9)2 + 0.05 (8 0.9)2
= 2.51

Solution using Excel :

A B C D E
1 x P(x) x*P(x) (x-mu)^2 P(x)*(x-mu)^2
2 -3 0.1 -0.3 15.2 1.521
3 -1 0.2 -0.2 3.6 0.722
4 0 0.15 0 0.8 0.122
5 1 0.25 0.25 0.0 0.003
6 2 0.1 0.2 1.2 0.121
7 3 0.1 0.3 4.4 0.441
8 5 0.05 0.25 16.8 0.841
9 8 0.05 0.4 50.4 2.521
10
11 Total = 1.00
12 Mean = 0.90
13 StdDev = 2.51
154 8. BASIC STATISTICS AND PROBABILITY

A B C D E
1 x P(x) x*P(x) (x-mu)^2 P(x)*(x-mu)^2
2 -3 0.1 =A2*B2 =(A2-B$12)^2 =B2*D2
3 -1 0.2 =A3*B3 =(A3-B$12)^2 =B3*D3
4 0 0.15 =A4*B4 =(A4-B$12)^2 =B4*D4
5 1 0.25 =A5*B5 =(A5-B$12)^2 =B5*D5
6 2 0.1 =A6*B6 =(A6-B$12)^2 =B6*D6
7 3 0.1 =A7*B7 =(A7-B$12)^2 =B7*D7
8 5 0.05 =A8*B8 =(A8-B$12)^2 =B8*D8
9 8 0.05 =A9*B9 =(A9-B$12)^2 =B9*D9
10
11 Total = =SUM(B2:B9)
12 Mean = =SUM(C2:C9)
13 StdDev = =SQRT(SUM(E2:E9))

Binomial Distribution
The binomial distribution is a standard discrete distribution that accounts for the case where there
are two possible events and the probabilities of each event are not the same. Given a number of
independent trials, n, of an experiment that has two possible outcomes (call them success and failure),
the probability of a certain number of successes occurring in those n trials is given by Equation 8.10:

Pn (x) = Cxn p x q nx (8.10)

where, Pn (x) = the probability of x successes in n trials


Cxn = the number of combinations of n items taken x at a time = n!/(x!(n x)!)
p = the probability of a success for any given trial
q = the probability of a failure for any given trial = 1 p

In addition to the probability of exactly x successes in n trials, it is also common to determine


the probability of less than k successes, greater than k successes, or between l and k successes.
8.3. PROBABILITY 155
Specifically,

k1
Pn (x < k) = Pn (j ) (8.11)
j =0
n
Pn (x > k) = Pn (j ) (8.12)
j =k+1

k1
Pn (l < x < k) = Pn (j ) (8.13)
j =l+1

Excel has a built-in function to compute a binomial distribution:


=BINOM.DIST(#_successes, #_trials, prob_of_success, cumulative).

where, #_successes = number of successes in n trials (x)


#_trials = number of trials (n)
prob_of_success = probability of success (p)
cumulative = FALSE for probability distribution
= TRUE for cumulative probability distribution

Example 8.10
The probability that a fuse will be defective when first installed is 0.08. If six fuses are selected
at random, find each of the following:
(a) The probability that less than two fuses are defective
(b) The probability that four or more fuses are defective
(c) The probability that at least one is defective
Solution:
Define a success as a fuse that is defective. Therefore, p = 0.08 and q = 0.92.
6!
P6 (0) = C60 (0.08)0 (0.92)6 = (0.08)0 (0.92)6
0!6!
= 0.606
6!
P6 (1) = C61 (0.08)1 (0.92)5 = (0.08)1 (0.92)5
1!5!
= 0.316
6!
P6 (2) = C62 (0.08)2 (0.92)4 = (0.08)2 (0.92)4
2!4!
= 0.0688
156 8. BASIC STATISTICS AND PROBABILITY
6!
P6 (3) = C63 (0.08)3 (0.92)3 = (0.08)3 (0.92)3
3!3!
= 0.00797
6!
P6 (4) = C64 (0.08)4 (0.92)2 = (0.08)4 (0.92)2
4!2!
= 0.000520
6!
P6 (5) = C65 (0.08)5 (0.92)1 = (0.08)5 (0.92)1
5!1!
= 0.0000181
6!
P6 (6) = C66 (0.08)6 (0.92)0 = (0.08)6 (0.92)0
6!0!
= 0.000000262
(a) P6 (x < 2) = P6 (0) + P6 (1) = 0.606 + 0.316 = 0.922
(b) P6 (x 4) = P6 (4) + P6 (5) + P6 (6) = 5.20 104 + 1.81 105 + 2.62 107 =
5.38 104
(c) P6 (x > 0) = P6 (1) + P6 (2) + P6 (3) + P6 (4) + P6 (5) + P6 (6) = 0.394
Alternatively, P6 (x > 0) = 1 P6 (0) = 1 0.606 = 0.394

Using Excel :
8.3. PROBABILITY 157
In graphical form, the discrete distribution for Example 8.10 can be shown as:

Binomial Distribuon
0.7 6.06E-01
0.6
0.5
0.4 3.16E-01
P(x)
0.3
0.2
6.88E-02
0.1 7.97E-03 5.20E-04 1.81E-05 2.62E-07
0
0 1 2 3 4 5 6
# of Successes

Continuous Distributions
When the value of the event, x, can take on a continuous set of probability values, rather than
a set of specific values, then a probability density function, p(x), exists. While there are a wide
variety of continuous distributions possible, the authors have chosen to present three continuous
distributions: the uniform distribution, the triangular distribution, and the normal or Gaussian
distribution. Figure 8.6 is a representation of a continuous distribution.
For continuous probability distributions, the following statements and equations are pertinent:

1. For a given x, p(x) is not the probability of that exact value occurring. Since there are an
infinite number of values for x, the probability of any one specific value of x would be zero.

2. The total area under the curve will equal the value of unity.

3. The mean of the data is calculated with Equation 8.14.



= xp(x)dx (8.14)

158 8. BASIC STATISTICS AND PROBABILITY

Connuous Distribuon
0.25

0.2

0.15
p(x)
0.1

0.05

0
0 2 4 6 8 10 12 14 16
x

Figure 8.6: Continuous distribution.

4. The standard deviation of the data is calculated with Equation 8.15:




= x 2 p(x)dx 2 (8.15)

5. The cumulative probability, F (x), is defined with Equation 8.16.


 x
F (x) = p(x)dx (8.16)

6. F (x1 ) represents the probability that the value of x is less than or equal to x1 .

7. The quantity (1 F (x1 )) represents the probability that the value of x is greater than or equal
to x1 .

Uniform or Rectangular Distribution


The uniform or rectangular distribution is represented in Figure 8.7. Each value of x has the same
probability of occurring.
Let a be the minimum value of x and b be the maximum value of x. Since the area under the
probability curve must be unity, the height of the uniform distribution will be given by Equation 8.17:

h = 1/(b a) (8.17)
8.3. PROBABILITY 159

Figure 8.7: Uniform or rectangular distribution.

The uniform distribution then has the following properties:

p(x) = h for a x b
p(x) = 0 for all other values of x
= (a + b)/2 (8.18)

= (b a)/ 12 (8.19)
F (x) = (x a)/(b a) (8.20)

Triangular Distribution
The triangular distribution is represented in Figure 8.8.
Let a be the minimum value of x, c be the maximum value of x, and b be the mode. P1 and P2
represent the areas from a to b and b to c, respectively. The triangular distribution has the following
properties:

h = 2/(c a) (8.21)
P1 = (b a)/(c a) (8.22)
P2 = (c b)/(c a) (8.23)
160 8. BASIC STATISTICS AND PROBABILITY

Figure 8.8: Triangular distribution.

= (a + b + c)/3 (8.24)

= (c a) (1 P1 P2 )/18 (8.25)
F (x) = P1 [(x a)/(b a)]2 for a x b (8.26)
F (x) = 1 P2 [(c x)/(c b)]2 for b x c (8.27)

Normal Distribution
The Normal or Gaussian distribution is a continuous probability function that takes on the common
bell-shaped curve as represented in Figure 8.9.
The shape of this distribution is calculated with Equation 8.28:

 
1 x 2
1
p(x) = e 2
(8.28)
2

where, = the mean of the data


= the standard deviation of the data
range of variable x: x
8.3. PROBABILITY 161

Figure 8.9: Representation of a unit normal distribution ( = 0, = 1).

When = 0 and = 1, the distribution is called a unit normal distribution and Equation 8.28
simplifies to Equation 8.29:
1 x2
p(x) = e 2 (8.29)
2
One can convert any set of normally distributed data to a unit normal distribution through
the substitution of the variable Z, defined as:
Z = (x )/ (8.30)
This allows one to then use Table 8.1 to determine values of p(Z) and F (Z) as defined above.
Since the unit normal distribution is symmetrical about Z = 0, one only needs the positive
portion of the table. If Z < 0, then use the following equations for p(Z) and F (z):
p(Z) = p(Z) (8.31)
F (Z) = 1 F (Z) (8.32)

Excel has a built-in function that calculates p(x) and F (x) given x, the mean, and the
standard deviation:
=NORM.DIST(x,Mean,Std_Dev,Cumulative)

where, x = value at which to find the value of either of p(x) of F (x)


Mean = mean of the distribution
Std_Dev = standard deviation of the distribution
Cumulative = FALSE for p(x) or =TRUE for F (x).
162 8. BASIC STATISTICS AND PROBABILITY

Table 8.1: Values of p(Z) and F (Z) for the unit normal distribution
Z p(Z) F(Z) Z p(Z) F(Z)
0.00 0.39894 0.50000 1.55 0.12001 0.93943
0.05 0.39844 0.51994 1.60 0.11092 0.94520
0.10 0.39695 0.53983 1.65 0.10226 0.95053
0.15 0.39448 0.55962 1.70 0.09405 0.95543
0.20 0.39104 0.57926 1.75 0.08628 0.95994
0.25 0.38667 0.59871 1.80 0.07895 0.96407
0.30 0.38139 0.61791 1.85 0.07206 0.96784
0.35 0.37524 0.63683 1.90 0.06562 0.97128
0.40 0.36827 0.65542 1.95 0.05959 0.97441
0.45 0.36053 0.67364 2.00 0.05399 0.97725
0.50 0.35207 0.69146 2.05 0.04879 0.97982
0.55 0.34294 0.70884 2.10 0.04398 0.98214
0.60 0.33322 0.72575 2.15 0.03955 0.98422
0.65 0.32297 0.74215 2.20 0.03547 0.98610
0.70 0.31225 0.75804 2.25 0.03174 0.98778
0.75 0.30114 0.77337 2.30 0.02833 0.98928
0.80 0.28969 0.78814 2.35 0.02522 0.99061
0.85 0.27798 0.80234 2.40 0.02239 0.99180
0.90 0.26609 0.81594 2.45 0.01984 0.99286
0.95 0.25406 0.82894 2.50 0.01753 0.99379
1.00 0.24197 0.84134 2.55 0.01545 0.99461
1.05 0.22988 0.85314 2.60 0.01358 0.99534
1.10 0.21785 0.86433 2.65 0.01191 0.99598
1.15 0.20594 0.87493 2.70 0.01042 0.99653
1.20 0.19419 0.88493 2.75 0.00909 0.99702
1.25 0.18265 0.89435 2.80 0.00792 0.99744
1.30 0.17137 0.90320 2.85 0.00687 0.99781
1.35 0.16038 0.91149 2.90 0.00595 0.99813
1.40 0.14973 0.91924 2.95 0.00514 0.99841
1.45 0.13943 0.92647 3.00 0.00443 0.99865
1.50 0.12952 0.93319
8.3. PROBABILITY 163
There is also a built-in function that calculates x given F (x), the mean, and the standard
deviation:
=NORM.INV(F(x),Mean,Std_Dev)

where, F(x) = value of the cumulative distribution at which to find the value of x

Example 8.11
An engineer estimates that the selling price of a particular commodity will range from a low
of $5.00 per item to a high of $10.00 per item.
(a) If the distribution is assumed to be uniform, calculate the mean (or expected) value and the
standard deviation for the price of this commodity. Also, calculate the probability that the
price will be greater than $9.00.

(b) If the distribution is assumed to be triangular with a most likely value (mode) of $7.00 per
item, calculate the mean (or expected) value and the standard deviation for the price of this
commodity. Also, calculate the probability that the price will be greater than $9.00.
Solution (a):
The distribution would be:

The mean would be: = (a + b)/2 = (5 + 10)/2 = 7.50



The standard deviation would be: = (b a)/ 12 = (10 5)/ 12 = 1.443
The probability of the price being greater than $9.00:

Prob(> $9) = 1 F (9) = 1 {(9 5)/(10 5)} = 0.20


164 8. BASIC STATISTICS AND PROBABILITY
Solution (b):

The distribution would be:

The mean would be: = (a + b + c)/3 = (5 + 7 + 10)/3 = 7.33


The areas would be: P1 = (b a)/(c a) = 2/5 = 0.4 and P2 = (c b)/(c a) =
3/5 = 0.6
The standard deviation would be:
 
= (c a) (1 P1 P2 )/18 = 5 (1 (0.4)(0.6))/18 = 1.03

The probability of the price being greater than $9.00:


 
Prob(> $9) = 1 F (9) = 1 1 (0.6) [(10 9)/(10 7)]2 = 0.067

Example 8.12
300 ball bearings are tested for their diameters. The mean diameter was determined to be
0.452 cm and the standard deviation was determined to be 0.010 cm. Assume that the diameters
are normally distributed.

(a) How many ball bearings would be expected to be smaller than 0.4425 cm?

(b) Seventy percent of the ball bearings would be expected to have a diameter greater than what
value?
8.3. PROBABILITY 165
The distribution would be:

Solution for (a) using the F (Z) table:

Z = (x )/ = (0.4425 0.452)/0.010 = 0.95

F (0.95) = 1 F (0.95) = 1 0.82894 = 0.17106

# of ball bearings less than 0.4425 cm diameter = 0.17106(300) = 51

Solution for (a) using Excel :

A B A B
1 Mean = 0.452 1 Mean = 0.452
2 StdDev = 0.010 2 St d D e v = 0.010
3 x= 0.4425 3 x= 0.4425
4 F(x) = 0.17106 4 F(x) = =NORM.DIST(B3,B1,B2,TRUE)
5 # bearings < x = 51 5 # bearings < x = =300*B4

Solution for (b) using the F (z) table:


One needs the value of Z that produces an F (Z) of 0.30 (for 70% greater than that value).
Since 0.30 is less than 0.5, one needs the negative side of the curve. F (Z) = 1 F (z) = 1 .3 =
.7. The value of Z for F (0.7) lies between 0.50 and 0.55. Interpolating, Z = 0.524. Therefore,
Z for F (Z) of 0.30 is Z = 0.524. Solving for x in Equation 8.30 yields x = Z + =
0.524(0.010) + 0.452 = 0.447 cm.Therefore, 70% of the ball bearings will have a diameter greater
than 0.447 cm.
166 8. BASIC STATISTICS AND PROBABILITY

Solution for (b) using Excel :

A B A B
1 Mean = 0.452 1 Mean = 0.452
2 StdDev = 0.010 2 StdDev = 0.010
3 F(x) = 0.3 3 F(x) = 0.3
4 x= 0.44676 4 x = =NORM.INV(B3,B1,B2)

Combined Distributions
In some applications, it will be necessary to work with more than one distribution to describe a
particular variable. In order to find the mean and standard deviation for the combined distributions,
the mean and standard deviation for each separate distribution are first determined. Equations 8.33
and 8.34 are then used to calculate the overall average and standard deviation:

c = Ai i (8.33)

  
c = Ai i2 + (i c )2 (8.34)

where, c = mean of the combined distributions


c = standard deviation of the combined distributions
 
Ai = probability area associated with each distribution Ai = 1
i = mean of each distribution
i = standard deviation of each distribution

Example 8.13
An oil well has a 25% chance of being a dry hole (no oil found) and a 75% chance of finding
an oil reservoir that contains between 10,000 and 60,000 barrels as shown in the distribution below.

(a) Calculate the mean and standard deviation of the combined distributions.

(b) What is the probability that the reservoir will contain less than 40,000 barrels?

(c) What is the probability that the reservoir will contain at least 50,000 barrels?

(d) Sketch the cumulative probability distribution, F (x).


8.3. PROBABILITY 167

0.25

p(x)

0 10,000 60,000
barrels

Solution for (a):


Since the discrete probability at x = 0 is 0.25, the remaining area under the uniform distribu-
tion is then 0.75. The mean of the discrete probability distribution is 0 barrels and the mean
of the uniform distribution is 35,000 barrels.
The mean of the combined distribution: c = 0.25(0) + 0.75(35, 000) = 26, 250 barrels
The standard deviation of the discrete probability distribution is 0 barrels and the standard
deviation of the uniform distribution is 14,434 barrels.
The standard deviation of the combined distribution is
    
c = 0.25 02 + (0 26, 250)2 + 0.75 14, 4342 + (35, 000 26, 250)2
= 19, 645 barrels

Solution for (b):


P rob(< 40, 000) = F (40, 000). Recall that F (x) is the area under the probability curve.
Therefore, F (40, 000) = 0.25 + 0.75 [(40, 000 10, 000)/(60, 000 10, 000)] = 0.70.
There is a 70% probability that the reservoir will contain less than 40,000 barrels.
Solution for (c):

P rob(> 50, 000) = 1 F (50, 000)


= 1 [0.25 + 0.75 [(50, 000 10, 000)/(60, 000 10, 000)]] = 0.15.
There is a 15% probability that the reservoir will contain at least 50,000 barrels.
168 8. BASIC STATISTICS AND PROBABILITY
Solution for (d):

Again, F (x) is the area under the probability distribution. Therefore,

for x<0 F (x) = 0


for 0 x 10, 000 F (x) = 0.25
for 10, 000 x 60, 000 F (x) = 0.25 + 0.75 [(x 10, 000)/(60, 000 10, 000)]
for x > 60, 000 F (x) = 1.0

Cumulave Probability
1
0.9
0.8
0.7
0.6
F(x) 0.5
0.4
0.3
0.2
0.1
0
-10000 0 10000 20000 30000 40000 50000 60000 70000
Barrels

8.4 PROBLEMS
8.1. The following values of Youngs Modulus for a rubber compound (in 1000 lb/in 2 ) have
been measured. Determine the following:

(a) The frequency distribution using class boundaries of 4-6, 6-8, etc
(b) True class boundaries and true class marks
(c) The histogram and cumulative frequency diagrams

8 5 12 14 13 10 9
11 6 11 9 8 15 8
5 6 8 11 4 8 10
13 12 6 10 9 8 13
8.4. PROBLEMS 169
8.2. For the data in Problem 8.1, calculate the mean, median, mode, and standard deviation.
Recalculate the mean and standard deviation using the frequency distribution determined
in Problem 8.1.

8.3. A particular event has two possible outcomes of true and false. There is a 50% probability of
getting a true outcome. The event is repeated four times. Construct a table that contains all
possible combinations of results and determine the probabilities of getting 0 true outcomes,
1 true outcome, 2 true outcomes, 3 true outcomes, and 4 true outcomes.

8.4. Fifteen castings of a certain type are produced per day in a foundry. The finished castings
are inspected and classified as defective or non-defective. Records indicate that of the last
500 castings inspected, 16 were defective. Based on this information, find the following:

(a) The probability of having at no defective castings in a days production

(b) The probability of having at least two defective castings in a days production

8.5. The height of trucks on an interstate highway is approximately normally distributed with
mean of 10 ft and standard deviation of 1.5 ft. What is the height of an overpass if the
probability that a truck will clear it is 0.999?

8.6. The average life of a certain type of compressor is 10 years with a standard deviation of 1
year. The lives of the compressors follow a normal distribution. The manufacturer replaces,
at no cost, all compressors that fail while under the guarantee. If the manufacturer is willing
to replace only 3% of all compressors sold, how long of a guarantee should they offer?

8.7. A discrete distribution is given in the table below. Calculate the mean and standard deviation
of the distribution.

x 1 2 3 4
p(x) 0.2 0.3 0.1 0.4
170 8. BASIC STATISTICS AND PROBABILITY
8.8. Determine the mean and standard deviation of the following combined distribution.

8.9. A company is desirous of purchasing a service. The service will cost $10,000 and have a
probability of its life that can be described by a triangular distribution with values of a, b,
and c equal to 1, 3, and 6 years, respectively.

(a) Calculate the mean and the standard deviation of the life of the service
(b) What is the probability that the service will last at least 2 years?
(c) What is the probability that the service will last at least 5 years?
171

CHAPTER 9

Sensitivity Analysis
9.1 INTRODUCTION

A simple way of incorporating the elements of uncertainty into an economic analysis is to use
sensitivity analysis. As described earlier, an evaluator will normally need to include a measure of the
uncertainty pertaining to one or more variables in the analysis. This uncertainty may, in turn, add
significant uncertainty about the profitability of an investment. This range of uncertainty about the
projects profitability is one way to define the risk in a project.
Uncertainty in any particular variable can occur for a number of reasons. For example, the
method of measuring a parameter may have a certain amount of inaccuracy, the parameter may have
to be predicted into the future, or there may be a limited amount of data for a certain parameter. In
any case, the best that can be done for a variable with an uncertain value is to choose a reasonable
range over which it may vary and, perhaps, the type of distribution that the variable might take on
over that range.
Two types of sensitivity analysis will be considered in this chapter. The first is called the
range approach and involves the systematic variation of key variables to determine their overall
effect on the profitability of the investment. The second approach uses the concepts of probability
and statistics and is referred to as Monte Carlo Simulation (MCS). MCS has also been called
probabilistic sensitivity analysis.

9.1.1 RANGE APPROACH


When applying the range approach, ranges of variations of key variables, defined by the evaluator, are
established. For example, the estimated sales price of a commodity to be sold could be allowed to vary
10% from a base value during the analysis. The economic analysis is conducted by: (a) choosing an
evaluation criteria (normally NPV or IRR); (b) computing the value of this criteria for a base case
set of variable values; and (c) repeating the computations by varying each key parameter within the
specified range.
There are, in general, two ways of conducting the range approach sensitivity analysis.
The first is by identifying the most likely, most optimistic, and the most pessimistic cases
by varying all of the parameters simultaneously. The most likely case is defined as the case where
all variables are at their respective mean values. This method allows the evaluator to determine
the minimum and maximum values that could be obtained for the evaluation criteria. It does not,
however, allow the evaluator to study the effects of any one variable on the economic analysis.
172 9. SENSITIVITY ANALYSIS
The second way is to use the mean values for each key variable and calculate the corresponding
value for the evaluation criteria. This is then designated as the base case value. Each key parameter
is then varied about its mean value while the other parameters are held constant at their base case
values and the evaluation criteria is recalculated. The process is repeated until each parameter has
been varied. Typically, each parameter is varied plus or minus 10 to 20% about its mean value. When
the calculations have been completed, the results are usually summarized in a spider plot. The
spider plot, shown schematically in Figure 9.1, is constructed by plotting the evaluation criteria on
the vertical axis and the percent variation on the horizontal axis. Quick inspection of the spider
plot provides information to the evaluator on which parameter or parameters affect the economic
analysis to the greatest degree. The parameter which yields the line with the greatest slope (positive
or negative slope) on the spider plot has the most effect on the analysis. As illustrated in the sample
plot shown in Figure 9.1, Variable A has the greatest effect on the evaluation criteria.

Variable A

Ev
Criteria Variable B

Variable C

-X% 0 +X%

% varion from the base case

Figure 9.1: Sample spider plot.

Example 9.1
A ten-year life project has an initial investment of $87,500, annual operating expenses of
$7,500, and annual incomes of $30,000. It is desired to conduct a range approach sensitivity analysis
by the two methods described earlier:
9.1. INTRODUCTION 173
(a) Determine the most likely, the most optimistic, and the most pessimistic values for the I RR
by assuming the values given are the mean values for each parameter and that each parameter
has a range of 20% from the mean value.

(b) Vary each parameter independently by 20% from the mean values while holding the other
two constant at their mean values and develop a spider plot for the calculated I RR values.

The cash flow diagram for the most likely (or base case) is as follows:

0 1 2 3 8 9 10

-87,500 22,500 22,500 22,500 ---- 22,500 22,500 22,500

Solution for (a):


The I RR for the most likely case is computed from the cash flow above to be 22.3%.
The most pessimistic case would be the combination of these three variables that would have
the most negative influence on the projects I RR. This would be a 20% increase in initial investment,
a 20% increase in operating expenses, and a 20% decrease in income.
The most optimistic case would be the combination of these three variables that would have
the most positive influence on the projects IRR. This would be a 20% decrease in initial investment,
a 20% decrease in operating expenses, and a 20% increase in income.

Solution for (b):


In this analysis, start with the most likely case from (a) and denote that as the base case. Then
vary one parameter at a time by plus and minus 20% and recalculate I RR for each new cash flow
diagram.
174 9. SENSITIVITY ANALYSIS

Spider Plot
35
30
25
20
IRR Inial Investment
15
Operang Expenses
10
Annual Income
5
0
-25 -15 -5 5 15 25
% son from Base Case

From the spider plot, it can be readily seen that the parameter which has the greatest effect on the
I RR is the annual income. A larger change in I RR is observed for the same percentage change in the
annual income than for initial investment or operating expenses. Changes in the operating expenses
have the least effect on the I RR. To minimize the risk that would be created by a 20% uncertainty
in the annual income, it would be beneficial for the evaluator to do additional research into this
portion of the cash flow calculation and determine if the range in uncertainty can be reduced.
In the example problem, only two values for each variable have been used.To be more complete,
several values between 20% and +20% could have been used to generate additional values of I RR.
9.1. INTRODUCTION 175
The primary drawback on the spider plot approach is that it ignores the interactions that occur
when more than one variable is allowed to change at a time. Not only does one have to define many
more cases to account for all possible interactions, but it is also difficult to tabulate these results in a
meaningful way. Often, the results simply become a tabulated list of I RRs for each case evaluated.
For example, consider a problem that has four variables that have uncertain values and that three
numerical values for each of the variables are chosen in the range approach. This will result in 81
(3x3x3x3) individual solutions to the problem that must be presented to evaluate the effect of all
four variables. It may be very difficult for the evaluator to draw any conclusions from a long tabular
list of 81 results.

9.1.2 MONTE CARLO SIMULATION


Probabilistic sensitivity analysis or Monte Carlo Simulation (MCS) was introduced in the early
1960s. While it is a very powerful technique, it is often avoided due to the general lack of knowledge
among engineers and managers on how it works and the conclusions that can be drawn from it. This
section will attempt to explain the technique and its benefits.
Consider the example proposed above that contains four variables with uncertain values.
Instead of creating a tabular list of 81 evaluator-chosen cases, MCS allows each of the variables to vary
between minimum and maximum values according to some prescribed probability distribution and
then solves the problem for a large, randomized set of these input variables. The results of the MCS
are presented graphically as a cumulative probability distribution of the dependent variable (e.g.,
I RR, N P V , etc). This probability can then be interpreted using statistical methods to determine
the likelihood of a particular solution occurring or not occurring.
Distributions that are frequently used are the uniform, triangular, and normal distributions
presented in Chapter 8. These are fairly easy to describe mathematically and to input into an Excel
spreadsheet or a computer program. The choice of the particular distribution for a certain variable
should be guided by the evaluators knowledge of that variable.
Within the MCS method, the selection of a value for an independent variable is accomplished
by using the fact that the cumulative probability distribution, F (x), will lie between zero and one
and will be monotonic in behavior. Thus, the selection of a random number between zero and one
will yield a distinct random value for the independent variable between the variables minimum and
maximum values. A different random number is chosen for each independent variable, resulting in
a truly random set of independent variable values. Once the values of all independent variables have
been determined, the dependent variable, i.e., an evaluation criteria such as I RR or NP V , can be
calculated. This process is then repeated a large number of times and the results of the dependent
variable calculations are grouped into class intervals and then the cumulative probability distribution
is constructed.
The following is a summary of the steps involved in the MCS method:

1. Select the independent variables that contain uncertainty in their values.


176 9. SENSITIVITY ANALYSIS
2. Estimate the minimum and maximum values for each independent variable.

3. Estimate the minimum and maximum values for the dependent variable and set up class
intervals in that range such that a probability distribution can be generated.

4. Select a probability distribution that best describes the behavior of each independent variable
between its minimum and maximum values.

5. Set up equations which will allow for the calculation of each of the independent variables.
This is done by determining expressions for the cumulative probability distributions, F (x),
for each independent variable and then solving this expression for the variable, x.

6. Generate a random number for each independent variable. A different random number is
determined for each independent variable. Random numbers are available from scientific
calculators, Excel , or by using Table 9.1. One can enter this table at any random point and
then proceed through the table either by rows or columns. Excel uses the =RAND() function
to generate a uniformly distributed random number between zero and one.

7. Use the random numbers to calculate the values for the independent variables using the
equations developed in step 5.

8. Calculate the dependent variable (or variables if necessary) for this set of independent variables
and increment a counter in the respective class interval.

9. Return to step 6 and repeat steps 6 through 8 a relatively large number of times. A large
number of trials might be 100, 1000, or 10,000 depending on the sensitivity of the dependent
variable to the independent variables.

10. Construct the cumulative probability distribution for the dependent variable.
177
9.1. INTRODUCTION

Table 9.1: Random numbers


178 9. SENSITIVITY ANALYSIS
Example 9.2
For the problem described in Example 9.1 and the distributions given below, conduct a Monte
Carlo Simulation and summarize the results in a cumulative probability distribution. Complete 10
cases using I RR as the dependent variable.

Initial Investment:

/al Investment
0.00003

0.000025

0.00002

p(x) 0.000015

0.00001

0.000005

0
60000 70000 80000 90000 100000 110000
Dollars

Operating Expenses:

Operng Expenses
0.0007
0.0006
0.0005
0.0004
p(x)
0.0003
0.0002
0.0001
0
5000 6000 7000 8000 9000 10000
Dollars/Year
9.1. INTRODUCTION 179
Annual Income:

Annual Income
0.00009
0.00008
0.00007
0.00006
0.00005
p(x)
0.00004
0.00003
0.00002
0.00001
0
23000 25000 27000 29000 31000 33000 35000 37000
Dollars/Year

Solution:

Develop equations for the independent variables:


Let x1 be the value for the initial investment. Since it has a uniform distribution, its cumulative
probability (F1 ) is given by

F1 = (x1 a)/(b a) = (x1 70000)/(105000 70000) = (x1 70000)/35000

Solving this equation for x1 yields

x1 = 35000F1 + 70000 (9.1)

Let x2 be the value for the operating expenses. Since it has a triangular distribution, its
cumulative probability (F2 ) is given by

P1 = (b a)/(c a) = (7500 6000)/(9000 6000) = 0.5


P2 = 1 P1 = 0.5

For 6000 x2 7500 or F2 0.5 (the value of P1 ),

F2 = P1 [(x2 a)/(b a)]2 = 0.5 [(x2 6000)/1500]2

Solving this equation for x2 yields



x2 = 6000 + 1500 2F2 (9.2)

For 7500 x2 9000 or F2 0.5 (the value of P1 ),

F2 = 1 P2 [(c x2 )/(c b)]2 = 1 0.5 [(9000 x2 )/1500]2


180 9. SENSITIVITY ANALYSIS
Solving this equation for x2 yields

x2 = 9000 1500 2(1 F2 ) (9.3)

Let x3 be the value for the annual income. Since it has a uniform distribution, its cumulative
probability (F3 ) is given by
F3 = (x3 a)/(b a) = (x3 24000)/(36000 24000) = (x3 24000)/12000

Solving this equation for x3 yields


x3 = 12000F1 + 24000 (9.4)

First iteration:
Choose the first random number from the table. This will be the value for F1 . (F1 = 0.90535).
Use this number in Equation 9.1 to determine the value to be used for the initial investment:
x1 = 35000(0.90535) + 70000 = 101, 700

Choose the second random number from the table. This will be the value for F2 . (F2 =
0.86245). Since F2 0.5, use this number in Equation 9.3 to determine the value to be used
for the operating expense:

x2 = 9000 1500 2(1 0.86245) = 8, 200

Choose the third random number from the table.This will be the value for F3 . (F3 = 0.32775).
Use this number in Equation 9.4 to determine the value to be used for the annual income:
x3 = 12000(0.32775) + 24000 = 27, 900

These values for initial investment, operating expense, and annual income yield the following
cash flow diagram for the project:

0 1 2 3 8 9 10

-101,700 19,700 19,700 19,700 ---- 19,700 19,700 19,700

I RR analysis yields a value of 14.3%. This value is tabulated in a list for further processing.
Second and successive iterations:
Follow the same procedure as listed for the first iteration. Three new random numbers are
used during each iteration. The results of the first ten iterations are shown in the table below.
9.1. INTRODUCTION 181

The cumulative probability distribution for I RR can be developed from information in the
following table:
182 9. SENSITIVITY ANALYSIS

Interval Mid-point Frequency Prob CumulProb


10.5-12.5 11.5 0 0.0 0.0
12.5-14.5 13.5 2 20.0 20.0
14.5-16.5 15.5 1 30.0 30.0
16.5-18.5 17.5 0 0.0 30.0
18.5-20.5 19.5 0 0.0 30.0
20.5-22.5 21.5 4 40.0 70.0
22.5-24.5 23.5 0 0.0 70.0
24.5-26.5 25.5 2 20.0 90.0
26.5-28.5 27.5 0 0.0 90.0
28.5-30.5 29.5 0 0.0 90.0
30.5-32.5 31.5 0 0.0 90.0
32.5-34.5 33.5 1 10.0 100.0

Cumue Probability for IRR


100.0
90.0
80.0
70.0
60.0
Cumul Prob, % 50.0
40.0
30.0
20.0
10.0
0.0
10 15 20 25 30 35
IRR, %

With only ten iterations, this graph is too jagged to interpret correctly. The figure below
shows the same analysis after 100 iterations. One can see that the curve is much smoother. If even
more iterations are added, the curve will become smoother yet. However, the usefulness of the curve
may not increase proportionally to the number of iterations. One should only complete enough
iterations to get a reasonably smooth curve. Generally, this takes about 100 iterations, but this may
be a function of the actual problem being solved.This number of calculations can be easily completed
with Excel .
9.1. INTRODUCTION 183

Cumue Probability for IRR


100.0
90.0
80.0
70.0
60.0
F(x), % 50.0
40.0
30.0
20.0
10.0
0.0
10 15 20 25 30 35 40
IRR, %

As discussed in Chapter 8, the use of this graph is as follows. The value of F (x) at any I RR
is the probability that the project will attain that I RR or less. For example, there is approximately
an 18% probability that the project will earn an I RR value of less than 15%. Thus, if one uses
the investors MARR, F (x) provides the probability that the project will earn less than that value.
The quantity (100 F (x)) would provide the probability that the project earns greater than that
MARR. For example, if the investors MARR is 20%, one would enter the horizontal axis at 20%
and read a cumulative probability of about 44%. The interpretation would be that there is a 56%
probability that the project will yield a 20% I RR or greater. This probability is then a direct measure
of risk associated with the project. If an evaluator feels that a 44% probability that the project will
not be economically viable is an unacceptable level of risk, then the project should be eliminated
from further consideration. However, if, in this example, the investors MARR is only 15%, there is
only a 18% probability that the project will not be economically viable. This investor would have a
more acceptable level of risk.

Example 9.3
A particular investment has three uncertain variables of initial cost, future value, and the
investment life.
The initial cost can be described by a uniform distribution from $100 to $200.
The future value can be described by a normal distribution with = $300 and = $30.
The investment life can be described by a discrete probability distribution with 40% probability
that n = 5 years, 30% probability that n = 6 years, 20% probability that n = 7 years, and 10%
probability that n = 8 years.
184 9. SENSITIVITY ANALYSIS
Based on this information,

(a) Calculate the minimum rate of return that can be earned, the maximum rate of return that
can be earned and the mean rate of return that will be earned. For the normal distribution,
assume that the minimum and maximum values of future value will be 3 from the mean
($210 and $390, respectively).

(b) Complete a Monte Carlo Simulation for this project to determine the probability that the
ROR will be at least 15%.

Solution for (a):

The mean values for the three distributions are:

Pmean = (100 + 200)/2 = 150


Fmean = 300
nmean = 0.4 5 + 0.3 6 + 0.2 7 + 0.1 8 = 6.0

The rate of return, ROR, can be calculated using the relationship, F = P (1 + i)n or

i = (F /P )1/n 1

The three cases will be defined as follows:

Case /l Cost, P Future Ve, F Life, n ROR


Minimum ROR $200 $210 8 0.61%
(Most P

Dn ROR $150 $300 6 12.2%

Dmum ROR $100 $390 5 31.3%


(Most K

9.1. INTRODUCTION 185
Solution for (b):

Develop equations for the independent variables:

Let x1 be the value for the initial investment. Since it has a uniform distribution, its cumulative
probability (F1 ) is given by

F1 = (x1 a)/(b a) = (x1 100)/(200 100) = (x1 100)/100

Solving this equation for x1 yields

x1 = 100F1 + 100

Let x2 be the value for the future value. Since it has a normal distribution, its cumulative
probability (F2 ) is given by the values in Table 8.1. Once F2 (Z) is randomly chosen, the
appropriate value of Z is determined from Table 8.1 and then x2 = Z + .

Let x3 be the value for the project life. Since it has a discrete distribution, its cumulative
probability (F3 ) is given by

Cumve Probability for Project Life


1
0.9
0.8
5 x3 < 6 F3 = 0.4 0.7
0.6
6 x3 < 7 F3 = 0.7 F(x) 0.5
7 x3 < 8 F3 = 0.9 0.4
0.3
x3 8 F3 = 1.0 0.2
0.1
0
4 5 6 7 8 9
n, years

Thus, once F3 is randomly chosen, the value for x3 will be:

F3 0.4 x3 =5
0.4 < F3 0.7 x3 =6
0.7 < F3 0.9 x3 =7
0.9 < F3 1.0 x3 =8
186 9. SENSITIVITY ANALYSIS
The dependent variable is ROR which is calculated by using: ROR = (x2 /x1 )1/x3 1.
Each iteration can be calculated using the following Excel spreadsheet:

If one tabulates the result in cell B9 into another column of results (for example start in cell
A20), the spreadsheet will automatically select three new random numbers and a new result of
ROR will be calculated. This result would then be tabulated in cell A21. In order to generate
the histogram, this process is repeated manually 100 times (which would result in a column
of data from A20 to A119).
The results would be as follows:

Cumue Probability for ROR


100.0
90.0
80.0
70.0
60.0
F(x), % 50.0
40.0
30.0
20.0
10.0
0.0
0 5 10 15 20 25 30 35
ROR, %

At ROR of 15%, F (x) is approximately 74%. This means that there is a 74% probability that
the ROR will be less than 15% or a 26% probability that the ROR will be at least 15%.
9.2. PROBLEMS 187
9.2 PROBLEMS
9.1. You are to conduct an extensive sensitivity analysis on the problem described below. The
sensitivity analysis will consist of three parts:

(a) A range approach where the most optimistic, most likely, and most pessimistic values
of the dependent variables NP V and I RR are determined.
(b) A range approach where the mean value is determined for each independent variable
and then each variable is allowed to vary 20% about that mean while all other
independent variables are held constant. Create spider plots for NP V and I RR using
the results.
(c) A probabilistic approach using Monte Carlo Simulation. Complete 10 iterations and
create cumulative probability curves for NP V and I RR.

The project life is 7 years and the MARR is 15%.


The initial investment is given by a uniform distribution between $200,000 and $300,000.
The annual profit is given by a triangular distribution that has a minimum value of
$55,000/year, a mode of $67,500/year, and a maximum value of $85,000/year.
The salvage value of the investment is given by a triangular distribution that has a minimum
value of $60,000, a mode of $75,000, and a maximum value of $85,000.
The cash flow diagram would be:

0 1 2 3 6 7

-Invest Prot Prot Prot ---- Prot Prot +


Salvage

9.2. Complete Problem 9.1 using an Excel spreadsheet to calculate 100 iterations. Create
cumulative probability curves for NP V and I RR.

9.3. The following distributions are given for three independent variables, x1 , x2 , and x3 and the
relationship for the dependent variable, y. Calculate the largest, smallest, and mean values
of the dependent variable, y.

x1 : Uniform distribution between 35 and 50


x2 : Triangular distribution between 20 and 40 with a mode of 35
188 9. SENSITIVITY ANALYSIS
x3 : Discrete distribution with a 50% probability that the value will be 2 and 50%
probability that the value will be 4

y = (x1 )(x2 ) + x3

9.4. Using the information given in Problem 9.3, use Monte Carlo Simulation to calculate 10
iterations of the dependent variable and create the cumulative probability diagram for the
dependent variable y.
9.5. Complete Problem 9.4 using an Excel spreadsheet and 100 iterations.
9.6. Complete Problem 9.4 using an Excel spreadsheet and assuming that independent variable
x2 has a normal distribution with a mean of 30 and a standard deviation of 3. Compute 100
iterations and create the cumulative probability diagram for the dependent variable y.
189

APPENDIX A

Compound Interest Factors


190 APPENDIX A

i= 1%
n F/P P/F F/A A/F P/A A/P A/G
1 1.01000 0.99010 1.0000 1.00000 0.9901 1.01000 0.00000
2 1.02010 0.98030 2.0100 0.49751 1.9704 0.50751 0.49751
3 1.03030 0.97059 3.0301 0.33002 2.9410 0.34002 0.99337
4 1.04060 0.96098 4.0604 0.24628 3.9020 0.25628 1.48756
5 1.05101 0.95147 5.1010 0.19604 4.8534 0.20604 1.98010
6 1.06152 0.94205 6.1520 0.16255 5.7955 0.17255 2.47098
7 1.07214 0.93272 7.2135 0.13863 6.7282 0.14863 2.96020
8 1.08286 0.92348 8.2857 0.12069 7.6517 0.13069 3.44777
9 1.09369 0.91434 9.3685 0.10674 8.5660 0.11674 3.93367
10 1.10462 0.90529 10.4622 0.09558 9.4713 0.10558 4.41792
11 1.11567 0.89632 11.5668 0.08645 10.3676 0.09645 4.90052
12 1.12683 0.88745 12.6825 0.07885 11.2551 0.08885 5.38145
13 1.13809 0.87866 13.8093 0.07241 12.1337 0.08241 5.86073
14 1.14947 0.86996 14.9474 0.06690 13.0037 0.07690 6.33836
15 1.16097 0.86135 16.0969 0.06212 13.8651 0.07212 6.81433
16 1.17258 0.85282 17.2579 0.05794 14.7179 0.06794 7.28865
17 1.18430 0.84438 18.4304 0.05426 15.5623 0.06426 7.76131
18 1.19615 0.83602 19.6147 0.05098 16.3983 0.06098 8.23231
19 1.20811 0.82774 20.8109 0.04805 17.2260 0.05805 8.70167
20 1.22019 0.81954 22.0190 0.04542 18.0456 0.05542 9.16937
25 1.28243 0.77977 28.2432 0.03541 22.0232 0.04541 11.48312
30 1.34785 0.74192 34.7849 0.02875 25.8077 0.03875 13.75566
35 1.41660 0.70591 41.6603 0.02400 29.4086 0.03400 15.98711
40 1.48886 0.67165 48.8864 0.02046 32.8347 0.03046 18.17761
45 1.56481 0.63905 56.4811 0.01771 36.0945 0.02771 20.32730
50 1.64463 0.60804 64.4632 0.01551 39.1961 0.02551 22.43635
55 1.72852 0.57853 72.8525 0.01373 42.1472 0.02373 24.50495
60 1.81670 0.55045 81.6697 0.01224 44.9550 0.02224 26.53331
65 1.90937 0.52373 90.9366 0.01100 47.6266 0.02100 28.52167
70 2.00676 0.49831 100.6763 0.00993 50.1685 0.01993 30.47026
75 2.10913 0.47413 110.9128 0.00902 52.5871 0.01902 32.37934
80 2.21672 0.45112 121.6715 0.00822 54.8882 0.01822 34.24920
85 2.32979 0.42922 132.9790 0.00752 57.0777 0.01752 36.08013
90 2.44863 0.40839 144.8633 0.00690 59.1609 0.01690 37.87245
95 2.57354 0.38857 157.3538 0.00636 61.1430 0.01636 39.62648
100 2.70481 0.36971 170.4814 0.00587 63.0289 0.01587 41.34257
COMPOUND INTEREST FACTORS 191

i= 2%

n F/P P/F F/A A/F P/A A/P A/G


1 1.02000 0.98039 1.0000 1.00000 0.9804 1.02000 0.00000
2 1.04040 0.96117 2.0200 0.49505 1.9416 0.51505 0.49505
3 1.06121 0.94232 3.0604 0.32675 2.8839 0.34675 0.98680
4 1.08243 0.92385 4.1216 0.24262 3.8077 0.26262 1.47525
5 1.10408 0.90573 5.2040 0.19216 4.7135 0.21216 1.96040
6 1.12616 0.88797 6.3081 0.15853 5.6014 0.17853 2.44226
7 1.14869 0.87056 7.4343 0.13451 6.4720 0.15451 2.92082
8 1.17166 0.85349 8.5830 0.11651 7.3255 0.13651 3.39608
9 1.19509 0.83676 9.7546 0.10252 8.1622 0.12252 3.86805
10 1.21899 0.82035 10.9497 0.09133 8.9826 0.11133 4.33674
11 1.24337 0.80426 12.1687 0.08218 9.7868 0.10218 4.80213
12 1.26824 0.78849 13.4121 0.07456 10.5753 0.09456 5.26424
13 1.29361 0.77303 14.6803 0.06812 11.3484 0.08812 5.72307
14 1.31948 0.75788 15.9739 0.06260 12.1062 0.08260 6.17862
15 1.34587 0.74301 17.2934 0.05783 12.8493 0.07783 6.63090
16 1.37279 0.72845 18.6393 0.05365 13.5777 0.07365 7.07990
17 1.40024 0.71416 20.0121 0.04997 14.2919 0.06997 7.52564
18 1.42825 0.70016 21.4123 0.04670 14.9920 0.06670 7.96811
19 1.45681 0.68643 22.8406 0.04378 15.6785 0.06378 8.40732
20 1.48595 0.67297 24.2974 0.04116 16.3514 0.06116 8.84328
25 1.64061 0.60953 32.0303 0.03122 19.5235 0.05122 10.97445
30 1.81136 0.55207 40.5681 0.02465 22.3965 0.04465 13.02512
35 1.99989 0.50003 49.9945 0.02000 24.9986 0.04000 14.99613
40 2.20804 0.45289 60.4020 0.01656 27.3555 0.03656 16.88850
45 2.43785 0.41020 71.8927 0.01391 29.4902 0.03391 18.70336
50 2.69159 0.37153 84.5794 0.01182 31.4236 0.03182 20.44198
55 2.97173 0.33650 98.5865 0.01014 33.1748 0.03014 22.10572
60 3.28103 0.30478 114.0515 0.00877 34.7609 0.02877 23.69610
65 3.62252 0.27605 131.1262 0.00763 36.1975 0.02763 25.21471
70 3.99956 0.25003 149.9779 0.00667 37.4986 0.02667 26.66323
75 4.41584 0.22646 170.7918 0.00586 38.6771 0.02586 28.04344
80 4.87544 0.20511 193.7720 0.00516 39.7445 0.02516 29.35718
85 5.38288 0.18577 219.1439 0.00456 40.7113 0.02456 30.60635
90 5.94313 0.16826 247.1567 0.00405 41.5869 0.02405 31.79292
95 6.56170 0.15240 278.0850 0.00360 42.3800 0.02360 32.91889
100 7.24465 0.13803 312.2323 0.00320 43.0984 0.02320 33.98628
192 APPENDIX A

i= 3%
n F/P P/F F/A A/F P/A A/P A/G
1 1.03000 0.97087 1.0000 1.00000 0.9709 1.03000 0.00000
2 1.06090 0.94260 2.0300 0.49261 1.9135 0.52261 0.49261
3 1.09273 0.91514 3.0909 0.32353 2.8286 0.35353 0.98030
4 1.12551 0.88849 4.1836 0.23903 3.7171 0.26903 1.46306
5 1.15927 0.86261 5.3091 0.18835 4.5797 0.21835 1.94090
6 1.19405 0.83748 6.4684 0.15460 5.4172 0.18460 2.41383
7 1.22987 0.81309 7.6625 0.13051 6.2303 0.16051 2.88185
8 1.26677 0.78941 8.8923 0.11246 7.0197 0.14246 3.34496
9 1.30477 0.76642 10.1591 0.09843 7.7861 0.12843 3.80318
10 1.34392 0.74409 11.4639 0.08723 8.5302 0.11723 4.25650
11 1.38423 0.72242 12.8078 0.07808 9.2526 0.10808 4.70494
12 1.42576 0.70138 14.1920 0.07046 9.9540 0.10046 5.14850
13 1.46853 0.68095 15.6178 0.06403 10.6350 0.09403 5.58720
14 1.51259 0.66112 17.0863 0.05853 11.2961 0.08853 6.02104
15 1.55797 0.64186 18.5989 0.05377 11.9379 0.08377 6.45004
16 1.60471 0.62317 20.1569 0.04961 12.5611 0.07961 6.87421
17 1.65285 0.60502 21.7616 0.04595 13.1661 0.07595 7.29357
18 1.70243 0.58739 23.4144 0.04271 13.7535 0.07271 7.70812
19 1.75351 0.57029 25.1169 0.03981 14.3238 0.06981 8.11788
20 1.80611 0.55368 26.8704 0.03722 14.8775 0.06722 8.52286
25 2.09378 0.47761 36.4593 0.02743 17.4131 0.05743 10.47677
30 2.42726 0.41199 47.5754 0.02102 19.6004 0.05102 12.31407
35 2.81386 0.35538 60.4621 0.01654 21.4872 0.04654 14.03749
40 3.26204 0.30656 75.4013 0.01326 23.1148 0.04326 15.65016
45 3.78160 0.26444 92.7199 0.01079 24.5187 0.04079 17.15557
50 4.38391 0.22811 112.7969 0.00887 25.7298 0.03887 18.55751
55 5.08215 0.19677 136.0716 0.00735 26.7744 0.03735 19.86004
60 5.89160 0.16973 163.0534 0.00613 27.6756 0.03613 21.06742
65 6.82998 0.14641 194.3328 0.00515 28.4529 0.03515 22.18407
70 7.91782 0.12630 230.5941 0.00434 29.1234 0.03434 23.21454
75 9.17893 0.10895 272.6309 0.00367 29.7018 0.03367 24.16342
80 10.64089 0.09398 321.3630 0.00311 30.2008 0.03311 25.03534
85 12.33571 0.08107 377.8570 0.00265 30.6312 0.03265 25.83490
90 14.30047 0.06993 443.3489 0.00226 31.0024 0.03226 26.56665
95 16.57816 0.06032 519.2720 0.00193 31.3227 0.03193 27.23505
100 19.21863 0.05203 607.2877 0.00165 31.5989 0.03165 27.84445
COMPOUND INTEREST FACTORS 193

i= 4%
n F/P P/F F/A A/F P/A A/P A/G
1 1.04000 0.96154 1.0000 1.00000 0.9615 1.04000 0.00000
2 1.08160 0.92456 2.0400 0.49020 1.8861 0.53020 0.49020
3 1.12486 0.88900 3.1216 0.32035 2.7751 0.36035 0.97386
4 1.16986 0.85480 4.2465 0.23549 3.6299 0.27549 1.45100
5 1.21665 0.82193 5.4163 0.18463 4.4518 0.22463 1.92161
6 1.26532 0.79031 6.6330 0.15076 5.2421 0.19076 2.38571
7 1.31593 0.75992 7.8983 0.12661 6.0021 0.16661 2.84332
8 1.36857 0.73069 9.2142 0.10853 6.7327 0.14853 3.29443
9 1.42331 0.70259 10.5828 0.09449 7.4353 0.13449 3.73908
10 1.48024 0.67556 12.0061 0.08329 8.1109 0.12329 4.17726
11 1.53945 0.64958 13.4864 0.07415 8.7605 0.11415 4.60901
12 1.60103 0.62460 15.0258 0.06655 9.3851 0.10655 5.03435
13 1.66507 0.60057 16.6268 0.06014 9.9856 0.10014 5.45329
14 1.73168 0.57748 18.2919 0.05467 10.5631 0.09467 5.86586
15 1.80094 0.55526 20.0236 0.04994 11.1184 0.08994 6.27209
16 1.87298 0.53391 21.8245 0.04582 11.6523 0.08582 6.67200
17 1.94790 0.51337 23.6975 0.04220 12.1657 0.08220 7.06563
18 2.02582 0.49363 25.6454 0.03899 12.6593 0.07899 7.45300
19 2.10685 0.47464 27.6712 0.03614 13.1339 0.07614 7.83416
20 2.19112 0.45639 29.7781 0.03358 13.5903 0.07358 8.20912
25 2.66584 0.37512 41.6459 0.02401 15.6221 0.06401 9.99252
30 3.24340 0.30832 56.0849 0.01783 17.2920 0.05783 11.62743
35 3.94609 0.25342 73.6522 0.01358 18.6646 0.05358 13.11984
40 4.80102 0.20829 95.0255 0.01052 19.7928 0.05052 14.47651
45 5.84118 0.17120 121.0294 0.00826 20.7200 0.04826 15.70474
50 7.10668 0.14071 152.6671 0.00655 21.4822 0.04655 16.81225
55 8.64637 0.11566 191.1592 0.00523 22.1086 0.04523 17.80704
60 10.51963 0.09506 237.9907 0.00420 22.6235 0.04420 18.69723
65 12.79874 0.07813 294.9684 0.00339 23.0467 0.04339 19.49093
70 15.57162 0.06422 364.2905 0.00275 23.3945 0.04275 20.19614
75 18.94525 0.05278 448.6314 0.00223 23.6804 0.04223 20.82062
80 23.04980 0.04338 551.2450 0.00181 23.9154 0.04181 21.37185
85 28.04360 0.03566 676.0901 0.00148 24.1085 0.04148 21.85693
90 34.11933 0.02931 827.9833 0.00121 24.2673 0.04121 22.28255
95 41.51139 0.02409 1012.7846 0.00099 24.3978 0.04099 22.65498
100 50.50495 0.01980 1237.6237 0.00081 24.5050 0.04081 22.98000
194 APPENDIX A

i= 5%
n F/P P/F F/A A/F P/A A/P A/G
1 1.05000 0.95238 1.0000 1.00000 0.9524 1.05000 0.00000
2 1.10250 0.90703 2.0500 0.48780 1.8594 0.53780 0.48780
3 1.15763 0.86384 3.1525 0.31721 2.7232 0.36721 0.96749
4 1.21551 0.82270 4.3101 0.23201 3.5460 0.28201 1.43905
5 1.27628 0.78353 5.5256 0.18097 4.3295 0.23097 1.90252
6 1.34010 0.74622 6.8019 0.14702 5.0757 0.19702 2.35790
7 1.40710 0.71068 8.1420 0.12282 5.7864 0.17282 2.80523
8 1.47746 0.67684 9.5491 0.10472 6.4632 0.15472 3.24451
9 1.55133 0.64461 11.0266 0.09069 7.1078 0.14069 3.67579
10 1.62889 0.61391 12.5779 0.07950 7.7217 0.12950 4.09909
11 1.71034 0.58468 14.2068 0.07039 8.3064 0.12039 4.51444
12 1.79586 0.55684 15.9171 0.06283 8.8633 0.11283 4.92190
13 1.88565 0.53032 17.7130 0.05646 9.3936 0.10646 5.32150
14 1.97993 0.50507 19.5986 0.05102 9.8986 0.10102 5.71329
15 2.07893 0.48102 21.5786 0.04634 10.3797 0.09634 6.09731
16 2.18287 0.45811 23.6575 0.04227 10.8378 0.09227 6.47363
17 2.29202 0.43630 25.8404 0.03870 11.2741 0.08870 6.84229
18 2.40662 0.41552 28.1324 0.03555 11.6896 0.08555 7.20336
19 2.52695 0.39573 30.5390 0.03275 12.0853 0.08275 7.55690
20 2.65330 0.37689 33.0660 0.03024 12.4622 0.08024 7.90297
25 3.38635 0.29530 47.7271 0.02095 14.0939 0.07095 9.52377
30 4.32194 0.23138 66.4388 0.01505 15.3725 0.06505 10.96914
35 5.51602 0.18129 90.3203 0.01107 16.3742 0.06107 12.24980
40 7.03999 0.14205 120.7998 0.00828 17.1591 0.05828 13.37747
45 8.98501 0.11130 159.7002 0.00626 17.7741 0.05626 14.36444
50 11.46740 0.08720 209.3480 0.00478 18.2559 0.05478 15.22326
55 14.63563 0.06833 272.7126 0.00367 18.6335 0.05367 15.96645
60 18.67919 0.05354 353.5837 0.00283 18.9293 0.05283 16.60618
65 23.83990 0.04195 456.7980 0.00219 19.1611 0.05219 17.15410
70 30.42643 0.03287 588.5285 0.00170 19.3427 0.05170 17.62119
75 38.83269 0.02575 756.6537 0.00132 19.4850 0.05132 18.01759
80 49.56144 0.02018 971.2288 0.00103 19.5965 0.05103 18.35260
85 63.25435 0.01581 1245.0871 0.00080 19.6838 0.05080 18.63463
90 80.73037 0.01239 1594.6073 0.00063 19.7523 0.05063 18.87120
95 103.03468 0.00971 2040.6935 0.00049 19.8059 0.05049 19.06894
100 131.50126 0.00760 2610.0252 0.00038 19.8479 0.05038 19.23372
COMPOUND INTEREST FACTORS 195

i= 6%
n F/P P/F F/A A/F P/A A/P A/G
1 1.06000 0.94340 1.0000 1.00000 0.9434 1.06000 0.00000
2 1.12360 0.89000 2.0600 0.48544 1.8334 0.54544 0.48544
3 1.19102 0.83962 3.1836 0.31411 2.6730 0.37411 0.96118
4 1.26248 0.79209 4.3746 0.22859 3.4651 0.28859 1.42723
5 1.33823 0.74726 5.6371 0.17740 4.2124 0.23740 1.88363
6 1.41852 0.70496 6.9753 0.14336 4.9173 0.20336 2.33040
7 1.50363 0.66506 8.3938 0.11914 5.5824 0.17914 2.76758
8 1.59385 0.62741 9.8975 0.10104 6.2098 0.16104 3.19521
9 1.68948 0.59190 11.4913 0.08702 6.8017 0.14702 3.61333
10 1.79085 0.55839 13.1808 0.07587 7.3601 0.13587 4.02201
11 1.89830 0.52679 14.9716 0.06679 7.8869 0.12679 4.42129
12 2.01220 0.49697 16.8699 0.05928 8.3838 0.11928 4.81126
13 2.13293 0.46884 18.8821 0.05296 8.8527 0.11296 5.19198
14 2.26090 0.44230 21.0151 0.04758 9.2950 0.10758 5.56352
15 2.39656 0.41727 23.2760 0.04296 9.7122 0.10296 5.92598
16 2.54035 0.39365 25.6725 0.03895 10.1059 0.09895 6.27943
17 2.69277 0.37136 28.2129 0.03544 10.4773 0.09544 6.62397
18 2.85434 0.35034 30.9057 0.03236 10.8276 0.09236 6.95970
19 3.02560 0.33051 33.7600 0.02962 11.1581 0.08962 7.28673
20 3.20714 0.31180 36.7856 0.02718 11.4699 0.08718 7.60515
25 4.29187 0.23300 54.8645 0.01823 12.7834 0.07823 9.07220
30 5.74349 0.17411 79.0582 0.01265 13.7648 0.07265 10.34221
35 7.68609 0.13011 111.4348 0.00897 14.4982 0.06897 11.43192
40 10.28572 0.09722 154.7620 0.00646 15.0463 0.06646 12.35898
45 13.76461 0.07265 212.7435 0.00470 15.4558 0.06470 13.14129
50 18.42015 0.05429 290.3359 0.00344 15.7619 0.06344 13.79643
55 24.65032 0.04057 394.1720 0.00254 15.9905 0.06254 14.34112
60 32.98769 0.03031 533.1282 0.00188 16.1614 0.06188 14.79095
65 44.14497 0.02265 719.0829 0.00139 16.2891 0.06139 15.16012
70 59.07593 0.01693 967.9322 0.00103 16.3845 0.06103 15.46135
75 79.05692 0.01265 1300.9487 0.00077 16.4558 0.06077 15.70583
80 105.79599 0.00945 1746.5999 0.00057 16.5091 0.06057 15.90328
85 141.57890 0.00706 2342.9817 0.00043 16.5489 0.06043 16.06202
90 189.46451 0.00528 3141.0752 0.00032 16.5787 0.06032 16.18912
95 253.54625 0.00394 4209.1042 0.00024 16.6009 0.06024 16.29050
100 339.30208 0.00295 5638.3681 0.00018 16.6175 0.06018 16.37107
196 APPENDIX A

i= 7%
n F/P P/F F/A A/F P/A A/P A/G
1 1.07000 0.93458 1.0000 1.00000 0.9346 1.07000 0.00000
2 1.14490 0.87344 2.0700 0.48309 1.8080 0.55309 0.48309
3 1.22504 0.81630 3.2149 0.31105 2.6243 0.38105 0.95493
4 1.31080 0.76290 4.4399 0.22523 3.3872 0.29523 1.41554
5 1.40255 0.71299 5.7507 0.17389 4.1002 0.24389 1.86495
6 1.50073 0.66634 7.1533 0.13980 4.7665 0.20980 2.30322
7 1.60578 0.62275 8.6540 0.11555 5.3893 0.18555 2.73039
8 1.71819 0.58201 10.2598 0.09747 5.9713 0.16747 3.14654
9 1.83846 0.54393 11.9780 0.08349 6.5152 0.15349 3.55174
10 1.96715 0.50835 13.8164 0.07238 7.0236 0.14238 3.94607
11 2.10485 0.47509 15.7836 0.06336 7.4987 0.13336 4.32963
12 2.25219 0.44401 17.8885 0.05590 7.9427 0.12590 4.70252
13 2.40985 0.41496 20.1406 0.04965 8.3577 0.11965 5.06484
14 2.57853 0.38782 22.5505 0.04434 8.7455 0.11434 5.41673
15 2.75903 0.36245 25.1290 0.03979 9.1079 0.10979 5.75829
16 2.95216 0.33873 27.8881 0.03586 9.4466 0.10586 6.08968
17 3.15882 0.31657 30.8402 0.03243 9.7632 0.10243 6.41102
18 3.37993 0.29586 33.9990 0.02941 10.0591 0.09941 6.72247
19 3.61653 0.27651 37.3790 0.02675 10.3356 0.09675 7.02418
20 3.86968 0.25842 40.9955 0.02439 10.5940 0.09439 7.31631
25 5.42743 0.18425 63.2490 0.01581 11.6536 0.08581 8.63910
30 7.61226 0.13137 94.4608 0.01059 12.4090 0.08059 9.74868
35 10.67658 0.09366 138.2369 0.00723 12.9477 0.07723 10.66873
40 14.97446 0.06678 199.6351 0.00501 13.3317 0.07501 11.42335
45 21.00245 0.04761 285.7493 0.00350 13.6055 0.07350 12.03599
50 29.45703 0.03395 406.5289 0.00246 13.8007 0.07246 12.52868
55 41.31500 0.02420 575.9286 0.00174 13.9399 0.07174 12.92146
60 57.94643 0.01726 813.5204 0.00123 14.0392 0.07123 13.23209
65 81.27286 0.01230 1146.7552 0.00087 14.1099 0.07087 13.47598
70 113.98939 0.00877 1614.1342 0.00062 14.1604 0.07062 13.66619
75 159.87602 0.00625 2269.6574 0.00044 14.1964 0.07044 13.81365
80 224.23439 0.00446 3189.0627 0.00031 14.2220 0.07031 13.92735
85 314.50033 0.00318 4478.5761 0.00022 14.2403 0.07022 14.01458
90 441.10298 0.00227 6287.1854 0.00016 14.2533 0.07016 14.08122
95 618.66975 0.00162 8823.8535 0.00011 14.2626 0.07011 14.13191
100 867.71633 0.00115 12381.6618 0.00008 14.2693 0.07008 14.17034
COMPOUND INTEREST FACTORS 197

i= 8%
n F/P P/F F/A A/F P/A A/P A/G
1 1.08000 0.92593 1.0000 1.00000 0.9259 1.08000 0.00000
2 1.16640 0.85734 2.0800 0.48077 1.7833 0.56077 0.48077
3 1.25971 0.79383 3.2464 0.30803 2.5771 0.38803 0.94874
4 1.36049 0.73503 4.5061 0.22192 3.3121 0.30192 1.40396
5 1.46933 0.68058 5.8666 0.17046 3.9927 0.25046 1.84647
6 1.58687 0.63017 7.3359 0.13632 4.6229 0.21632 2.27635
7 1.71382 0.58349 8.9228 0.11207 5.2064 0.19207 2.69366
8 1.85093 0.54027 10.6366 0.09401 5.7466 0.17401 3.09852
9 1.99900 0.50025 12.4876 0.08008 6.2469 0.16008 3.49103
10 2.15892 0.46319 14.4866 0.06903 6.7101 0.14903 3.87131
11 2.33164 0.42888 16.6455 0.06008 7.1390 0.14008 4.23950
12 2.51817 0.39711 18.9771 0.05270 7.5361 0.13270 4.59575
13 2.71962 0.36770 21.4953 0.04652 7.9038 0.12652 4.94021
14 2.93719 0.34046 24.2149 0.04130 8.2442 0.12130 5.27305
15 3.17217 0.31524 27.1521 0.03683 8.5595 0.11683 5.59446
16 3.42594 0.29189 30.3243 0.03298 8.8514 0.11298 5.90463
17 3.70002 0.27027 33.7502 0.02963 9.1216 0.10963 6.20375
18 3.99602 0.25025 37.4502 0.02670 9.3719 0.10670 6.49203
19 4.31570 0.23171 41.4463 0.02413 9.6036 0.10413 6.76969
20 4.66096 0.21455 45.7620 0.02185 9.8181 0.10185 7.03695
25 6.84848 0.14602 73.1059 0.01368 10.6748 0.09368 8.22538
30 10.06266 0.09938 113.2832 0.00883 11.2578 0.08883 9.18971
35 14.78534 0.06763 172.3168 0.00580 11.6546 0.08580 9.96107
40 21.72452 0.04603 259.0565 0.00386 11.9246 0.08386 10.56992
45 31.92045 0.03133 386.5056 0.00259 12.1084 0.08259 11.04465
50 46.90161 0.02132 573.7702 0.00174 12.2335 0.08174 11.41071
55 68.91386 0.01451 848.9232 0.00118 12.3186 0.08118 11.69015
60 101.25706 0.00988 1253.2133 0.00080 12.3766 0.08080 11.90154
65 148.77985 0.00672 1847.2481 0.00054 12.4160 0.08054 12.06016
70 218.60641 0.00457 2720.0801 0.00037 12.4428 0.08037 12.17832
75 321.20453 0.00311 4002.5566 0.00025 12.4611 0.08025 12.26577
80 471.95483 0.00212 5886.9354 0.00017 12.4735 0.08017 12.33013
85 693.45649 0.00144 8655.7061 0.00012 12.4820 0.08012 12.37725
90 1018.91509 0.00098 12723.9386 0.00008 12.4877 0.08008 12.41158
95 1497.12055 0.00067 18701.5069 0.00005 12.4917 0.08005 12.43650
100 2199.76126 0.00045 27484.5157 0.00004 12.4943 0.08004 12.45452
198 APPENDIX A

i= 9%
n F/P P/F F/A A/F P/A A/P A/G
1 1.09000 0.91743 1.0000 1.00000 0.9174 1.09000 0.00000
2 1.18810 0.84168 2.0900 0.47847 1.7591 0.56847 0.47847
3 1.29503 0.77218 3.2781 0.30505 2.5313 0.39505 0.94262
4 1.41158 0.70843 4.5731 0.21867 3.2397 0.30867 1.39250
5 1.53862 0.64993 5.9847 0.16709 3.8897 0.25709 1.82820
6 1.67710 0.59627 7.5233 0.13292 4.4859 0.22292 2.24979
7 1.82804 0.54703 9.2004 0.10869 5.0330 0.19869 2.65740
8 1.99256 0.50187 11.0285 0.09067 5.5348 0.18067 3.05117
9 2.17189 0.46043 13.0210 0.07680 5.9952 0.16680 3.43123
10 2.36736 0.42241 15.1929 0.06582 6.4177 0.15582 3.79777
11 2.58043 0.38753 17.5603 0.05695 6.8052 0.14695 4.15096
12 2.81266 0.35553 20.1407 0.04965 7.1607 0.13965 4.49102
13 3.06580 0.32618 22.9534 0.04357 7.4869 0.13357 4.81816
14 3.34173 0.29925 26.0192 0.03843 7.7862 0.12843 5.13262
15 3.64248 0.27454 29.3609 0.03406 8.0607 0.12406 5.43463
16 3.97031 0.25187 33.0034 0.03030 8.3126 0.12030 5.72446
17 4.32763 0.23107 36.9737 0.02705 8.5436 0.11705 6.00238
18 4.71712 0.21199 41.3013 0.02421 8.7556 0.11421 6.26865
19 5.14166 0.19449 46.0185 0.02173 8.9501 0.11173 6.52358
20 5.60441 0.17843 51.1601 0.01955 9.1285 0.10955 6.76745
25 8.62308 0.11597 84.7009 0.01181 9.8226 0.10181 7.83160
30 13.26768 0.07537 136.3075 0.00734 10.2737 0.09734 8.66566
35 20.41397 0.04899 215.7108 0.00464 10.5668 0.09464 9.30829
40 31.40942 0.03184 337.8824 0.00296 10.7574 0.09296 9.79573
45 48.32729 0.02069 525.8587 0.00190 10.8812 0.09190 10.16029
50 74.35752 0.01345 815.0836 0.00123 10.9617 0.09123 10.42952
55 114.40826 0.00874 1260.0918 0.00079 11.0140 0.09079 10.62614
60 176.03129 0.00568 1944.7921 0.00051 11.0480 0.09051 10.76832
65 270.84596 0.00369 2998.2885 0.00033 11.0701 0.09033 10.87023
70 416.73009 0.00240 4619.2232 0.00022 11.0844 0.09022 10.94273
75 641.19089 0.00156 7113.2321 0.00014 11.0938 0.09014 10.99396
80 986.55167 0.00101 10950.5741 0.00009 11.0998 0.09009 11.02994
85 1517.93203 0.00066 16854.8003 0.00006 11.1038 0.09006 11.05508
90 2335.52658 0.00043 25939.1842 0.00004 11.1064 0.09004 11.07256
95 3593.49715 0.00028 39916.6350 0.00003 11.1080 0.09003 11.08467
100 5529.04079 0.00018 61422.6755 0.00002 11.1091 0.09002 11.09302
COMPOUND INTEREST FACTORS 199

i= 10%

n F/P P/F F/A A/F P/A A/P A/G


1 1.10000 0.90909 1.0000 1.00000 0.9091 1.10000 0.00000
2 1.21000 0.82645 2.1000 0.47619 1.7355 0.57619 0.47619
3 1.33100 0.75131 3.3100 0.30211 2.4869 0.40211 0.93656
4 1.46410 0.68301 4.6410 0.21547 3.1699 0.31547 1.38117
5 1.61051 0.62092 6.1051 0.16380 3.7908 0.26380 1.81013
6 1.77156 0.56447 7.7156 0.12961 4.3553 0.22961 2.22356
7 1.94872 0.51316 9.4872 0.10541 4.8684 0.20541 2.62162
8 2.14359 0.46651 11.4359 0.08744 5.3349 0.18744 3.00448
9 2.35795 0.42410 13.5795 0.07364 5.7590 0.17364 3.37235
10 2.59374 0.38554 15.9374 0.06275 6.1446 0.16275 3.72546
11 2.85312 0.35049 18.5312 0.05396 6.4951 0.15396 4.06405
12 3.13843 0.31863 21.3843 0.04676 6.8137 0.14676 4.38840
13 3.45227 0.28966 24.5227 0.04078 7.1034 0.14078 4.69879
14 3.79750 0.26333 27.9750 0.03575 7.3667 0.13575 4.99553
15 4.17725 0.23939 31.7725 0.03147 7.6061 0.13147 5.27893
16 4.59497 0.21763 35.9497 0.02782 7.8237 0.12782 5.54934
17 5.05447 0.19784 40.5447 0.02466 8.0216 0.12466 5.80710
18 5.55992 0.17986 45.5992 0.02193 8.2014 0.12193 6.05256
19 6.11591 0.16351 51.1591 0.01955 8.3649 0.11955 6.28610
20 6.72750 0.14864 57.2750 0.01746 8.5136 0.11746 6.50808
25 10.8347 0.09230 98.3471 0.01017 9.0770 0.11017 7.45798
30 17.4494 0.05731 164.494 0.00608 9.4269 0.10608 8.17623
35 28.1024 0.03558 271.024 0.00369 9.6442 0.10369 8.70860
40 45.2593 0.02209 442.593 0.00226 9.7791 0.10226 9.09623
45 72.8905 0.01372 718.905 0.00139 9.8628 0.10139 9.37405
50 117.391 0.00852 1163.91 0.00086 9.9148 0.10086 9.57041
55 189.059 0.00529 1880.59 0.00053 9.9471 0.10053 9.70754
60 304.482 0.00328 3034.82 0.00033 9.9672 0.10033 9.80229
65 490.371 0.00204 4893.71 0.00020 9.9796 0.10020 9.86718
70 789.747 0.00127 7887.47 0.00013 9.9873 0.10013 9.91125
200 APPENDIX A

i= 12%
n F/P P/F F/A A/F P/A A/P A/G
1 1.12000 0.89286 1.0000 1.00000 0.8929 1.12000 0.00000
2 1.25440 0.79719 2.1200 0.47170 1.6901 0.59170 0.47170
3 1.40493 0.71178 3.3744 0.29635 2.4018 0.41635 0.92461
4 1.57352 0.63552 4.7793 0.20923 3.0373 0.32923 1.35885
5 1.76234 0.56743 6.3528 0.15741 3.6048 0.27741 1.77459
6 1.97382 0.50663 8.1152 0.12323 4.1114 0.24323 2.17205
7 2.21068 0.45235 10.0890 0.09912 4.5638 0.21912 2.55147
8 2.47596 0.40388 12.2997 0.08130 4.9676 0.20130 2.91314
9 2.77308 0.36061 14.7757 0.06768 5.3282 0.18768 3.25742
10 3.10585 0.32197 17.5487 0.05698 5.6502 0.17698 3.58465
11 3.47855 0.28748 20.6546 0.04842 5.9377 0.16842 3.89525
12 3.89598 0.25668 24.1331 0.04144 6.1944 0.16144 4.18965
13 4.36349 0.22917 28.0291 0.03568 6.4235 0.15568 4.46830
14 4.88711 0.20462 32.3926 0.03087 6.6282 0.15087 4.73169
15 5.47357 0.18270 37.2797 0.02682 6.8109 0.14682 4.98030
16 6.13039 0.16312 42.7533 0.02339 6.9740 0.14339 5.21466
17 6.86604 0.14564 48.8837 0.02046 7.1196 0.14046 5.43530
18 7.68997 0.13004 55.7497 0.01794 7.2497 0.13794 5.64274
19 8.61276 0.11611 63.4397 0.01576 7.3658 0.13576 5.83752
20 9.64629 0.10367 72.0524 0.01388 7.4694 0.13388 6.02020
25 17.0001 0.05882 133.334 0.00750 7.8431 0.12750 6.77084
30 29.9599 0.03338 241.333 0.00414 8.0552 0.12414 7.29742
35 52.7996 0.01894 431.663 0.00232 8.1755 0.12232 7.65765
40 93.0510 0.01075 767.091 0.00130 8.2438 0.12130 7.89879
45 163.988 0.00610 1358.23 0.00074 8.2825 0.12074 8.05724
50 289.002 0.00346 2400.02 0.00042 8.3045 0.12042 8.15972
55 509.321 0.00196 4236.01 0.00024 8.3170 0.12024 8.22513
60 897.597 0.00111 7471.64 0.00013 8.3240 0.12013 8.26641
65 1581.87 0.00063 13173.9 0.00008 8.3281 0.12008 8.29222
70 2787.80 0.00036 23223.3 0.00004 8.3303 0.12004 8.30821
COMPOUND INTEREST FACTORS 201

i= 15%

n F/P P/F F/A A/F P/A A/P A/G


1 1.15000 0.86957 1.0000 1.00000 0.8696 1.15000 0.00000
2 1.32250 0.75614 2.1500 0.46512 1.6257 0.61512 0.46512
3 1.52088 0.65752 3.4725 0.28798 2.2832 0.43798 0.90713
4 1.74901 0.57175 4.9934 0.20027 2.8550 0.35027 1.32626
5 2.01136 0.49718 6.7424 0.14832 3.3522 0.29832 1.72281
6 2.31306 0.43233 8.7537 0.11424 3.7845 0.26424 2.09719
7 2.66002 0.37594 11.0668 0.09036 4.1604 0.24036 2.44985
8 3.05902 0.32690 13.7268 0.07285 4.4873 0.22285 2.78133
9 3.51788 0.28426 16.7858 0.05957 4.7716 0.20957 3.09223
10 4.04556 0.24718 20.3037 0.04925 5.0188 0.19925 3.38320
11 4.65239 0.21494 24.3493 0.04107 5.2337 0.19107 3.65494
12 5.35025 0.18691 29.0017 0.03448 5.4206 0.18448 3.90820
13 6.15279 0.16253 34.3519 0.02911 5.5831 0.17911 4.14376
14 7.07571 0.14133 40.5047 0.02469 5.7245 0.17469 4.36241
15 8.13706 0.12289 47.5804 0.02102 5.8474 0.17102 4.56496
16 9.35762 0.10686 55.7175 0.01795 5.9542 0.16795 4.75225
17 10.7613 0.09293 65.0751 0.01537 6.0472 0.16537 4.92509
18 12.3755 0.08081 75.8364 0.01319 6.1280 0.16319 5.08431
19 14.2318 0.07027 88.2118 0.01134 6.1982 0.16134 5.23073
20 16.3665 0.06110 102.444 0.00976 6.2593 0.15976 5.36514
25 32.9190 0.03038 212.793 0.00470 6.4641 0.15470 5.88343
30 66.2118 0.01510 434.745 0.00230 6.5660 0.15230 6.20663
35 133.176 0.00751 881.170 0.00113 6.6166 0.15113 6.40187
40 267.864 0.00373 1779.09 0.00056 6.6418 0.15056 6.51678
45 538.769 0.00186 3585.13 0.00028 6.6543 0.15028 6.58299
50 1083.66 0.00092 7217.72 0.00014 6.6605 0.15014 6.62048
55 2179.62 0.00046 14524.1 0.00007 6.6636 0.15007 6.64142
60 4384.00 0.00023 29220.0 0.00003 6.6651 0.15003 6.65298
65 8817.79 0.00011 58778.6 0.00002 6.6659 0.15002 6.65929
70 17735.7 0.00006 118231 0.00001 6.6663 0.15001 6.66272
202 APPENDIX A

i= 20%

n F/P P/F F/A A/F P/A A/P A/G


1 1.20000 0.83333 1.0000 1.00000 0.8333 1.20000 0.00000
2 1.44000 0.69444 2.2000 0.45455 1.5278 0.65455 0.45455
3 1.72800 0.57870 3.6400 0.27473 2.1065 0.47473 0.87912
4 2.07360 0.48225 5.3680 0.18629 2.5887 0.38629 1.27422
5 2.48832 0.40188 7.4416 0.13438 2.9906 0.33438 1.64051
6 2.98598 0.33490 9.9299 0.10071 3.3255 0.30071 1.97883
7 3.58318 0.27908 12.9159 0.07742 3.6046 0.27742 2.29016
8 4.29982 0.23257 16.4991 0.06061 3.8372 0.26061 2.57562
9 5.15978 0.19381 20.7989 0.04808 4.0310 0.24808 2.83642
10 6.19174 0.16151 25.9587 0.03852 4.1925 0.23852 3.07386
11 7.43008 0.13459 32.1504 0.03110 4.3271 0.23110 3.28929
12 8.91610 0.11216 39.5805 0.02526 4.4392 0.22526 3.48410
13 10.6993 0.09346 48.4966 0.02062 4.5327 0.22062 3.65970
14 12.8392 0.07789 59.1959 0.01689 4.6106 0.21689 3.81749
15 15.4070 0.06491 72.0351 0.01388 4.6755 0.21388 3.95884
16 18.4884 0.05409 87.4421 0.01144 4.7296 0.21144 4.08511
17 22.1861 0.04507 105.931 0.00944 4.7746 0.20944 4.19759
18 26.6233 0.03756 128.117 0.00781 4.8122 0.20781 4.29752
19 31.9480 0.03130 154.740 0.00646 4.8435 0.20646 4.38607
20 38.3376 0.02608 186.688 0.00536 4.8696 0.20536 4.46435
25 95.3962 0.01048 471.981 0.00212 4.9476 0.20212 4.73516
30 237.376 0.00421 1181.88 0.00085 4.9789 0.20085 4.87308
35 590.668 0.00169 2948.34 0.00034 4.9915 0.20034 4.94064
40 1469.77 0.00068 7343.86 0.00014 4.9966 0.20014 4.97277
45 3657.26 0.00027 18281.3 0.00005 4.9986 0.20005 4.98769
50 9100.44 0.00011 45497.2 0.00002 4.9995 0.20002 4.99451
55 22644.8 0.00004 113219 0.00001 4.9998 0.20001 4.99757
60 56347.5 0.00002 281733 0.00000 4.9999 0.20000 4.99894
COMPOUND INTEREST FACTORS 203

i= 25%
n F/P P/F F/A A/F P/A A/P A/G
1 1.25000 0.80000 1.0000 1.00000 0.8000 1.25000 0.00000
2 1.56250 0.64000 2.2500 0.44444 1.4400 0.69444 0.44444
3 1.95313 0.51200 3.8125 0.26230 1.9520 0.51230 0.85246
4 2.44141 0.40960 5.7656 0.17344 2.3616 0.42344 1.22493
5 3.05176 0.32768 8.2070 0.12185 2.6893 0.37185 1.56307
6 3.81470 0.26214 11.2588 0.08882 2.9514 0.33882 1.86833
7 4.76837 0.20972 15.0735 0.06634 3.1611 0.31634 2.14243
8 5.96046 0.16777 19.8419 0.05040 3.3289 0.30040 2.38725
9 7.45058 0.13422 25.8023 0.03876 3.4631 0.28876 2.60478
10 9.31323 0.10737 33.2529 0.03007 3.5705 0.28007 2.79710
11 11.6415 0.08590 42.5661 0.02349 3.6564 0.27349 2.96631
12 14.5519 0.06872 54.2077 0.01845 3.7251 0.26845 3.11452
13 18.1899 0.05498 68.7596 0.01454 3.7801 0.26454 3.24374
14 22.7374 0.04398 86.9495 0.01150 3.8241 0.26150 3.35595
15 28.4217 0.03518 109.687 0.00912 3.8593 0.25912 3.45299
16 35.5271 0.02815 138.109 0.00724 3.8874 0.25724 3.53660
17 44.4089 0.02252 173.636 0.00576 3.9099 0.25576 3.60838
18 55.5112 0.01801 218.045 0.00459 3.9279 0.25459 3.66979
19 69.3889 0.01441 273.556 0.00366 3.9424 0.25366 3.72218
20 86.7362 0.01153 342.945 0.00292 3.9539 0.25292 3.76673
25 264.698 0.00378 1054.79 0.00095 3.9849 0.25095 3.90519
30 807.794 0.00124 3227.17 0.00031 3.9950 0.25031 3.96282
35 2465.19 0.00041 9856.76 0.00010 3.9984 0.25010 3.98580
40 7523.16 0.00013 30088.7 0.00003 3.9995 0.25003 3.99468
45 22958.87 0.00004 91831.5 0.00001 3.9998 0.25001 3.99804
50 70064.92 0.00001 280256 0.00000 3.9999 0.25000 3.99929
55 213821.2 0.00000 855281 0.00000 4.0000 0.25000 3.99974
60 652530.4 0.00000 2610118 0.00000 4.0000 0.25000 3.99991
205

Authors Biographies

DAVID L. WHITMAN
David L. Whitman, P.E., Ph.D. received a B.S. degree (1975) in Electrical Engineering from the
University of Wyoming (UW). He also received a Ph.D. degree (1978) in Mineral Engineering
from the University of Wyoming. He worked in the synthetic fuels arena prior to becoming a faculty
member in Petroleum Engineering at the University of Wyoming in 1981. From 1989 to 2005,
he was the Associate Dean of Academics and since 2005 has been a professor of Electrical and
Computer Engineering. He received UWs Ellbogen Outstanding Teacher Award in 1985, UWs
College of Engineering Outstanding Undergraduate Teaching Award in 1990 and 2004 and the
ASEE Rocky Mountain Section Outstanding Teaching Award in 2001. He is a Past President
of the National Council of Examiners for Engineers and Surveyors (NCEES), chairman of the
IEEE-USA Licensure & Registration Committee, and an active member of ASEE.

RONALD E. TERRY
Ronald E. Terry, Ph.D. received a B.S. in Chemical Engineering from Oregon State University
(1971) and a Ph.D. from Brigham Young University (BYU) (1976). He worked for Phillips Petroleum
Company after graduate school and began his academic career in 1977 at the University of Kansas
in the Chemical and Petroleum Engineering Department. He taught in the Petroleum Engineering
Department at the University of Wyoming (1981-1987) and at BYU in the Chemical Engineering
Department (1987-2007) and in the Technology and Engineering Education Department (2007-
present). He has received teaching awards at the University of Kansas, University of Wyoming, and
at Brigham Young University.
Early in his career, his scholarship efforts involved researching methods to enhance the pro-
duction of oil and gas. After joining BYU, his scholarship centered on pedagogy, student learning,
and engineering ethics. He has served as acting department chair, associate dean, and in BYUs
central administration as an Associate in the Office of Planning and Assessment for five years
(2003-2008). He is past president of the Rocky Mountain Section of the American Society for
Engineering Education.

Anda mungkin juga menyukai